Biochemistry: Chapter 11-20

Lakukan tugas rumah & ujian kamu dengan baik sekarang menggunakan Quizwiz!

Match the organ with its specialized role: 1) brain A)continuous contraction 2) liver B) osmotic transport of solutes 3) heart C) biosynthesis 4) kidney D) control of bodily functions

!:D; 2:C; 3:A; 4:B

Identify the most likely additional substrates, products, and coenzymes for each reaction in the following imaginary pathway. Drag the terms on the left to the appropriate blanks on the right to complete the sentences. Not all terms will be used.

1. Additional substrate / product / coenzyme for reaction A is 5'-Deoxyadenosyl-B12 2. Additional substrate / product / coenzyme for reaction B is tetrahydrofolate. 3. Additional substrate / product / coenzyme for reaction C is ATP + glutamine 4. Additional substrate / product / coenzyme for reaction D is α-ketoglutarate + pyridoxal phosphate 5. Additional substrate / product / coenzyme for reaction E is S-adenosylmethionine.

Mutants of Neurospora crassa that lack carbamoyl phosphate synthetase I (CPSI) require arginine in the medium in order to grow, whereas mutants that lack carbamoyl-phosphate synthetase II (CPSII) require a pyrimidine, such as uracil. A priori, one would expect the active CPSII in the arginine mutants to provide sufficient carbamoyl phosphate for arginine synthesis, and the active CPSI in the pyrimidine mutants to "feed" the pyrimidine pathway. Explain these observations. Match the words in the left column to the appropriate blanks in the sentences on the right. Make certain each sentence is complete before submitting your answer.

1. CPS I is in MITOCHONDRIA, and CPS II is in CYTOSOL. 2. Carbamoyl phosphate CANNOT cross the mitochondrial membrane. 3. So the carbamoyl phosphate formed in mitochondria can be used only for ARGININE synthesis, and that formed in cytosol is used only for PYRIMIDINE synthesis.

Briefly describe how cyclic AMP controls phospholipid synthesis. Drag the terms on the left to the appropriate blanks on the right to complete the sentences.

1. Cyclic AMP promotes triacylglycerol breakdown, through activation of hormone-sensitive lipase. 2. This probably will increase intracellular levels of diacylglycerol, which could in turn increase flux through the last reactions in the salvage pathways to PE and PC.

Using the principles described in the text regarding pyridoxal phosphate mechanisms, propose a schema for the reaction catalyzed by serine hydroxymethyltransferase. Drag the stages on the left to the appropriate blanks on the right to complete the schema.

1. Displacement -> 2. Deprotonation -> 3. Reprotonation -> 4. Hydrolysis -> 5. Second reaction

Write a balanced equation for the conversion in the glyoxylate cycle of two acetyl units, as acetyl-CoA, to oxaloacetate.

2 acetyl-CoA + 2NAD+ + E-FAD + 3H2O -> oxaloacetate + 2NADH + E-FADH2 + 2CoA-SH + 2H+

Freshly prepared mitochondria were incubated with β-hydroxybutyrate, oxidized cytochrome c, ADP, Pi, and cyanide. β-hydroxybutyrate is oxidized by an NAD+-dependent dehydrogenase. The experimenter measured the rate of oxidation of β-hydroxybutyrate and the rate of formation of ATP. How many moles of ATP would you expect to be formed per mole of β-hydroxybutyrate oxidized in this system? Express your answer as an integer.

2 mol

Place the following sequence of events for a transamination reaction that requires pyridoxal phosphate in the correct order: 1) deprotonation by lysine at the α-carbon of the amino acid tethered to pyridoxal phosphate 2) exchange of the amino acid that makes the Schiff base, releasing free lysine 3) hydrolysis via a cabinolamine intermediate, yielding an α-ketoacid and pyridoxamine phosphate 4) reprotonation at the α-carbon, resulting in tautomerization of the imine carbon atom 5) a second α-ketoacid reacting with pyridoxamine to provide a new amino acid

2-1-4-3-5

Arrange the sequence of events for the conversion of succinyl-CoA to succinate and ATP (or GTP) in the correct order: 1) The phosphate group is transferred from a histidine residue of the phosphorylated enzyme intermediate to the nucleotide. 2) Inorganic phosphate reacts at the thioester of succinyl-CoA to yield a mixed phosphoanhydride. 3) A His residue from the protein is phosphorylated by succinyl phosphate.

2-3-1

Arrange the following sequence of events in the correct order for the Calvin cycle: 1) ATP is consumed in a reaction that yields 1,3-bisphosphoglycerate. 2) CO2 is fixed by reaction with ribulose-1,5-bisphosphate, catalyzed by Rubisco. 3) The unstable intermediate of this reaction is immediately cleaved into two molecules of 3-phosphogycerate. 4) NADPH is employed to prepare glyceraldehyde-3-phosphate. 5) Ribulose-1,5-bisphosphate is regenerated.

2-3-1-4-5

In Engelmann's experiment, he used aerotactic (oxygen-seeking) bacteria to determine which wavelengths of visible light were most effective in driving the reactions of photosynthesis in green algae. A diagram of his apparatus is shown below. Can you deduce the logical link between light of different wavelengths and the distribution of bacteria that Engelmann observed? Drag the labels onto the flowchart to show the relationship between the production of protons by the sun (Engelmann's light source) and the distribution of bacteria that Engelmann observed under his microscope. Not all labels will be used.

2. Prism disperses sunlight into individual wavelengths. 3. Alga's photosynthetic pigments absorb photons at specific wavelengths. 4. Absorbed photons drive photosynthesis in alga. 5. Alga gives off oxygen as it photosynthesizes. 6. Bacteria attracted to regions of highest oxygen concentration.

Methyl-labeled [14C]methionine at a specific activity of 2.0 millicuries per millimole was injected into rats. Six hours later the rats were killed. Phosphatidylcholine was isolated from the liver and found to have a specific activity of 1.5 millicuries per millimole. Calculate the proportions of phosphatidylcholine synthesized by the phosphatidylserine pathway. Express your answer as an integer.

25%

What is the standard free energy change per mol O2 produced?

273 kJ/mol O2

What proportion of the total energy consumption supports brain function in an average resting human? Express your answer using two significant figures.

28%

Calculate the amount of energy required to make one mole of hexose from CO2 and water by photosynthesis if the average photon has an energy of 2.77 x 10^-19 J and 48 photons are required for each 6C sugar. Assume the efficiency of photosynthesis is ~35%.

2802 kJ/mol

Write a balanced equation for the complete oxidation of oleic acid. Express your answer as a chemical equation.

2CH3(CH2)14(CH)2COOH + 51O2 -> 36CO2 + 34H2O

When pure reduced cytochrome c is added to carefully prepared mitochondria along with ADP, Pi, antimycin A, and oxygen, the cytochrome c becomes oxidized, and ATP is formed, with a P/O ratio approaching 1.0. Write a balanced equation for the overall reaction (including cyt c oxidation and ATP synthesis).

2Cytcred+12O2+ADP+Pi+2H+⟶2Cytcox+ATP+2H2O2

Label the steps of electron transport leading to oxidative phosphorylation where ATP is synthesized from ADP using the energy stored by the electron-transport chain. Drag the appropriate labels to their respective targets.

2nd box: Electron flow is accompanied by proton transfer into the intermembrane space. 4th box: Protons return to the matrix through a proton-specific channel. 5th box: Protons release potential energy, which drives the phosphorylation of ADP to ATP.

Arrange the sequence of events that occur in complex I in their proper order. 1)Fe+2 is oxidized to Fe+3. 2)FMN is reduced to FMNH2. 3)NADH is oxidized to NAD+. 4)CoQ is reduced to CoQH2.

3, 2, 1, 4

Arrange the steps of G-protein activation in the proper order. 1) cAMP synthesis results from the activation of adenylate cyclase. 2) GTP exchanges with bound GDP. 3) The signal binds to the extracellular component of the receptor, inducing a conformational change. 4) cAMP activates several kinases. 5) The G protein activates adenylate cyclase.

3, 2, 5, 1, 4

Place the following sequence of events for the reaction catalyzed by fructose-1,6-phosphate aldolase in the correct order: 1) deprotonation and cleavage to release GAP and enamine products 2) protonation of the enamine 3) conversion of the ketone to the imine derived from lysine 4) hydrolysis to release DHAP

3-1-2-4

In some instances, the levels of LDL cholesterol exceed the ability of the body to maintain homeostasis, and as a result, the excess material accumulates into fatty deposits that harden into plaques. Arrange in the correct order the sequence of events that leads to this condition, known as atherosclerosis: 1) The plaques harden, blocking the flow of blood in major arteries. 2) White blood cells become engorged with fatty deposits. 3) Excess LDL cholesterol accumulates in the interior of arterial walls. 4) Macrophages are drawn to this site.

3-4-2-1

Arrange the order of events in the fatty acid (FA) metabolism pathway in the correct order: 1) The modified FA is transported across the inner membrane of the mitochondrion. 2) Dehydration occurs to generate an α,β-unsaturated FA. 3) The free FA is converted to acetyl CoA ester. 4) Thiolytic cleavage occurs, releasing a molecule of acetyl CoA and a new FA with two fewer carbon atoms. 5) The FA CoA ester is converted to its carnitine ester. 6) Hydration occurs to yield a β-hydroxy ester. 7) The FA CoA ester is regenerated.

3-5-1-7-2-6-4

On her way to class this morning, the student stops at her favorite coffeehouse and grabs a caffe mocha and a blueberry scone. The caffe mocha contains 260 Calories and the scone contains 460 Calories. Convert the calories student consumed for breakfast into kilojoules. Express your answer to two significant figures and include the appropriate units.

3000 kJ

What is the total number of ATP molecules that can be produced from the complete oxidation of one glucose molecule? Express your answer numerically as an integere.

32

How much energy would be derived from the metabolic oxidation of 1 mole of isoleucine to CO2, H2O, AND NH3?

33.5 ATPs

Calculate the number of ATPs generated by the complete metabolic oxidation of tripalmitin (tripalmitoylglycerol). Hydrolysis of the triacylglycerol occurs at the cell surface. Consider the energy yield from catabolism of glycerol, as well as from the fatty acids. Express your answer using one decimal place.

336.5 ATPs

On her way to class this morning, the student stops at her favorite coffeehouse and grabs a caffe mocha and a blueberry scone. The caffe mocha contains 260 Calories and the scone contains 460 Calories. What fraction of the recommended daily caloric intake did she just consume? The estimated calorie the average calorie requirement for a moderately active 20-year-old female at 2100 Calories per day. Express your answer using two significant figures.

34%

How much energy would be derived from the metabolic oxidation of 1 mole of tyrosine to CO2, H2O, and NH3?

35.5 ATPs

Assume that 2.5 ATPs are generated per NADH and 1.5 ATPs per FADH2. How many ATPs are generated from the FADH2 and NADH molecules from each repetition of the ββbeta-oxidation pathway? Express your answer as an integer.

4 ATP(s)

How many moles of glucose are required to provide the carbon for the synthesis of one mole of palmitate? Express your answer as an integer.

4 mol of glucose

Arrange the events of glycogen synthesis in the proper order. 1) The receptor dimerizes. 2) Proteins, such as IRS-1, are recruited to the phosphorylated tyrosine domain of the receptor. 3) Autophosphorylation of the intracellular domain takes place. 4) Insulin binds to the insulin receptor. 5) IRS-1 is phosphorylated, which then activates phosphoinositide 3-kinase. 6) Protein kinase B is activated. 7) Phosphatidylinositol 4,5-bisphosphate is converted to phosphatidylinositol 3,4,5-trisphosphate. 8) Protein kinase B phosphorylates glycogen synthase kinase 3, inactivating it.

4, 1, 3, 2, 5, 7, 6, 8

Arrange the sequence of events for the overall mitochondrial respiratory assembly in the correct order. 1)Coenzyme Q-cytochrome c reductase passes electrons to cytochrome c. 2)Coenzyme Q releases its electrons to complex III. 3)Cytochrome oxidase catalyzes the reduction of O2 to water. 4)Succinate, through complex II, and NADH, via complex I, release electrons to coenzyme Q.

4, 2, 1, 3

Arrange the sequence of reactions that occur in the conversion of pyruvate to acetyl-CoA in the correct order: 1) The anion of the hydroxyethyl group attacks one sulfur of the disulfide in lipoic acid, and the resulting intermediate is oxidized to an acetyl thioester. 2) FAD is reduced to FADH2, while the two thiols of reduced lipoic acid are reoxidized back to a disulfide. 3) FADH2 is oxidized by NAD+. 4) Thiamine pyrophosphate decarboxylates pyruvate to yield hydroxyethyl-TPP. 5) The thioester derived from lipoic acid transfers its acetyl group to CoA.

4-1-5-2-3

Place the following sequence of events for the conversion of GAP to 1,3-bisphosphoglycerate in the correct order: 1) attack by Pi 2) release of cysteine as a free thiol 3) transfer of hydride to NAD+ 4) attack by cysteine at the carbonyl carbon of GAP

4-3-1-2

Starting with carbamoyl phosphate, place the sequence of events for the urea cycle in the correct order: 1) Argininosuccinase hydrolyzes its substrate to arginine and fumarate. 2) Ornithine and urea are the products of the reaction catalyzed by arginase. 3) Citrulline is combined with aspartate in the presence of argininosuccinate synthetase to provide argininosuccinate. 4) Ornithine transcarbamoylase catalyzes the transfer of a carbamoyl group to ornithine.

4-3-1-2

The β-oxidation of odd-numbered FAs presents a small problem in that once most of the oxidation has occurred, the last component is propionyl-CoA rather than acetyl CoA. This has one more carbon and as such cannot be utilized any further. Outline the events that control the metabolism of this unit in their proper order: 1) Methylmalonyl-CoA epimerase converts S-methylmalonyl-CoA to R-methylmalonyl-CoA. 2) Methylmalonyl-CoA mutase demethylates R-methylmalonyl-CoA to succinyl-CoA. 3) Propionyl-CoA carboxylase catalyzes the addition of CO2 to propionyl-CoA. 4) The odd-carbon FA is metabolized to propionyl-CoA.

4-3-1-2

Place the following events in the correct order for the two-photosystem light reactions. 1) Electrons are transferred to the b6F complex with concomitant influx of protons into the thylakoid lumen. 2) Electrons are passed through to ferredoxin and then ultimately to NADP+, reducing the latter to NADPH. 3) Electrons are transferred to plastoquinones, which serve as intermediate electron transporters. 4) Light energy is absorbed by photosystem II, placing it in an excited state, causing it to release electrons, which are replaced by the oxidation of water to oxygen. 5) Electrons are transferred to photosystem I via plastocyanin.

4-3-1-5-2

How many moles of glucose are required if all of the glucose first proceeds through the pentose phosphate pathway before proceeding through the rest of glycolysis on its way to pyruvate? Express your answer using one decimal place.

4.8 mol of glucose

Consider an ATP synthase complex with 12 "c" subunits in its F0 rotor (as illustrated in the molecular model of the E. coli enzyme in section II of the tutorial). How many protons must translocate through the F0 complex in order to generate 10 molecules of ATP?

40

Approximately what wavelength of light is best absorbed by chlorophyll a, the pigment that participates directly in the light reactions?

435 nm

Which wavelength of light is best absorbed by chlorophyll b?

455 nm

Use numbers 1 to 5 to identify each carbon atom in the product of this reaction. Drag the appropriate labels to their respective targets.

5 4 2 1 3

Using Figure 16.9 on page 515 in the text, arrange the order of events for the internalization of LDL particles into cells: 1) A combination of cholesterol and apolipoprotein binds to LDL receptors and becomes internalized as endocytotic vesicles. 2) The environment of the endosome becomes acidic, which causes the LDL to dissociate from its receptor; additionally, the endosome fuses with a lysosome. 3) Several endocytotic vesicles fuse to form an endosome. 4) The LDL particle is degraded by the lysosome. 5) LDL receptors migrate to the cell surface and cluster in clathrin-coated pits.

5-1-3-2-4

The following diagram shows the biosynthesis of B12 coenzymes, starting with the vitamin. DMB is dimethylbenzimidazole. What one additional substrate or cofactor is required by enzyme B?

5-Methyltetrahydrofolate

Adenine phosphoribosyltransferase converts adenine to AMP. If you were to determine whether a similar reaction converts adenine directly to dAMP, what metabolite would you need to find in cells at appreciable concentrations?

5-Phospho-2'-deoxyribosyl-1-pyrophosphate

For each glucose that enters glycolysis, ______ NADH+ H+ are produced by the citric acid cycle.

6

Calculate the number of molecules of acetyl-SCoA derived from a saturated fatty acid with 12 carbon atoms. Express your answer as an integer.

6 molecule(s)

Assume that 2.5 ATPs are generated per NADH and 1.5 ATPs per FADH2. What is the total number of ATPs generated from 6 acetyl-SCoA molecules? Express your answer as an integer.

60 ATP(s)

Calculate the amount of ATP in kg that is turned over by a resting human every 24 hours.Assume that a typical human contains ~50g of ATP (Mr 505) and consumes ~8000 kJ of energy in food each day. The energy stored in the terminal anhydride bond of ATP under standard conditions is 30.6 kJmol-1. Assume also that the dietary energy is channeled through ATP with an energy transfer efficiency of ~50%.

66 kg

Taking one mole of glucose through glycolysis and the citric acid cycle generates:

6C02, 10 NADH/H+, 2FADH2 and 4 ATP.

Which of these equations best summarizes photosynthesis?

6CO2 + 6H2O -> C6H12O6 + 6O2

How many tritium atoms (3H) are incorporated into palmitate when fatty acid synthesis is carried out in vitro with the following labeled substrate? Express your answer as an integer.

7 tritium atoms

Methyl-labeled [14C]methionine at a specific activity of 2.0 millicuries per millimole was injected into rats. Six hours later the rats were killed. Phosphatidylcholine was isolated from the liver and found to have a specific activity of 1.5 millicuries per millimole. Calculate the proportions of phosphatidylcholine synthesized by the pathway starting from free choline. Express your answer as an integer.

75%

Activation of the fatty acid (converting it to fatty acyl-SCoA) requires the expenditure of 2 ATPs. Use your answers from parts B and C to calculate the total number of ATPs generated from the metabolism of a saturated fatty acid with 12 carbon atoms including both the citric acid cycle and the ββbeta-oxidation pathway as well as the initial ATP required to produce the acyl-SCoA molecule that starts the process. Express your answer as an integer.

78 ATP(s)

The standard free energy change of the glycolytic pathway to pyruvate is -79.9 kJ/mol, while the standard free energy change associated with gluconeogenesis from pyruvate is -42.7 kJ/mol. What would the standard free energy change be for a direct reversal of the glycolytic pathway.

79.9 kJ/mol

The overall net synthesis of palmitate starting from acetyl CoA requires:

8 acetyl-CoA, 7 ATP and 14 NADPH and 7 H+

The flux of solar energy reaching the Earth's surface is approximately 7 J/s cm^2. Assume that all of this energy is used by a green leaf (10 cm^2 in area), with the maximal efficiency of 35%. How many moles of hexose could the leaf theoretically generate in an hour? Use 600 nm for an average wavelength. Express your answer using two significant figures.

9.2x10^-3 mol

Assimilation is indicated by the letter(s) _____.

A

BLANK splits water into 1/2 O2, H+, and e-.

A

Which folate structure (from the list below) Is the coenzyme for the thymidylate synthase reaction?

A

Chlorophyll can be found in BLANK.

A and C

Choose the correct balanced equations for the three known reactions that transfer an amino group to a substrate by condensation with aspartate to give an intermediate that then undergoes an α,β-elimination to give the product plus fumarate. Select all that apply.

- 4 − carboxy − 5 − aminoimidazoleribonucleotide + aspartate + ATP ⟶ N − succinylo − 5 − aminoimidazole − 4 − carboxamideribonucleotide + ADP + Pi - IMP + aspartate + GTP ⟶ adenylosuccinate + GDP + Pi - Citrulline + aspartate + ATP ⟶ argininosuccinate + AMP + PPi

Suppose that you wanted to determine the metabolic half-life of glutamine synthetase in HeLa cells (a line of human tumor cells) growing in tissue culture. Describe how this could be done experimentally. Check all stages of the the experiment. Check all that apply.

- At intervals following removal of the labeled amino acid (a pulse-chase experiment), prepare a cell-free protein extract, treat an aliquot with antiserum, and count the radioactivity in the immunoprecipitate. - Purify the enzyme from HeLa cells or another human tissue and prepare antiserum against the enzyme. - Label HeLa cell cultures by growth in a radioactive amino acid.

In the pyrimidine degradative pathway, all pyrimidines undergo conversion to uracil, which undergoes an NADPH-dependent reduction. Choose plausible reactions leading from cytidine, cytosine, and uridine to uracil. Select all that apply.

- Cytosine + H2O ⟶ uracil + NH3 - Uridine + Pi ⟶ uracil + ribose−1−phosphate - Cytidine + H2O ⟶ uridine + NH3

Glucagon secretion causes inhibition of intracellular acetyl-CoA carboxylase activity by several mechanisms. Name all you can think of. Check all that apply.

- Glucagon promotes triacylglycerol breakdown, yielding increased levels of fatty acids, which could prevent polymerization and activation of acetyl-CoA carboxylase. - Glucagon activates phosphorylation of pyruvate dehydrogenase, which inhibits the formation of acetyl-CoA, the substrate for the enzyme.

Choose necessary reactants for getting this intermediate. Check all that apply.

- H2O - H+

Essential amino acids must be obtained from the diet, whereas nonessential amino acids can be synthesized in the body. Which of the following statements about essential and nonessential amino acids are true? Check all that apply.

- Humans obtain phenylalanine from their diet. - Nonessential amino acids are synthesized by the human body. - About half of the 20 amino acids must be acquired from food. - Many of the nonessential amino acids are derived from other amino acids.

Consider either the purinosome or a multifunctional protein such as the CAD protein. Choose the evidence showing that the enzyme "channels" substrates through a multistep reaction sequence. Select all that apply.

- Intermediates in a pathway accumulate only to a low level, because they are rapidly transferred to the next enzyme in a sequence. - The product of a reaction sequence reaches its maximal rate of synthesis faster than in a nonchanneled pathway, because each intermediate need not diffuse to find the next enzyme and becomes rapidly bound to that enzyme. - Upon addition of one of the intermediates to a system carrying out the pathway then the added intermediate will not be incorporated into the product.

J. C. Servaites, in Plant Physiol. (1985) 78:839-843, observed that Rubisco from tobacco leaves collected before dawn had a much lower specific activity than the enzyme collected at noon. This difference persisted despite extensive dialysis, gel filtration, or heat treatment. However, precipitation of the predawn enzyme by 50%% (NH4) 2SO4 restored the specific activity to the level of the noon-collected enzyme. Suggest an explanation. Check all that apply.

- It indicates presence of a strong Rubisco inhibitor. - This substance has strong binding with the enzyme. - It is a low-molecular-weight phosphorylated compound.

Which of the following statements about the lipoproteins given in the introduction are true? Check all that apply.

- Lipoproteins are complexes of lipids and proteins. - Lipoproteins can be distinguished by their densities and composition. -Chylomicrons are the only lipoproteins that exclusively carry dietary lipids. - Lipoprotein densities vary because lipoproteins have different lipid-to-protein ratios.

Bisphenol A (BPA) is widely used as a building block in polymer synthesis and is found in the polycarbonate hard plastics of reusable drink containers, DVDs, cell phones, and other consumer goods. Bisphenol A is reported to have estrogenic activity, and its widespread occurrence in our environment is a potential concern. Choose all the suggestions that correctly describe biochemical experiments that could be done to compare the activity of bisphenol A with that of its estradiol, its structural relative. Select all that apply.

- Look directly for binding of radiolabeled BPA to receptors for estrogens and related hormones. - Check for competitive or synergistic effects of coadministration of BPA and estradiol. - Compare the effects on transcription of estradiol and bisphenol A in estrogen-responsive cells.

Identify and briefly discuss each mechanism ensuring against simultaneous fatty acid synthesis and oxidation in the same cell. Check all that apply.

- Malonyl-CoA, a key intermediate in fatty acid synthesis, inhibits carnitine acyltransferase I, thereby blocking the entry of fatty acyl units into the mitochondrion for oxidation. - Fatty acyl-CoAs, the substrates for fatty acid oxidation, inhibit fatty acid synthesis by interfering with the polymerization of acetyl-CoA carboxylase. - Hormonal effects on adipocytes: insulin promotes fatty acid synthesis by several mechanisms, while glucagon promotes fat breakdown and fatty acid oxidation.

cis-Vaccenate is an 18-carbon unsaturated fatty acid abundant in E. coli membrane lipids. Propose a metabolic route for synthesis of this fatty acid, in light of the fact that stearic acid, the C18 saturated analogous fatty acid, is virtually absent from E. coli lipids. Drag the appropriate labels to their respective targets. Note: not all labels will be used.

- NADH + H+ + O2 - palmitoyl-CoA delta9 desaturase - NAD+ + 2H2O - palmitoleoyl-CoA - malonyl-CoA + 2NADPH + 2H+ - C2 elongation - CoA-SH + 2NADP+ + H2O + CO2

As a representation of the respiratory chain, what is wrong with this picture? There are four deliberate errors.

- NADH is oxidized by FMN, not FAD. - O2 is reduced to H2O, not H2O2. - Cyt c1 accepts e- from cyt b, and is then oxidized by cyt c. - Reduced flavin (FADH2 or FMNH2) is oxidized by CoQ, not CoQH2.

List factors that make it advantageous for peptide hormones to be synthesized as inactive prohormones that are activated by proteolytic cleavage. Check all that apply.

- Signal sequence on the prohormone molecule could direct the protein to the correct subcellular site, then be cleaved off when no longer needed. - Suppression of the hormonal signal until it is needed. - Additional polypeptide sequence may promote correct folding of the polypeptide chain.

Describe a mechanism by which a steroid hormone might act to increase intracellular levels of cyclic AMP. Check all that apply.

- Steroid hormones could activate transcription of the gene for adenylate cyclase and, hence, increase the steady-state level of this enzyme. - The mechanism could include induced synthesis of a phosphodiesterase inhibitor or repressed synthesis of cAMP phosphodiesterase.

As described in the Foundation Figure, one can consider the conversion of substrate to product as a series of interactions with an enzyme, which serves as an organic catalyst: E+S k1⇌k−1 ES k2⇌k−2 EP k3⇌k−3 E+P Leanor Michaelis, Maude Menten, G.E. Briggs and J.B.S. Haldane simplified this equation by making assumptions. Which of the following were assumptions that helped to simplify this reaction to the form that was then used for the Michaelis-Menten equation? Check all that apply.

- The back-reaction between enzyme (E) and product (P) is negligible, making k-2 and k-3 essentially equal to zero. - The catalytic step, k2, is slow relative to the other rate constants, making k2 the rate-limiting step in catalysis. - The binding of substrate (S) to enzyme (E) reaches a rapid equilibrium, such that the concentration of the ES complex is relatively constant after a brief mixing period.

Methyl-labeled [14C]methionine at a specific activity of 2.0 millicuries per millimole was injected into rats. Six hours later the rats were killed. Phosphatidylcholine was isolated from the liver and found to have a specific activity of 1.5 millicuries per millimole. What further information would you need for your calculated values to reflect the true rates of these processes? Check all that apply.

- The specific radioactivity of the final intermediate in each pathway. - The rate of degradation of the product.

Given what you know about the function of the glyoxylate cycle and the regulation of the citric acid cycle, propose control mechanisms that might regulate the glyoxylate cycle. Check all that apply.

- activation of citrate lyase by acetyl-CoA or fatty acids - inhibition of isocitrate lyase by succinate - substrate-level control of citrate synthase

Show the fate of the carbon atoms in the metabolism of glutamate (shown below) by dragging the appropriate molecule names onto the flowchart. (insert molecule) Drag the appropriate labels to their respective targets.

- glutamate - α-ketoglutarate - succinyl-SCoA - succinate - fumarate - malate - oxaloacetate - glucose

Formaldehyde reacts nonenzymatically with tetrahydrofolate to generate 5,10-methylenetetrahydrofolate. [14C]Formaldehyde can be used to prepare serine labeled in the β-carbon. What else would be needed? Check all that apply.

- glycine - serine hydroxymethyltransferase - tetrahydrofolate

Mammalian cells can become resistant to the lethal action of methotrexate by the selective survival of cells containing increases in dihydrofolate reductase gene copy number so that intracellular levels of the enzyme become very high. What other biochemical or genetic changes in cells could cause them to become resistant to methotrexate? Select all that apply.

- induction of enzymes for MTX catabolism - mutation in gene-regulatory machinery that leads to DHFR overproduction - mutation in the DHFR structural gene rendering the enzyme resistant to MTX inhibition - deficiency of folate polyglutamate synthesis, that makes it difficult for the cell to retain MTX after uptake - transport defect that blocks uptake of methotrexate into the cell

The text describes a form of gout that results from HGPRT deficiency. Which enzyme abnormalities might similarly lead to hyperuricemia? Select all that apply.

- mutations that cause overexpression of xanthine dehydrogenase - mutations that weaken allosteric regulation of the conversion of PRPP to 5-phosphoribosylamine - mutations that lead to overexpression of PRPP synthetase

Calculate the standard free energy change for the reaction FADH2 + 1/2O2 -> FAD + H2O given that the standard reduction potential for the reduction of oxygen to water is +0.82 V and for the reduction of FAD to FADH2 is +0.03 V.

-152 kJ/mol

Complete a series of balanced equations and a summary equation for the reactions of the glucose-alanine cycle. Drag the terms on the left to the appropriate blanks on the right to complete the sentences.

1. . NH+4(muscle) + α-ketoglutarate + NADH + H+ -> glutamate + NAD+ + H2O 2. Glutamate + pyruvate -> alanine + α-ketoglutarate 3. Alanine + α-ketoglutarate -> glutamate + pyruvate 4. Glutamate + H2O + NAD+ -> NH+4(liver) + NADH + H+ + α-ketoglutarate 5. Sum: NH+4(muscle) -> NH+4(liver)

Glycolysis and gluconeogenesis are opposed pathways in that they begin or end with the same metabolites and share common intermediates and/or enzymes. Yet, for energetic reasons, the two processes cannot be the exact reverse of each other. How is this possible? 1) Because not every intermediate or enzyme participates in both pathways, conditions will dictate which pathway is stimulated while the other is inhibited. 2) Levels of ATP will dictate which pathway is operational. 3) Both pathways make use of substrate cycles.

All three statements are correct.

BLANK releases energy that is used to pump hydrogen ions from the stroma into the thylakoid compartment.

B

AMPK and mTOR can both be considered intracellular signal integrators. Explain this definition.

Both AMPK and mTOR receive a number of intracellular upstream inputs of environmental signals, combine them and process. Then they transmit a response.

Which of the following is an endocrine regulator that inhibits food intake? - Adiponectin - Ghrelin - Leptin - Insulin - Both C and D

Both C and D

How would anaerobic conditions (when no O2 is present) affect the rate of electron transport and ATP production during oxidative phosphorylation? (Note that you should not consider the effect of ATP synthesis in glycolysis or the citric acid cycle.)

Both electron transport and ATP synthesis would stop.

ATP synthesis in chloroplasts is very similar to that in mitochondria: Electron transport is coupled to the formation of a proton (H+) gradient across a membrane. The energy in this proton gradient is then used to power ATP synthesis. Two types of processes that contribute to the formation of the proton gradient are: -processes that release H+ from compounds that contain hydrogen, and -processes that transport H+ across the thylakoid membrane. Drag the labels to the appropriate locations on the diagram of he thylakoid membrane. Use only the blue labels for the blue targets, and only the pink labels for the pink targets. Note: One blue target and one pink target should be left empty.

Bottom left Red: site of H+ release Very left Blue: H+ pumped across membrane Very right Blue: H+ diffuses across membrane Top right Red: site of ATP synthesis

Explain why a deficiency of steroid 21-hydroxylase leads to excessive production of sex steroids (androgens and estrogens).

By shutting down the pathway leading to aldosterone, this deficiency increases the supply of progesterone available for conversion to sex steroids.

Energized electrons from BLANK enter an electron transport chain and are then used to reduce NADP+.

C

Which folate structure (from the list below) Has the most highly oxidized one-carbon substituent?

C

Which folate structure (from the list below) Is used in purine nucelotide synthesis?

C

Consider the fate of pyruvate labeled with 14C in each of the following positions: carbon 1 (carboxyl), carbon 2 (carbonyl), and carbon 3 (methyl). Predict the fate of each labeled carbon during one turn of the citric acid cycle.

C-1 released as CO2. C-2 and C-3: all retained in oxaloacetate.

Suppose it were possible to label glucose with 14C at any position or combination of positions. For yeast fermenting glucose to ethanol, which form or forms of labeled glucose would give the most radioactivity in CO2CO2 and the least in ethanol? Check all that apply.

C-3 C-4

Which carbon or carbons of glucose, if metabolized via glycolysis and the citric acid cycle, would be most rapidly lost as CO2? Check all that apply.

C-3 C-4

Write a balanced equation for the complete oxidation of ethanol. Express your answer as a chemical equation.

C2H5OH + 3O2 -> 2CO2 + 3H2O

Draw the amino acid product that results from a transamination reaction involving pyruvate and glutamate, shown in the image below. Draw the product as it would occur at the pH of most body fluids, including all charges. You do not need to draw the other product of the reaction. (insert reaction) Draw the molecule on the canvas by choosing buttons from the Tools (for bonds), Atoms, and Advanced Template toolbars. The single bond is active by default. Include all hydrogen atoms and charges.

CH(COO-)(CH3)(NH3+)

Draw the structure for the intermediate G. Draw the molecule on the canvas by choosing buttons from the Tools (for bonds and charges), Atoms, and Templates toolbars, including charges where needed.

CH2==CH--O--POOO^2-

Write a balanced equation for the complete oxidation of linoleic acid. Express your answer as a chemical equation.

CH3(CH2)12(CH)4COOH + 25O2 -> 18CO2 + 16H2O

Write a balanced equation for the complete oxidation of stearic acid. Express your answer as a chemical equation.

CH3(CH2)16COOH + 26O2 -> 18CO2 + 18H2O

Draw the structure for the intermediate I. Draw the molecule on the canvas by choosing buttons from the Tools (for bonds and charges), Atoms, and Templates toolbars, including charges where needed.

CH3--C=O--O--POOO^2-

Draw the structure for the intermediate H. Draw the molecule on the canvas by choosing buttons from the Tools (for bonds and charges), Atoms, and Templates toolbars, including charges where needed.

CH3--CH==O

Draw the product of the oxidative deamination of alanine. (insert reaction) Draw the molecule on the canvas by choosing buttons from the Tools (for bonds), Atoms, and Advanced Template toolbars. The single bond is active by default. Include all hydrogen atoms and charges.

CH3--CO--CO--O^-

Write a balanced equation for the complete oxidation of acetic acid. Express your answer as a chemical equation.

CH3COOH + 2O2 -> 2CO2 + 2H2O

Psilocybin is a hallucinogenic compound found in some mushrooms. Complete a straightforward pathway for its biosynthesis from one of the aromatic amino acids. Drag the appropriate labels to their respective targets.

CO2 PLP --> ahydrobiopt O,hydrobione --> 2AdoMet 2AdoHcy --> ATP ADP

Consider the following questions about glutamate dehydrogenase. Draw the structure of intermediate for this reaction. Draw the molecule on the canvas by choosing buttons from the Tools (for bonds and charges), Atoms, and Templates toolbars, including charges where needed.

COO^- --CH2--CH2--C=NH2+--COO^-

Some nonessential amino acids are synthesized in the body by a simple transamination. The transamination between oxaloacetate, an αα-keto acid, and glutamate, an amino acid with the side chain −CH2CH2COO−, proceeds according to the following reaction: (insert reaction) Predict the structure of the amino acid product, indicated by amino acid 2, for the reaction. Draw the molecule on the canvas by choosing buttons from the Tools (for bonds), Atoms, and Advanced Template toolbars. The single bond is active by default. Include all hydrogen atoms and formal charges.

COO^- --CHNH3+--CH2--COO^-

Suppose ribulose-5-phosphate, labeled with 14C in carbon 1, is used as the substrate in dark reactions. In which carbon of 3PG will the label appear?

Carbon 3

Leukemia is a neoplastic (cancerous) proliferation of white blood cells. Clinicians are currently testing deoxycoformycin, an adenosine deaminase inhibitor, as a possible antileukemic agent. Why might one expect this therapy to be effective? Drag the terms on the left to the appropriate blanks on the right to complete the sentences.

Deoxycoformycin might lead to ADENOSINE AND DEOXYADENOSINE accumulation, since their deamination would be inhibited, as in adenosine deaminase DEFICIENCY. This could cause dATP to ACCUMULATE, thereby inhibiting all four activities of ribonucleotide reductase. There is evidence that depletion of the other dNTPs leads to DNA SINGLE-STRAND breaks.

Folic acid is synthesized in bacteria as dihydrofolate, in a pathway starting from guanosine triphosphate. In this pathway, C−8 is lost as formate. From the structural similarities between guanine and pterin, predict which carbon and nitrogen atoms of GTP are the precursors to N−1, C−2, C−4, N−5, C−7, N−8, and C−9 of dihydrofolate. Drag the appropriate labels to their respective targets.

Dihydrofolate: GTP: N-1 N-3 C-2 C-2 C-4 C-6 N-5 N-7 C-7 C-1' N-8 N-9 C-9 C-3'

Where does the Calvin cycle occur?

E

Which folate structure (from the list below) Transfers its one-carbon substituent to a B12 coenzyme? What amino acid is synthesized as the end result of this reaction?

E

Which of these phosphorylates ADP to make ATP?

E

How does it compare with the energy in an einstein of 700 nm photons?

E = 171 kJ

The oxidation of glucose and palmitic acid yields 15.64 kJ/g and 38.90 kJ/g, respectively. Calculate the oxidation energy in term of kJ/mol for glucose. Express your answer to four significant figures and include the appropriate units.

E = 2815 kJ/mol

The oxidation of glucose and palmitic acid yields 15.64 kJ/g and 38.90 kJ/g, respectively. Calculate the oxidation energy in term of kJ per carbon atom oxidized for glucose. Express your answer using four significant figures.

E = 469.2 kJ/mol of C

The oxidation of glucose and palmitic acid yields 15.64 kJ/g and 38.90 kJ/g, respectively. Calculate the oxidation energy in term of kJ per carbon atom oxidized for palmitic acid. Express your answer using four significant figures.

E = 622.4 kJ/mol of C

The oxidation of glucose and palmitic acid yields 15.64 kJ/g and 38.90 kJ/g, respectively. Calculate the oxidation energy in term of kJ/mol for palmitic acid. Express your answer to four significant figures and include the appropriate units.

E = 9959 kJ/mol

Which of the following does NOT apply to the reaction catalyzed by the pyruvate dehydrogenase complex?

Each intermediate in the five step reaction is able to diffuse to the next active site in a sequential manner.

Which of the following is an extracellular messenger involved in signal transduction?

Epinephrine

Studies on cAMP actions in cultured cells usually involve adding to the cell culture not cAMP, but dibutyryl cAMP (see structure below). A. Why is this structural modification necessary? B. How could you test the premise that di-Bu-cAMP has the same biochemical effects as cAMP? Select all that apply.

A. Negative charge on the phosphate of cAMP prevents its passage through cell membranes. The butyrate chains make the molecule more lipophilic, although the negative charge on the phosphate is unaffected. B. - One could assess the effect of dibutyryl-cAMP on purified cAMP target proteins. - One could compare the actions of cAMP and dibutyryl-cAMP in cells whose plasma membrane had been treated with an agent to increase its permeability.

Suppose that you measured binding to the isolated EGF receptor of EGF at various concentrations. A. Would you expect the binding curve to be hyperbolic? B. Choose the correct explanation of your answer.

A. No, the binding curve will not be hyperbolic. B. The reaction of EGF with its receptor should be trimolecular at low EGFR concentrations, because EGF binding should promote dimerization of the monomeric receptor.

Aconitase catalyzes the reaction: citrate⇌isocitratecitrate⇌isocitrate The standard free energy change, ΔG∘', for this reaction is +6.3 kJ/mol. However, the observed free energy change (ΔG) for this reaction in mammalian mitochondria at 25 ∘C is ~ 0 kJ/mol. A. Calculate the ratio of [isocitrate]/[citrate] in mitochondria. Express your answer using two significant figures. B. Is this reaction likely to be a control point for the citric acid cycle? Why or why not?

A. [isocitrate]/[citrate] = 7.9*10^-2 B. No, because aconitase catalyzes a freely reversible reaction.

A. Which of these enters the citric acid cycle? B. In the citric acid cycle, ATP molecules are produced by _____. C. Which of these is NOT a product of the citric acid cycle?

A. acetyl CoA B. substrate-level phosphorylation C. acetyl CoA

Certain microorganisms with a modified citric acid cycle decarboxylate αα-ketoglutarate to produce succinate semialdehyde. A. Succinate semialdehyde is then converted to succinate, which is further metabolized by standard citric acid cycle enzymes. What kind of reaction is required to convert succinate semialdehyde to succinate? B. Show any coenzymes that might be involved. Check all that apply. C. Based on your answer in part A, how does this pathway compare to the standard citric acid cycle in energy yield?

A. an oxidation reaction B. E-FAD, NAD+ C. The modified pathway generates one less ATPATP than the standard pathway. There is no difference in the number of reduced cofactors generated.

A. The magnification of the signal from a water-soluble hormone is achieved through an increase in ________. B. Water-soluble hormones affect target cells by binding to ________. C. How do endocrine hormones reach their target cells? Select the best answer. D. What is the role of activated protein kinases? Select the best answer. E. Cyclic AMP is degraded by ________.

A. cAMP in the cytoplasm B. plasma membrane receptors C. Hormones are transported through the blood stream to target cells. D. Phosphorylate proteins E. phosphodiesterase

A. Choose the correct balanced equation for the hydrolysis of cGMP, catalyzed by cGMP phosphodiesterase. B. Would you expect an inhibitor of this enzyme to potentiate or antagonize the action of Viagra? Choose the correct explanation.

A. cGMP + H2O -> 5'-GMP B. Viagra acts by inhibiting this reaction, any other compound acting similarly should potentiate the effect of Viagra.

A. Hormones that use the tyrosine kinase receptor mechanism as shown in the animation are important for B. Autophosphorylation is an important part of the mechanism of the receptor tyrosine kinases. Why is this? C. Sos activates Ras. Sos is an example of a D. Receptor tyrosine kinases are activated by hormones that cause cell division (mitogens). Which of the following describes a way that a protein in the pathway could be mutated to result in its constitutive activation causing unregulated cell division? E. The dominant negative mutation of the receptor in the animation is effective in blocking the hormone signal because

A. cellular growth and survival. B. The phosphorylation provides sites for recruitment of other signal transduction molecules. C. guanine-nucleotide exchange factor (GEF). D. a mutation in Ras that makes it unable to hydrolyze GTP E. it is able to dimerize with a functional receptor without the activation of autophosphorylation.

Choose a balanced equation for each of the following reactions or reaction sequences. A. The reaction catalyzed by PFK−2. B. The conversion of 2 moles of oxaloacetate to glucose. C. The conversion of glucose to UDP−Glc. D. The conversion of 2 moles of glycerol to glucose. E. The conversion of 2 moles of malate to glucose-6-phosphate.

A. fructose-6-phosphate + ATP -> fructose-2,6-biphosphate + ADP B. 2 oxaloacetate + 2ATP + 2GTP + 2NADH + 2H+ + 4H2O -> glucose + 2CO2 + 2NAD+ + 2ADP + 2GDP + 4Pi C. glucose + ATP + UTP -> UDP-Glc + PPi + ADP D. 2 glycerol + 2ATP + 2NAD+ + 2H2O -> glucose + 2ADP + 2NADH + 2H+ 2Pi E. 2 malate + 2ATP + 2GTP + 3H2O -> glucose-6-phosphate + 2CO2 + 2ADP + 2GDP + 3Pi

Sulfonamide drugs like sulfanilamide inhibit tetrahydrofolate biosynthesis. A. What intermediate or intermediates in purine synthesis would you expect to accumulate in sulfanilamide-treated bacteria? B. Suppose that a cultured mammalian cell line was treated with sulfanilamide. What intermediates might accumulate in these cells?

A. glycinamide ribonucleotide and 5-aminoimidzole-4-carboxamide B. Mammalian cells are not influenced by sulfanilamide, therefore no products will accumulate.

Methanol is highly toxic, not because of its own biological activity but because it is converted metabolically to formaldehyde, through action of alcohol dehydrogenase. Part of the medical treatment for methanol poisoning involves administration of large doses of ethanol. Explain why this treatment is effective. Match the words in the left column to the appropriate blanks in the sentences on the right. Make certain each sentence is complete before submitting your answer.

Ethanol will generate NADH, through the action of alcohol dehydrogenase, and this will REDUCE formaldehyde back to methanol, also through alcohol dehydrogenase. The ACETALDEHYDE formed can be metabolized further to acetate. Ethanol may also compete with methanol for binding to alcohol dehydrogenase.

Sort the phrases into the appropriate bins depending on whether they describe exocytosis, endocytosis, or both.

Exocytosis: - increases the surface area of the plasma membrane - secretes large molecules out of the cell - requires fusion of vesicles with the plasma membrane Endocytosis: - decreases the surface area of the plasma membrane - forms vesicles from inward folding of the plasma membrane Both: - transported substances never physically cross the plasma membrane - requires cellular energy

Briefly explain your answer.

E∘′ for NAD+/NADH is the same as for NADP+/NADPH

Which folate structure (from the list below) Is not known to exist in nature?

F

Examine the molecular model of the partial structure of the yeast ATP synthase. Which part of the molecule spans the inner mitochondrial membrane?

F0 rotor

5-phosphoribosyl-α-D-1-pyrophosphate is a key intermediate in de novo synthesis of nucleotides but not in salvage pathways.

False

A proton gradient is formed across the thylakoid membrane with the stroma becoming more acidic than the lumen.

False

All of the cytochromes in the mitochondrial respiratory chain contain the same heme moiety as found in hemoglobin.

False

All of the dNTPs are produced in approximately the same amounts because the first step in each pathway is catalyzed by the same ribonucleotide reductase enzyme.

False

All of the reactions of both glycolysis and gluconeogenesis occur in the cytosol.

False

All six atoms in the pyrimidine ring are derived from two amino acids.

False

Elongation of fatty acids from palmitate occurs only in the mitochondrion.

False

Enzyme-catalyzed sequential reactions that make up a metabolic pathway are normally separated in the cell for equal access to reactants.

False

Epinephrine has a longer term effect on metabolism than does glucagon.

False

Excess cholesterol is metabolically degraded in the liver.

False

Hormone action is ultimately controlled by the cells that secrete the hormone.

False

Insulin and glucagon are both synthesized by the same cell types in the pancreas.

False

Ketone bodies can be used as an alternative fuel source by the liver.

False

Liver pyruvate kinase is activated by dephosphorylation in response to glucagon.

False

Most synaptic transmission events are mediated electrically through gap junctions between cells.

False

Nitrate can be reduced to ammonia by virtually all organisms.

False

Nitrogen fixation requires the hydrolysis of 8 ATPs to produce 2 NH3 molecules from 1 N2.

False

Phosphorylases and phosphatases catalyze the same reaction, the removal of a phosphate group.

False

Photons with any energy value are able to cause transitions in the electronic states of organic molecules to drive reactions and thus capture light energy in chemical form.

False

Receptor tyrosine kinases are membrane-spanning proteins with an intrinsic tyrosine kinase domain on the extracellular side of the membrane.

False

Substrate-level phosphorylation is a term given to the loss of free energy when ATP is hydrolyzed.

False

The ATP yield from NADH transported across the mitochondrial inner membrane by the glycerol phosphate shuttle is the same as if the malate/aspartate shuttle were used.

False

The H2S produced by metabolism of cysteine is a waste product with no physiological or biochemical function.

False

The Krebs-Henseleit urea cycle takes place in liver cells entirely within the mitochondrion.

False

The Pasteur effect describes the observation that yeast-metabolizing glucose anaerobically will dramatically increase their rate of glucose utilization when provided with air.

False

The citric acid cycle is classed as a reductive pathway as it produces reduced electron carriers.

False

The flux rate through the gluconeogenic pathway is directly proportional to the amount of carbohydrate in the diet.

False

Suppliers of radioisotopically labeled compounds usually provide each product as a mixture of labeled and unlabeled material. Unlabeled material is added deliberately as a carrier, partly because the specific activity of the carrier-free product is too high to be useful and partly because the product is more stable at lower specific activities. Using the radioactive decay law, calculate the following. The specific activity of carrier-free [32P]-orthophosphate, in mCi/mmol. (t1/2 = 14.2 days)

Activity = 9.27*10^6 mCi/mmol

The two carbon atoms that are lost as CO2 in the third and fourth steps of the citric acid cycle are the same as the two carbon atoms of acetyl CoA because of the stereochemistry of the isocitrate dehydrogenase reaction.

False

True or false? The chemiosmotic hypothesis states that the synthesis of ATP generates a proton gradient that leads to electron flow through an electron transport chain.

False

True or false? The region of ATP synthase that catalyzes the production of ATP from ADP and inorganic phosphate spans the chloroplast membrane.

False

Tyrosine is the precursor of serotonin which has a number of roles in the nervous system.

False

β-oxidation of unsaturated fatty acids yields the same amount of energy as saturated fatty acids with the same number of carbons.

False

Which of the following statements regarding the β-oxidation of FA is NOT correct? - Enoyl-CoA hydratase catalyzes the hydration of the α,β-unsaturated CoA thioester to a β-hydroxy CoA thioester. - Acyl-CoA dehydrogenase catalyzes the synthesis of a cis α,β-unsaturated CoA thioester. - Thiolase catalyzes the thiolytic cleavage of a 3-ketoacyl CoA ester to release acetyl CoA and a new FA with two fewer carbons. - 3-L-hydroxyacyl-CoA dehydrogenase catalyzes the oxidation of the 3-hydroxyacetyl CoA ester to the corresponding 3-ketone.

Acyl-CoA dehydrogenase catalyzes the synthesis of a cis α,β-unsaturated CoA thioester.

In the early days of "mitochondriology", P/O ratios were determined from measurements of volume of O2 taken up by respiring mitochondria and chemical assays for disappearance of inorganic phosphate. Now, however, it is possible to measure P/O ratios simply with a recording oxygen electrode. How might this be done?

Add ADP in limiting amount and measure O2 uptake. The ratio of umol ADP consumed to uatom oxygen taken up is identical to the P/O ratio.

The substance dichlorophenyldimethylurea (DCMU) is an herbicide that inhibits photosynthesis by blocking electron transfer between plastoquinones in photosystem II. Normally, DCMU blocks O2 evolution, but addition of ferricyanide to chloroplasts allows O2 evolution in the presence of DCMU. Explain.

Addition of ferricyanide as an electron donor allows a Hill reaction.

Phosphatidylserine (PS) is considered to be an intermediate in the biosynthesis of phosphatidylethanolamine (PE) in E. coli, yet PS is not found in appreciable amounts among E. coli membrane phospholipids. What kinds of experiments could test your proposed explanation?

Addition of radiolabeled PS to an enzyme system would not label PE, because phosphatidylserine decarboxylase would act only on PS generated by phosphatidylserine synthase.

Considering the evidence that led Krebs to propose a cyclic pathway for oxidation of pyruvate, discuss the type of experimental evidence that might have led to realization of the cyclic nature of the glyoxylate pathway.

Addition to isolated glyoxysomes of citrate, isocitrate, glyoxylate, malate, or oxaloacetate would stimulate succinate formation out of proportion to the amount added.

Adipose tissue cannot resynthesize triacylglycerols from glycerol released during lipolysis (fat breakdown). Why not?

Adipose tissue lacks glycerol kinase.

NADH and FADH2 are both electron carriers that donate their electrons to the electron transport chain. The electrons ultimately reduce O2 to water in the final step of electron transport. However, the amount of ATP made by electrons from an NADH molecule is greater than the amount made by electrons from an FADH2 molecule. Which statement best explains why more ATP is made per molecule of NADH than per molecule of FADH2?

Fewer protons are pumped across the inner mitochondrial membrane when FADH2 is the electron donor than when NADH is the electron donor.

Design a radiotracer experiment that would allow you to determine which proportion of glucose catabolism in a given tissue preparation occurs through the pentose phosphate pathway and which proportion occurs through glycolysis and the citric acid cycle. Assume that you can synthesize glucose labeled with 14C in any desired position or combination of positions. Assume also that you can trap CO2 after administration of labeled glucose and determine its radioactivity.

Administer separately 1-[14C]glucose and 6-[14C]glucose, and measure initial rates of 14CO2 formation. The ratio gives relative flux rates through the two pathways.

Proline betaine is a putative osmoprotectant in plants and bacteria, helping to prevent dehydration of cells. Propose a plausible pathway for biosynthesis of this compound.

AdoMet -> AdoMet

Sort the characteristics as belonging to the first half of the pathway that requires energy or to the second half of the pathway that produces energy during glycolysis. Drag the appropriate items to their respective bins.

First half of the pathway that requires energy: - ATP is utilized. - Six-carbon compounds start the pathway. - Overall reaction: glucose -> dihydroxyacetone phosphate + glyceraldehyde-3-phosphate Second half of the pathway that produces energy: - ATP is synthesized. - Three-carbon compounds start the pathway. - NADH + H+ is produced. - Overall reaction: dihydroxyacetone phosphate + glyceraldehyde-3-phosphate -> pyruvate

A clinical test sometimes used to diagnose folate deficiency or B12 deficiency is a histidine tolerance test, where one injects a large dose of histidine into the bloodstream and then carries out a series of biochemical determinations. What histidine metabolite would you expect to accumulate in a folate- or B12- deficient patient, and why?

Formiminoglutamate, because the next reaction in its catabolism requires tetrahydrofolate.

Suppliers of radioisotopically labeled compounds usually provide each product as a mixture of labeled and unlabeled material. Unlabeled material is added deliberately as a carrier, partly because the specific activity of the carrier-free product is too high to be useful and partly because the product is more stable at lower specific activities. Using the radioactive decay law, calculate the following. The fraction of H atoms that are radioactive in a preparation of uniform-label [3H]-leucine, provided at 10 mCi/mmol. (t1/2 = 12.1 years)

Fraction = 2.61*10^-3%

As early as the 1930s, it was known that frog muscles could still contract when glycolysis was inhibited. Where did the ATP come from to drive these contractions?

From the creatine phosphate stores in the muscle.

Which of the following intermediates of the Calvin cycle is NOT used directly to regenerate ribulose-1,5-bisphosphate? -Dihydroxyacetone phosphate -Glyceraldehyde 3-phosphate -Fructose 6-phosphate -Fructose 1,6-bisphosphate

Fructose 1,6-bisphosphate

Which of the following compounds is NOT a precursor for nucleotide biosynthesis?

Fumarate

Which statement about G proteins is NOT true?

G proteins possess very active GTPase activity.

Suppose a brief pulse of 14CO2 is taken up by a green plant. Trace the 14C label through the steps leading to fructose-1,6-biphosphate synthesis, showing which carbon atoms in each compound should carry the label during the first cycle. Drag the terms on the left to the appropriate blanks on the right to complete the sentences.

After being attached to C-2 of ribulose-1,5-biphosphate, the 14C will become the carboxylate (C-1) of one of the two molecules of 3-phosphoglycerate. This becomes the carbonyl carbon (C-1) in 3-phosphoglyceraldehyde, or C-1 in dihydroxyacetone phosphate. Upon condensation to form fructose-1,6-bisphosphate, the 14C will chow up in C-3 and/or C-4.

Which of the following types of reaction does NOT occur in glycolysis?

Aldol condensation

Radioactive hypoxanthine (HX) can be used to label purine residues. Match the cofactors to the reactions they are involved in and their effect on this reactions Drag the terms on the left to the appropriate blanks on the right to complete the sentences. Terms can be used once, more than once, or not at all.

GDP+[2H] -> dGDP+H2O Cofactor: dTTP Effect: activation IMP+Asp+GTP -> Adenylosuccinate+GDP+Pi Cofactor: AMP Effect: inhibition IMP+NAD^+ +H2O -> XMP+NADH+H^+ Cofactor: GMP Effect: inhibition ADP+[2H] -> dADP+H2O Cofactor: dGTP Effect: activation

Identify each reaction catalyzed by a nucleotidase, a phosphorylase and a phosphoribosyltransferase. Drag the appropriate labels to their respective targets. Labels can be used once or more than once. Not all targets will be used.

GMP <--nucleotidase(top)--> Guanosine Guanine <--phosphorylase(x2)--> Guanosine Guanine <--phosphoribosyl-transferase(x2)--> GMP

Sort each of the following amino acids based on whether it is glucogenic, ketogenic, or both glucogenic and ketogenic. Drag the appropriate amino acids to their respective bins.

Glucogenic: - proline - asparagine - glutamine - serine Ketogenic: - leucine Both glucogenic and ketogenic: - phenylalanine - isoleucine - tyrosine

Under anaerobic conditions (a lack of oxygen), glycolysis continues in most cells despite the fact that oxidative phosphorylation stops, and its production of NAD+ (which is needed as an input to glycolysis) also stops. The diagram illustrates the process of fermentation, which is used by many cells in the absence of oxygen. In fermentation, the NADH produced by glycolysis is used to reduce the pyruvate produced by glycolysis to either lactate or ethanol. Fermentation results in a net production of 2 ATP per glucose molecule. During strenuous exercise, anaerobic conditions can result if the cardiovascular system cannot supply oxygen fast enough to meet the demands of muscle cells. Assume that a muscle cell's demand for ATP under anaerobic conditions remains the same as it was under aerobic conditions. What would happen to the cell's rate of glucose utilization?

Glucose utilization would increase a lot.

Which of the following statements about β-oxidation of fatty acids is CORRECT? - Both dehydrogenation reactions use FAD as the electron acceptor. - Both dehydrogenation reactions produce a C=C double bond. - Fatty acids of any length are oxidized by exactly the same enzymes. - Thiolytic cleavage releases acetyl-CoA and a free fatty acid in the final step of the cycle. - All fatty acids must be activated to acyl-CoAs before entering the pathway.

All fatty acids must be activated to acyl-CoAs before entering the pathway.

The combustion of glucose ultimately involves the transfer of electrons to oxygen (O2). In biological systems, this takes place without a large increase in temperature. Which of the following statements about this process in biological systems is true? 1) Coupled oxidation-reduction reactions occur with electrons being passed to intermediate electron carriers. 2) The process involves the transfer of 12 pairs of electrons to oxygen. 3) The transfer of electrons from intermediate electron carriers to molecular oxygen is catalyzed by the electron transport chain. 4) Though the process is a controlled oxidation, some energy is lost.

All four statements are true.

Insulin stimulates fatty acid synthesis by which of the following mechanisms? - stimulation of dephosphorylation of pyruvate dehydrogenase - activation of citrate lyase - dephosphorylation of acetyl-CoA carboxylase - stimulation of a glucose transport across the plasma membrane - All of the above

All of the above

Which of the following can be used as a metabolic control mechanism? - Regulation of enzyme degradation - Covalent modification of an enzyme - Action of hormones - Enzyme compartmentation - All of the above

All of the above

Which of the following fuel reserves can be used by skeletal muscle? - Glycogen - Creatine phosphate - Fatty acids - Protein - All of the above

All of the above

Which of the following are part of the chemiosmotic coupling model? 1) Protons are pumped out of the mitochondrial matrix into the intermembrane space. 2) The active transport of protons is driven by the free energy derived from electron transport. 3) Increasing the concentration of protons on the other side of the inner membrane creates a charge and a pH imbalance that releases energy upon proton flow back into the matrix. 4) The energy released by the return of protons to the matrix is what drives ATP synthesis.

All of the listed choices are correct.

Which of the following coenzymes participate in the reactions of the pyruvate dehydrogenase complex? 1) thiamine pyrophosphate 2) lipoic acid 3) FAD 4) NAD+ 5) CoA

All of the listed coenzymes participate.

One pathway that appears to relate elevated cholesterol levels to atherosclerosis involves the oxidation of LDL. Which of the following are consequences of this process? - Though the mechanism by which this occurs is not completely understood, this oxidation includes the peroxidation of fatty acids, the hydroxylation of cholesterol, and the oxidation of amino acids. - The presence of scavenger receptors on macrophages allows unlimited cholesterol uptake, leading to its conversion to foam cells. - Oxidized LDL products induce an immune response that includes recruitment of monocytes and T lymphocytes.

All of the listed responses are consequences of this process.

Which of the following enzymes are required for the oxidation of unsaturated FA? - 2,4-Dienoyl-CoA reductase - acyl-CoA dehydrogenase - enoyl-CoA isomerase

All of the listed responses are correct.

Which of the following are characteristics of G protein-coupled receptors? 1) They typically have seven transmembrane domains that are embedded in the cell membrane. 2) They carry hydrophilic portions of protein that extend into the extracellular matrix as well as the cytosol. 3) The receptor region is located on the extracellular portion of the protein. 4) Most work in concert with G proteins.

All of the listed statements are characteristics of G protein-coupled receptors.

Which of the following are key discoveries of the experiments conducted by Brown and Goldstein? 1) Cholesterol is taken into cells through the action of a specific receptor. 2) The receptor for the LDL particle recognizes B-100 apolipoprotein. 3) The entire receptor complex, unlike hormones involved in signaling pathways, enters the cell via endocytosis. 4) Familial hypercholesterolemia patients have defective or deficient LDL receptors.

All of the listed statements are true.

Which of the following processes describe the way excited molecules can return to their ground state? 1) They can return via radiationless dissipation of energy. 2) They can lose this energy as a different wavelength of light in a process known as fluorescence. 3) They can transfer this energy to adjacent molecules in a process known as resonance transfer. 4) They can transfer an excited electron to a nearby molecule with a slightly lower excited state in a process known as electron transfer.

All of the listed statements are true.

Which of the following statements about the two-system light reactions are true?1) The first reaction takes electrons from water, releasing oxygen. 2) The electrons released above are then accepted by NADP+. 3) During the transfer of electrons throughout this series of reactions, protons are pumped into the thylakoid lumen, generating a proton gradient. 4) The proton gradient represents a large difference in free energy, sufficient to drive the synthesis of ATP.

All of the listed statements are true.

Would the corresponding energy yield in a fish be higher or lower? Why?

All of these pathways would generate a little more energy in a fish because it is not necessary to consume ATP in converting ammonia to urea for excretion.

Glycolysis breaks down glucose in 10 enzymatic steps. In the steps where the glycolysis intermediates are oxidized, the electrons are transferred to electron-carrier molecules that bring the reducing equivalents to the electron-transport chain. The electron-transport chain uses the reducing equivalents to generate a proton gradient across the inner mitochondrial membrane. This proton gradient is used to drive the synthesis of ATP by the ATPase enzyme. Sort the following molecules according to whether they are produced in the glycolytic pathway, either as products of the reactions or as side products. Drag the items into the appropriate bin.

Product of glycolysis: pyruvate, NADH Other: glucose, creatine kinase, arachidonic acid, NADH2

Choose a pathway whereby some of the carbon from a fatty acid with an odd-numbered carbon chain could undergo a net conversion to carbohydrate.

Propionyl-CoA -> methylmalonyl-CoA -> succinyl-CoA -> oxaloacetate -> PEP -> glucose

Which of the following statements about the adrenergic receptor is NOT true?

Propranolol is an agonist for this receptor.

In early studies of adrenergic signaling, it was thought that the epinephrine receptor and adenylate cyclase were one and the same protein. What kind of evidence would prove otherwise?

Protein fractionation, with a demonstration that epinephrine-binding activity and adenylate cyclase activity could be physically separated from one another can be done.

Nitrogen is a limited resource, despite the presence of strong demand. Which of the following is NOT a metabolic consequence of the inability to store nitrogen?

Proteolysis is a non-selective, exergonic process.

According to the chemiosmotic hypothesis, what provides the energy that directly drives ATP synthesis?

Proton gradient

Which of the following particles can pass through the ATP synthase channel?

Protons

Sort the following protein complexes of the electron transport chain according to whether they are involved in pumping protons across the inner mitochondrial membrane or not. Drag the appropriate items to their respective bins.

Pumps protons: Complex I, Complex III, Complex IV Does not pump protons: Complex II, Cytochrome C, Coenzyme Q

[14C]Serine, prepared as described above, is useful for many things, but you would probably not want to use it for studies on protein synthesis because it would label nucleic acids, carbohydrates, and lipids, as well as proteins. Indicate how each of these classes of compounds could become labeled by this precursor. Drag the appropriate labels to their respective targets.

Pyruvate Oxaloacetate Phosphoenol-pyruvate Glucose Serine Phosphatidyl-serine Phosphatidylethanol-amine Phosphatidylcholine Thymine nucleotides Nuclelic acids

Calculate the respiratory quotient for ethanol. Express your answer using two significant figures.

RQ = 0.67

Calculate the respiratory quotient for stearic acid. Express your answer using two significant figures.

RQ = 0.69

Calculate the respiratory quotient for oleic acid. Express your answer using two significant figures.

RQ = 0.71

Calculate the respiratory quotient for linoleic acid. Express your answer using two significant figures.

RQ = 0.72

Calculate the respiratory quotient for acetic acid. Express your answer using two significant figures.

RQ = 1.0

What is the rate (pmol/10^6 cells/s) of metabolic flux between B and C? Express your answer using two significant figures.

Rate = 20 pmol/10^6 cells/s

Ammonia is a toxic byproduct of protein metabolism. To get rid of it, the body converts it to water-soluble urea through a process called the urea cycle. Identify the reactants and the products of the preparation step to the urea cycle. Drag the appropriate items to their respective bins.

Reactants: NH4+, 2ATP, CO2, H2 Products: H2N(CO)OPO3^2- (carbomoyl phosphate), 2ADP, Pi

Which of the following reactions ensures that the Calvin cycle can make a continuous supply of glucose?

Regeneration of RuBP

When the protein gramicidin is integrated into a membrane, an H+ channel forms and the membrane becomes very permeable to protons (H+ ions). If gramicidin is added to an actively respiring muscle cell, how would it affect the rates of electron transport, proton pumping, and ATP synthesis in oxidative phosphorylation? (Assume that gramicidin does not affect the production of NADH and FADH2 during the early stages of cellular respiration.) Sort the labels into the correct bin according to the effect that gramicidin would have on each process.

Remains the same: - rate of oxygen uptake - proton pumping rate - electron transport rate Decreases (or goes to zero): -size of the proton gradient - rate of ATP synthesis

Which of the following is NOT true considering the regulation of photosynthesis?

Rubisco is activated by 2-carboxy-D-arabinitol-1-phosphate, which is synthesized in the dark.

Pyridoxal phosphate is a versatile coenzyme as it is capable of forming a stable ________ base between an amino acid substrate and the coenzyme.

Schiff

In the late 1980s, Robert Weinberg isolated DNA from bladder cancer cells and used this to transfect normal mouse fibroblasts. Which of the following statements is NOT true of this experiment?

Sequence analysis of the transforming gene H-ras was found to contain multiple mutations.

Which of the following statements about purine biosynthesis in vertebrates is FALSE?

Six proteins form a multienzyme complex with covalent linkages.

The citric acid cycle can be broken into three stages. In the first stage, metabolic fuels (fats, amino acids, sugars) are converted into a common currency that is used by the second stage. The second stage is a substrate cycle in the mitochondrial matrix: the citric acid cycle proper. The third stage occurs in the inner mitochondrial membrane and is the principle source of cellular energy. Match the following features to the stage in which they can be found. Drag the appropriate items to their respective bins.

Stage 1: -conversion of pyruvate to acetyl-CoA -production of one molecule of CO2 Stage 2: -joining of acetyl-CoA to oxaloacetate -production of two molecules of CO2 Stage 3: -primary source of ATP production -reduction of oxygen to water -oxidation of NADH and FADH2 back to NAD+ and FAD

Transamination reactions have equilibrium constants close to unity. What does this mean? 1) In the absence of any other forces, the ratio of products to starting materials is nearly one. 2) Intracellular conditions can dictate the direction the equilibrium favors. 3) The forward reaction is not favored in cells. 4) Aminotransferases can alter the position of the equilibrium.

Statements 1 and 2 are accurate.

Antimetabolites are clinically useful for which of the following reasons? 1) They interfere with the utilization of the natural substrate (the metabolite) because they are mimics of it. 2) They can interfere with the biosynthesis of thymidine and then DNA synthesis. 3) They can block proliferation of cells, especially cancer cells. 4) They can allosterically alter the function of dihydrofolate reductase.

Statements 1, 2, and 3 are correct.

Which statements about complex IV are true? 1)This complex catalyzes the reduction of oxygen to water. 2)Despite consuming protons, this complex pumps H+ across the membrane at a rate of 4 per molecule of O2 reduced. 3)The protons flow across a gradient to the inner membrane space. 4)Conformational changes as electrons flow through the complex alter the ability of some residues to bind protons.

Statements 1, 2, and 3 are correct.

Photosynthesis takes place in all higher plants and algae in chloroplasts. Which of the following statements accurately describe these organelles? 1) In plants, they are located in mesophyll cells. 2) They are semiautonomous in that they carry their own DNA and ribosomes. 3) Unlike mitochondria, they have three membranes. 4) The carbohydrates produced by this reaction are stored as cellulose.

Statements 1, 2, and 3 are true.

Which of the following statements about AMPK are true? 1) It inhibits pathways that make use of ATP. 2) It is activated when the cell's energy charge is low. 3) It initiates signaling processes that conserve cellular energy. 4) It is a tyrosine kinase found in all eukaryotes.

Statements 1, 2, and 3 are true.

Which of the following statements about diabetes are true? 1) Type 1 is an autoimmune disease in which the beta cells of the pancreas are destroyed. 2) One hypothesis suggests that lipid overload interferes with insulin signaling and blocks the translocation of the GLUT4 transporter to the membrane. 3) One hypothesis suggests adipose cells secrete inflammatory cytokines that then interfere with insulin signaling. 4) Type 2 diabetes, the result of an autoimmune disease in which the pancreas alpha cells are destroyed, can be treated by insulin injections.

Statements 1, 2, and 3 are true.

The effects of folate deficiencies are manifested in which of the following ways? 1) elevated levels of homocysteine that are correlated with an increased incidence of heart attack 2) the presence of reduced amounts of erythrocytes that are immature and enlarged, a condition known as megaloblastic anemia 3) a decrease in the synthesis of vitamin B12 4) neural tube defects in early stages of pregnancy

Statements 1, 2, and 4 are correct.

Which regulatory effects are results of internalized cholesterol? 1) It activates acyl-CoA:cholesterol acyltransferase. 2) It increases endogenous cholesterol synthesis. 3) It downregulates the production of LDL receptors. 4) It inhibits HMG-CoA reductase.

Statements 1, 3, and 4 are direct results of the internalization of cholesterol.

Chlorophyll molecules are the most abundant pigments in plants that are responsible for collecting light for photosynthesis. Which of the following statements is not true? 1) The two main forms of chlorophyll are chlorophyll a and b. 2) These molecules resemble the protoporphyrins in hemoglobin and myoglobin, except these contain Fe+2 rather than Mg+2. 3) They require the presence of accessory pigments such as beta-carotene and lutein in order to function. 4) They absorb light in the blue and red region of the visible spectrum, which is why they appear green.

Statements 1, 3, and 4 are true.

Which of the following citric acid intermediates is(are) not used in the preparation of the products shown? 1) citrate ⟶ fatty acids, steroids 2) alpha-ketoglutarate ⟶ steroids 3) oxaloacetate ⟶ purines, pyrimidines 4) alpha-ketoglutarate ⟶ heme

Statements 2 and 4 are incorrect.

Insulin in a signal that indicates a fed state, which promotes which of the following function? 1) an increase in gluconeogenesis 2) the biosynthesis of macromolecules 3) an increase in fuel storage 4) the uptake of fuel molecules in some cells

Statements 2, 3, and 4 are part of the functions promoted by insulin.

Which of following statements about photosynthesis are true? 1) This process describes the catabolism of carbohydrates to CO2. 2) For plants, water is the ultimate reducing agent in this process. 3) The process releases oxygen as a byproduct. 4) The energy required for this transformation comes in the form of light.

Statements 2, 3, and 4 are true

Which of the following statements is FALSE? - The cholesterol ester content of HDL is generally less than that of chylomicrons. - Some fatty acids become complexed with serum albumin for transport in the blood to peripheral cells. - VLDL transports triacylglycerol synthesized in the liver to peripheral tissues. - LDL is the main form in which cholesterol is transported to tissues. - Chylomicrons are lipoproteins that transport dietary fat from intestine to peripheral tissues.

The cholesterol ester content of HDL is generally less than that of chylomicrons.

Which of the following statements about complex II is NOT true?

The complex draws electrons from succinate derived from fatty acid oxidation.

Consider the following questions about glutamate dehydrogenase. The thermodynamic equilibrium for the reaction greatly favors α-ketoglutarate reduction, yet in mitochondria the enzyme acts primarily to oxidize glutamate to α-ketoglutarate. Explain.

The direction of a reaction depends both on the equilibrium constant and the concentrations of reactants and products. The intramitochondrial [NAD+]/[NADH] ratio is high, and this drives the reaction toward α-ketoglutarate.

Compare these two values.

The energy yield from tripalmitin is larger than the energy yield from glucose.

Which of the following statements about glutamate dehydrogenase is NOT true?

The enzyme is under allosteric control in that ATP stimulates its activity, whereas ADP inhibits its function.

Which step in lipid metabolism would you expect to be affected by 3,4-dihydroxybutyl-1-phosphonic acid (shown here)? Explain your answer.

The esterification of glycerol-3-phosphate with fatty acyl-CoA, catalyzed by glycerophosphate acyltransferase. This is a nonhydrolyzable analog of glycerol-3-phosphate, so it is acylated without difficulty to give the phosphonate analog of diacylglycerol.

The net reaction of the Calvin cycle is the conversion of CO2 into the three-carbon sugar G3P. Along the way, reactions rearrange carbon atoms among intermediate compounds and use the ATP and NADPH produced by the light reactions. In this exercise, you will track carbon atoms through the Calvin cycle as required for the net production of one molecule of G3P. For each intermediate compound in the Calvin cycle, identify the number of molecules of that intermediate and the total number of carbon atoms contained in those molecules. As an example, the output G3P is labeled for you: 1 molecule with a total of 3 carbon atoms. Labels may be used once, more than once, or not at all.

a. 3 molecules 3 carbons b. 6 molecules 18 carbons c. 6 molecules 18 carbons d. 5 molecules 15 carbons e. 3 molecules 15 carbons f. 3 molecules 15 carbons

Discuss the metabolic rationale for phosphorylation of acetyl-CoA carboxylase by AMP-activated protein kinase (AMPK) and cyclic AMP-dependent protein kinase (PKA).

This could be a way for a cell to inhibit fatty acid synthesis under conditions where substrates are needed for oxidation.

CTP synthetase is allosterically activated by GTP. What function might this play in the cell?

This is a way to keep purine and pyrimidine nucleotide pools in balance.

How would you expect this toxin to affect blood pressure? Drag the terms on the left to the appropriate blanks on the right to complete the sentences. Not all terms will be used.

This would INACTIVATE Gi, BLOCKING the inhibition of adenylate cyclase and leading to ACCUMULATION of cAMP, with a corresponding INCREASE in blood pressure.

Cell cultures can be synchronized, or brought into the same phase of the cell cycle, by various means. For example, adding thymidine to a cell culture causes all cells to become arrested early in S phase. What is the mechanism by which thymidine treatment blocks DNA replication? Drag the terms on the left to the appropriate blanks on the right to complete the sentences.

Thymidine added to cell culture medium is taken up into cells and converted by salvage pathways to thymidine triphosphate. dTTP accumulation blocks DNA synthesis REVERSIBLY by allosteric inhibition of CDP REDUCTION by ribonucleotide reductase. So thymidine-treated cells progress through the cell cycle until they reach S phase, where they are blocked because A DEFICIENCY of dCTP makes DNA synthesis impossible. Eventually, when all cells have become blocked in early S phase, the cells are transferred to thymidine-free medium, whereupon all of the cells begin DNA synthesis in synchrony and remain synchronized for at lease ONE complete cell cycle.

Which of the following does NOT belong to a receptor tyrosine kinase family?

Thyroid receptor

Supposing that an average human consumes energy at the rate of 1500 kcal/day at rest and that long-distance running consumes energy at 10 times that rate, how long would the glycogen reserves last during a marathon run? Assume that normal glycogen reserves are about 6800 kJ for a 70 kg person. Express your answer to two significant figures and include the appropriate units.

Time = 2.6 h

To carefully prepared mitochondria were added succinate, oxidized cytochrome c, ADP, orthophosphate, and sodium cyanide. Referring to the following figure, answer the following. Why was cyanide added in this experiment?

To block cytochrome oxidase and force electrons to exit the respiratory chain at cytochrome c.

Freshly prepared mitochondria were incubated with β-hydroxybutyrate, oxidized cytochrome c, ADP, Pi, and cyanide. β-hydroxybutyrate is oxidized by an NAD+-dependent dehydrogenase. The experimenter measured the rate of oxidation of β-hydroxybutyrate and the rate of formation of ATP. What is the function of the cyanide?

To block cytochrome oxidase, so that electrons exit the chain at cytochrome c.

When pure reduced cytochrome c is added to carefully prepared mitochondria along with ADP, Pi, antimycin A, and oxygen, the cytochrome c becomes oxidized, and ATP is formed, with a P/O ratio approaching 1.0. Why was antimycin A added?

To block oxidation of endogenous substrates and to inhibit reverse electron flow (i.e., to prevent electrons flowing back up the respiratory chain).

If you were to determine the P/O ratio of oxidation of α-ketoglutarate, you would probably include some malonate in your reaction system. Why?

To block succinate dehydrogenase and measure phosphorylation resulting only from the α-ketoglutarate dehydrogenase reaction.

Triacylglycerols (TAGs) represent a major store of energy, allowing survival for up to several months under starvation conditions. Which of the statements about TAG metabolism under starvation conditions is NOT correct?

To compensate for low concentrations of carbohydrates, the body hydrolyzes TAGs to free fatty acids and glycerol, the latter of which is provided in ample supply to feed the citric acid cycle.

In the diagram below, the red arrows show the flow of energy through the electron-transport chain. Follow the flow of electrons through the electron-transport chain, and label the components of the chain. Drag the appropriate labels to their respective targets.

Top Row: 1. Coenzyme Q 2. Cytochrome c Bottom Row: 1. Complex I 2. Complex II 3. Complex III 4. Complex IV 5. ATP synthase

Briefly discuss why each of the three common forms of galactosemia involves impaired utilization of galactose. Which metabolic process is blocked in each condition? Drag the appropriate labels to their respective targets.

Top: blocked in galactokinase deficiency Bottom Left - blocked in epimerase Bottom Right - blocked in uridyl transferase deficiency

Determine whether each reaction represents a transamination or an oxidative deamination. Drag the appropriate items to their respective bins.

Transamination: - Aspartic acid is transferred from an amino acid to a keto acid. - Glutamate aminotransferase catalyzes a reaction. Oxidative deamination: - Alanine dehydrogenase, which requires a coenzyme, catalyzes a reaction. - The ammonium ion is converted to urea.

A side effect of diabetes is the production of acetone from the decarboxylation of acetoacetate due to ketoacidosis.

True

AMPK and mTOR play opposing roles in regulating metabolic activity in response to energy status.

True

All of the enzymes of the citric acid cycle are located in the mitochondrion.

True

Anaerobic glycolysis can produce ATP at a much faster rate than aerobic oxidative phosphorylation.

True

Arginine and methionine can be synthesized by mammals but are generally classed as essential amino acids.

True

Both glycolysis and gluconeogenesis are controlled by fructose 2,6-bisphosphate in response to hormones.

True

Catabolic processes involve degradation of complex molecules into simpler molecules with the net release of chemical energy.

True

Clathrin is a self-associating protein, which is able to form cage-like structures, which facilitate receptor-mediated endocytosis.

True

CoQ carries electrons from NADH-Coenzyme Q reductase and succinate dehydrogenase as well as other flavoproteins to CoQ:Cytochrome c oxidoreductase.

True

Flux through the pentose phosphate pathway is controlled mainly by the NADP+/NADPH ratio in the cell.

True

G proteins are membrane-bound proteins that bind GDP when inactive.

True

Glycogen is a major energy source for skeletal muscle contraction.

True

HMG-CoA reductase, the major control point of cholesterol biosynthesis is regulated by transcriptional and post-transcriptional mechanisms mediated by Insig proteins bound in the endoplasmic reticulum membrane.

True

In animal cells the mitochondrial electron transport chain is responsible for reoxidizing most of the NADH produced by oxidative pathways regardless of the cellular compartment in which it was produced.

True

Mono- and poly-unsaturated fatty acids are produced in the endoplasmic reticulum by desaturase enzymes.

True

Much of the regulation of gluconeogenesis is a result of the inhibition of glycolysis.

True

NAD+, coenzyme A, thiamine pyrophosphate, lipoic acid and FAD are all cofactors used in the reaction catalyzed by pyruvate dehydrogenase.

True

Nucleoside phosphorylase cleaves a glycosidic bond to yield a base and either ribose-1-phosphate or deoxyribose-1-phosphate.

True

Photosynthesis is the major source of oxygen in Earth's atmosphere.

True

Proton-driven rotation of the c-ring of the F0 unit of the F1F0 ATP synthase is required for complete passage of protons from the intermembrane space to the matrix.

True

The C4 photosynthesis pathway helps to provide CO2 to the carboxylase active site of Rubisco in conditions of high O2 concentration.

True

The anabolic and catabolic processes of cellular metabolism can be coupled by ATP.

True

The end result of a signal transduction pathway is often a change in gene expression.

True

The heart is a completely aerobic tissue that can make use of fatty acids, lactate, glucose and ketone bodies as fuel sources.

True

The initial end products of purine nucleotide biosynthesis are monophosphates.

True

The liver is the central organ in animals for processing lipoproteins whether they are derived from the diet or from biosynthetic pathways.

True

The mechanism of action of dihydrofolate reductase inhibitors used for chemotherapy is based on their ability to block the synthesis of dTMP.

True

The oxidation of branched chain amino acids shares a similar chemical strategy with β-oxidation of fatty acids.

True

Transamination reactions can be used to provide intermediates for the citric acid cycle.

True

True or false? The light-dependent reactions of photosynthesis use water and produce oxygen.

True

Water is oxidized to oxygen at the oxygen-evolving complex of photosystem II and released into the lumen of the thylakoid and diffuses out of the chloroplast.

True

With appropriate nutrition, animals maintain nitrogen intake and excretion at equivalent rates.

True

Which of the following amino acids is both glucogenic and ketogenic?

Tryptophan

How many molecules of ATP are used in the reaction, and how many are directly made?

Two are used to phosphorylate the glucose, and four are directly made from 1,3-bisphosphoglycerate and phosphoenolpyruvate.

Radioactive uracil can be used to label all of the pyrimidine residues in DNA. Using either names or structures, complete pathways for the conversion of uracil to dTTP and to dCTP. Drag the terms on the left to the appropriate blanks on the right to complete the sentences.

UDP+ATP -> UTP+ADP dCMP+H2O -> dUMP+NH3 dUMP+5,10-methylene-THF -> dTMP+DHF 2dCDP -> dCTP+dCMP Uridine+ATP -> UMP+ADP dTMP+ATP -> dTDP+ADP UMP+ATP -> UDP+ADP dTDP+ATP -> dTTP+ADP UTP+Gln+ATP -> CTP+Glu+ADP+Pi CDP+[2H] -> dCDP+H2O CTP+CMP -> 2CDP Uracil+ribose-1-phosphate -> uridine+Pi

Radioactive uracil can be used to label all of the pyrimidine residues in DNA. Match the cofactors to the reactions they are involved in and their effect on this reactions. Drag the terms on the left to the appropriate blanks on the right to complete the sentences. Terms can be used once, more than once, or not at all.

UTP+Gln+ATP -> CTP+Glu+ADP+Pi Cofactor:GTP Effect:activation dCMP+H2O -> dUMP+NH3 Cofactor:dTTP Effect:inhibition CDP+[2H] -> dCDP+H2O Cofactor:dGTP Effect:inhibition

Phosphatidylserine (PS) is considered to be an intermediate in the biosynthesis of phosphatidylethanolamine (PE) in E. coli, yet PS is not found in appreciable amounts among E. coli membrane phospholipids. Because PS must be present in the membrane to serve as an intermediate, how might you explain its failure to accumulate to a significant extent?

Under certain conditions PS could never accumulate, because, once formed by phosphatidylserine synthase, it would immediately be converted to PE by phosphatidylserine decarboxylase.

How is receptor-mediated endocytosis (RME) different from pinocytosis and phagocytosis?

Unlike pinocytosis, RME is specific; unlike phagocytosis, RME is for small particles.

Consider the following questions about glutamate dehydrogenase. Glutamate dehydrogenase has a KM for ammonia (NH3) of ~1 mM. However, at physiological pH the dominant ionic species is ammonium ion, NH4+ (pKa=9.2). Calculate the velocity (as a fraction of Vmax) that would be achieved by glutamate dehydrogenase if the total intracellular ammonia concentration (NH3+NH4+) is 100 μM (approximate physiological concentration). Assume a mitochondrial matrix pH of 8.0. Express your answer using three significant figures.

V/Vmax = 5.90*10^-3

Upon which region of the light spectrum does photosynthesis mainly depend?

Visible and infrared

Which of the following is NOT involved in the enzymatic inactivation of reactive oxygen species?

Vitamin C

During constant flight, hummingbirds expend about 2.9 kJ/hr, relying on fat oxidation as an energy source. If the birds relied on carbohydrate for this energy, how much additional weight would they have to start the flight with? Express your answer to two significant figures and include the appropriate units.

Weight = 10 g

A transamination can be summarized as an amino group (−NH3+) of an amino acid and the keto group (C=O) of an α-keto acid changing places and forming different amino acid and α-keto acid products, as in the generic reaction below: amino acid 1 + α−keto acid 1 ⟶ α−keto acid 2 + amino acid 2 Draw the α-keto acid product of the transamination of an αα-keto acid with an amino acid that has the side chain X. (insert reaction) Draw the molecule on the canvas by choosing buttons from the Tools (for bonds), Atoms, and Advanced Template toolbars. The single bond is active by default. Show formal charges on all atoms.

X--CO--COO^-

The Calvin cycle depends on inputs of chemical energy (ATP) and reductant (NADPH) from the light reactions to power the conversion of CO2 into G3P. In this exercise, consider the net conversion of 3 molecules of CO2 into 1 molecule of G3P. Drag the labels to the appropriate targets to indicate the numbers of molecules of ATP/ADP, NADPH/NADP+, and Pi (inorganic phosphate groups) that are input to or output from the Calvin cycle. Labels can be used once, more than once, or not at all.

a. 6 ATP 6 ADP b. 6 NADPH 6 NADP+ c. 6 Pi d. 2 Pi e. 3 ADP 3 ATP

In the animation, you saw that both high blood glucose levels and low blood glucose levels are stimuli that trigger specific responses by the body. These responses cause blood glucose levels to return to their normal range. In this activity, you will explore how this happens. Drag the labels to their appropriate locations on the diagram.

a. Beta cells of pancreas release insulin. b. Body cells take up more glucose. c. Liver takes up glucose and builds glycogen. d. Blood glucose levels fall. e. Alpha cells of pancreas release glucagon. f. Liver breaks down glycogen and releases glucose. g. Blood glucose levels rise.

In the sequential reactions of acetyl CoA formation and the citric acid cycle, pyruvate (the output from glycolysis) is completely oxidized, and the electrons produced from this oxidation are passed on to two types of electron acceptors. Drag the labels on the left to show the net redox reaction in acetyl CoA formation and the citric acid cycle. Note that two types of electron carriers are involved.

a. CO2 b. NADH c. FAD d. FADH2

All chemical synapses exhibit the same general sequence of events during the transmission of information across the synaptic cleft. This sequence is always initiated by an action potential that travels down the presynaptic cell (the sending neuron) to its synaptic terminal(s). Drag the labels onto the flowchart to indicate the sequence of events that occurs in the presynaptic cell (orange background) and the postsynaptic cell (blue background) after an action potential reaches a chemical synapse.

a. Ca2+ channels in presynaptic membrane open briefly b. Ca2+ ions enter presynaptic cells c. neurotransmitter-counting vesicles fuse with presynaptic membrane d. neurotransmitter released into synaptic cleft e. neurotransmitter binds to ligand-gated ion channels in postsynaptic membrane; channels open f. neurotransmitter degraded or removed from cleft; ligand-gated ion channels close

Glucose has been found to react nonenzymatically with hemoglobin, through Schiff base formation between C−1 of glucose and the amino termini of the β chains. How might this finding be applied in monitoring diabetic patients? Drag the terms on the left to the appropriate blanks on the right to complete the sentences.

Glucosylated hemoglobin accumulates in the BLOOD of diabetics and can easily be measured. Because the glycosylation reaction is COVALENT, the level of glucosylated hemoglobin reflects the level of blood glucose OVER A PERIOD OF ABOUT FOUR MONTHS, whereas a single determination of glucose reflects the value AT THE TIME OF BLOOD SAMPLING. Thus, LONGER-TERM diabetes control can be monitored with greater accuracy if glucosylated hemoglobin is measured.

Which of the following enzymes is NOT involved in the conversion of NH3 to an organic nitrogen-containing compound?

Glutamate-oxaloacetate transaminase

Which of the following is NOT involved in glycogen synthesis?

Glycogen phosphorylase

Which of the following statements is FALSE?

Glycogen phosphorylase is an enzyme target of insulin but not of glucagon.

Which statement about control of glycogen metabolism is INCORRECT?

Glycogen synthase is phosphorylated at only one site.

Adipose tissue cannot resynthesize triacylglycerols from glycerol released during lipolysis (fat breakdown). Describe the metabolic route that is used to generate a glycerol compound for triacylglycerol synthesis.

Glycolysis generates dihydroxyacetone phosphate, which is reduced to glycerol 3-phosphate.

Each of the four stages of cellular respiration occurs in a specific location inside or outside the mitochondria. These locations permit precise regulation and partitioning of cellular resources to optimize the utilization of cellular energy. (insert image) Match each stage of cellular respiration with the cellular location in which it occurs. Labels may be used once, more than once, or not at all.

Glycolysis: cytosol Acetyl CoA formation: mitochondrial matrix Citric Acid Cycle: mitochondrial matrix Oxidative Phosphorylation: inner mitochondrial membrane

Predict the product(s) for the third reaction. Draw the molecule(s) on the canvas by choosing buttons from the Tools (for bonds and charges), Atoms, and Templates toolbars.

H2C=CH2

Radioactive hypoxanthine (HX) can be used to label purine residues. Using either names or structures, complete pathways for the conversion of hypoxanthine to dATP and dGTP. Drag the terms on the left to the appropriate blanks on the right to complete the sentences.

HX+PRPP -> IMP+PPi IMP+Asp+GTP -> Adenylosuccinate+GDP+Pi dADP+ATP -> dATP+ADP dGDP+ATP -> dGTP+ADP GDP+[2H] -> dGDP+H2O Adenylosuccinate -> AMP+Fumarate IMP+NAD^+ +H2O -> XMP+NADH+H^+ ADP+[2H] -> dADP+H2O GMP+ATP -> GDP+ADP AMP+ATP -> 2ADP XMP+Gln+ATP -> GMP+Glu+ADP+Pi

The structure shown above is an intermediate in the synthesis of which biogenic amine? Spell out the full name of the compound.

Histamine

The diagram below shows the transport functions of four types of lipoproteins. For example, one type of lipoprotein transports triacylglycerols (TAGs) from the liver to the peripheral tissues. Identify the type of lipoprotein associated with each function. Drag the appropriate labels to their respective targets.

IDL HDL VLDL LDL

Sort the molecules in the glycolysis pathway based on whether they are intermediates or products in the first half of the pathway that requires energy, or are intermediates in the second half of the pathway that produces energy. Drag the appropriate items to their respective bins.

Intermediate in the first half of the pathway: - glucose-6-phosphate - fructose-6-phosphate - fructose-1,6-biphosphate - glyceraldehyde-3-phosphate - dihydroxyacetone phosphate Intermediate in the second half of the pathway: - 1,3-biphosphoglycerate - 3-phosphoglycerate - 2-phosphoglycerate - phosphoenolpyruvate

Identify each of the following reactions as an isomerization, phosphorylation, or phosphate transfer. Drag the appropriate items to their respective bins.

Isomerization: - glucose-6-phosphate -> fructose-6-phosphate - dihydroxyacetone phosphate -> glyceraldehyde-3-phosphate Phosphorylation: - glucose -> glucose-6-phosphate - fructose-6-phosphate -> fructose-1,6-biphosphate Phosphate transfer: - 1,3-biphosphoglycerate -> 3-phosphoglycerate - phosphoenolpyruvate -> pyruvate

What is the initial fate of an uncoated vesicle?

It fuses with an early endosome.

Which of the following is true of Gly-10 (pyruvate kinase), which converts phosphoenol pyruvate to pyruvate?

It is an example of substrate-level phosphorylation, producing ATP.

Examine the F1 complex of the ATP synthase from bovine heart mitochondria. What prevents this F1 complex from rotating with the F0 c-ring complex?

It is bound by the stator, which is connected to the stationary "a" subunit of F0.

In the oxidation of pyruvate to acetyl CoA, one carbon atom is released as CO2. However, the oxidation of the remaining two carbon atoms—in acetate—to CO2 requires a complex, eight-step pathway—the citric acid cycle. Consider four possible explanations for why the last two carbons in acetate are converted to CO2 in a complex cyclic pathway rather than through a simple, linear reaction. Use your knowledge of the first three stages of cellular respiration to determine which explanation is correct.

It is easier to remove electrons and produce CO2 from compounds with three or more carbon atoms that from a two-carbon compound such as acetyl CoA.

Marathon runners preparing for a race engage in "carb loading" to maximize their carbohydrate reserves. This involves eating large quantities of starchy foods. Why is starch preferable to candy or sugar-rich foods?

It is presumed that starch increases blood glucose levels less than simple sugars do. Thus, there is less stimulation of insulin secretion. Insulin would tend to retard energy mobilization from intracellular stores - something not desirable during a marathon.

The fourth reaction involving Gly-4 (aldolase) is an especially important reaction in glycolysis. Why is this?

It results in a splitting of a 6-carbon molecule into two 3-carbon molecules.

How does this compare to the energy required to drive the synthesis of ATP?

It's several times more.

From E∘′ values in Table 14.1 in the textbook, calculate the equilibrium constant for the glutathione peroxidase reaction at 37 degrees C. Express your answer using two significant figures.

Keq = 1.7*10^17

Which of the following cannot be used as a precursor for gluconeogenesis?

Leucine

Ketone bodies are exported from liver for use by other tissues. Because many tissues can synthesize ketone bodies, what enzymatic property of liver might contribute to its special ability to export these compounds?

Liver contains low levels of the enzyme that synthesizes acetoacetyl-CoA from acetoacetate, ATP, and CoA-SH. Therefore, when liver synthesizes ketone bodies, they cannot readily be activated for catabolism within the hepatocyte. Instead, they are released and utilized by other tissues.

Which of the following amino acids cannot be used to provide an intermediate of the citric acid cycle?

Lysine

Briefly describe the relationship between intracellular malonyl-CoA levels in the liver and the control of ketogenesis.

Malonyl-CoA at high levels inhibits carnitine acyltransferase I, and this inhibits ketogenesis by blocking the transport of fatty acids into mitochondria.

During constant flight, hummingbirds expend about 2.9 kJ/hr, relying on fat oxidation as an energy source. Calculate the grams of fat necessary to sustain a 20-hr nonstop flight. Express your answer to two significant figures and include the appropriate units.

Mass = 1.6 g

Based on the metabolism of such molecules, what quantity of a C−19 fatty acid would be required to produce 1 g of glucose? Express your answer to two significant figures and include the appropriate units.

Mass = 3.3 g

Considering that odd-chain fatty acids make up approximately 1%% of the fat in our diet, what quantity of fatty acids would be needed to produce 1 g of glucose? Express your answer to two significant figures and include the appropriate units.

Mass = 330 g

If palmitic acid is subjected to complete combustion in a bomb calorimeter, one can calculate a standard free energy of combustion of 9788 kJ/mol. From the ATP yield of palmitate oxidation, what is the metabolic efficiency of the biological oxidation, in terms of kilojoules saved as ATP per kilojoule released? (Ignore the cost of fatty acid activation.)

Metabolic efficiency = 35.5%

In acetyl CoA formation, the carbon-containing compound from glycolysis is oxidized to produce acetyl CoA. From the following compounds involved in cellular respiration, choose those that are the net inputs and net outputs of acetyl CoA formation. Drag each compound to the appropriate bin. If a compound is not involved in acetyl CoA formation, drag it to the "not input or output" bin. (Note that not all of the inputs and outputs of acetyl CoA formation are included.)

Net input: NAD+, pyruvate, coenzyme A Net output: acetyl CoA, NADH, CO2 Not input or output: O2, ADP, ATP, glucose

In the last stage of cellular respiration, oxidative phosphorylation, all of the reduced electron carriers produced in the previous stages are oxidized by oxygen via the electron transport chain. The energy from this oxidation is stored in a form that is used by most other energy-requiring reactions in cells. From the following compounds involved in cellular respiration, choose those that are the net inputs and net outputs of oxidative phosphorylation. Drag each compound to the appropriate bin. If a compound is not involved in oxidative phosphorylation, drag it to the "not input or output" bin. (Note that not all of the inputs and outputs of oxidative phosphorylation are listed.)

Net input: NADH, O2, ADP Net output: ATP, NAD+, water Not input or output: glucose, acetyl CoA, CO2, pyruvate, coenzyme A

In the citric acid cycle (also known as the Krebs cycle), acetyl CoA is completely oxidized. From the following compounds involved in cellular respiration, choose those that are the net inputs and net outputs of the citric acid cycle. Drag each compound to the appropriate bin. If a compound is not involved in the citric acid cycle, drag it to the "not input or output" bin. (Note that not all of the inputs and outputs of the citric acid cycle are included.)

Net input: acetyl CoA, NAD+, ADP Net output: coenzyme A, CO2, NADH, ATP Not input or output: O2, glucose, pyruvate

Under conditions where ketone bodies are being produced in the liver, how many ATPs can be produced from a molecule of palmitic acid if all resulting molecules of acetyl-CoA are converted into β-hydroxybutyrate? Express your answer using one decimal place.

Net yield = 16.0 ATP

Calculate the number of ATPs generated from the metabolic oxidation of the four carbons of acetoacetyl-CoA to CO2. Express your answer using one decimal place.

Net yield = 20.0 ATPs

Now consider the homolog derived from oxidation of an odd-numbered carbon chain, namely propionoacetyl-CoA. Calculate the net ATP yield from oxidation of the five carbons of this compound to CO2. Express your answer using one decimal place.

Net yield = 20.5 ATPs

_____ removes nitrogen from the atmosphere.

Nitrogen fixation

Suppose that you had a monoclonal antibody that recognized phosphotyrosine. How would you expect that antibody to affect insulin signaling? Drag the terms on the left to the appropriate blanks on the right to complete the sentences. Not all terms will be used.

No effect would likely be observed in a WHOLE-CELL ASSAY system because the phosphotyrosine residues are located on the INTRACELLULAR DOMAIN. But in a cell-free system, it could interfere with the interaction between insulin receptor and INSULIN RECEPTOR SUBSTRATE, IRS-1.

The substance dichlorophenyldimethylurea (DCMU) is an herbicide that inhibits photosynthesis by blocking electron transfer between plastoquinones in photosystem II. Would you expect DCMU to interfere with cyclic photophosphorylation?

No. Plastoquinones are not involved in this process.

After pyruvate is formed, it must either be converted to another molecule to enter the citric acid cycle and ultimately produce ATP by utilizing the electron-transport chain, or go through alternate pathways to produce NAD+NAD+. Identify the products formed in the pathway of pyruvate and indicate whether NADH+H+NADH+H+ or NAD+NAD+ are produced under aerobic and anaerobic respiration. Drag the appropriate labels to their respective targets.

Normal Cell: End Products - Acetyl coenzyme A NAD+/NADH+H+ - NADH + H+ produced Exercising Muscle: End Products - Lactate NAD+/NADH+H+ - NAD+ produced Yeast: End Products - Alcohol (ethanol) NAD+/NADH+H+ - NAD+ produced

Consider the large number of therapeutically effective nucleoside analogs. Why do you suppose that these drugs are administered as nucleosides, rather than as nucleotides, which would be more readily converted to the active form of the drug?

Nucleotides, being charged by the presence of phosphate, can be taken up into cells only by specific transport systems, whereas uncharged nucleosides are readily taken up into most cells by facilitated diffusion.

Draw the structure for the intermediate F. Draw the molecule on the canvas by choosing buttons from the Tool

OH--CH2--CH2--O--POOO2^-

Suppose a researcher is carrying out studies in which she adds a nonphysiological electron donor to a suspension of chloroplasts. Illumination of the chloroplasts yields oxidation of the donor. How could she tell whether photosystem I, II, or both are involved?

Observe the oxidation state of plastoquinones and plastocyanin using illumination at both 700 and 680 nm. If only p700 is involved, these will be oxidized. If only p680, they will be reduced. If both are involved, simultaneous illumination at the two wavelengths will give enhanced oxidation of the donor.

The oxidation of glucose requires 6 moles of O2 and generates 6 moles of CO2. The oxidation of palmitic acid requires 23 moles of O2 and yields 16 moles of CO2. Which of the following statements is true?

On a molar basis, the combustion of palmitic acid has greater potential to generate more ATP.

Can you tell from these absorption spectra whether red light is effective in driving photosynthesis?

One cannot tell from this graph, but because chlorophyll a does absorb red light, we can predict that it would be effective in driving photosynthesis.

Upon activation by a receptor, a G protein exchanges bound GDP for GTP, rather than phosphorylating GDP that is already bound. Similarly, the α subunit-GTP complex has a slow GTPase activity that hydrolyzes bound GTP, rather than exchanging it for GDP. Complete the description of experimental evidence that would be consistent with these conclusions. Drag the terms on the left to the appropriate blanks on the right to complete the sentences. Terms can be used once, more than once, or not at all.

One could treat a G protein-GDP complex with γ-[32P]ATP and ask whether radiolabeled GTP is synthesized. The fact that it is not indicates that the G protein cannot phosphorylate the bound GDP, at least not with ATP as a phosphate donor. One can also show that radiolabeled GTP can displace bound, unlabeled GDP under various conditions. One can also show that activation of the G protein requires the presence of GTP. Finally, one can demonstrate that the isolated α subunit has GTPase activity, by showing its ability to convert GTP to GDP and Pi in the absence of added GDP.

To carefully prepared mitochondria were added succinate, oxidized cytochrome c, ADP, orthophosphate, and sodium cyanide. Referring to the following figure, answer the following. What would the P/O ratio be if the same experiment were run with addition to the mitochondria of 2,4-dinitrophenol? Express your answer using one significant figure.

P/O ratio = 0

Referring to the following figure, predict the P/O ratio for oxidation of ascorbate by isolated mitochondria. Express your answer using one significant figure.

P/O ratio = 0.5

If you were to determine the P/O ratio of oxidation of α-ketoglutarate, you would probably include some malonate in your reaction system. Under these conditions, what P/O ratio would you expect to observe? Express your answer using two significant figures.

P/O ratio = 3.5

How would increasing PDK expression decrease the rate of mitochondrial respiration?

PDK phosphorylates the E1 subunit of PDH complex, decreasing flu x through citric acid cycle. Thus, NADH production and respiration is decreased.

Explain the basis for the following statement: As a coenzyme, pyridoxal phosphate is covalently bound to enzymes with which it functions, yet during catalysis the coenzyme is not covalently bound.

PLP forms a covalent Schiff base between the aldehyde carbon of the coenzyme and an ϵ-amino group of a lysine residue in the active site of the enzyme. This bond must be broken for the coenzyme to form a Schiff base with an amino acid substrate.

Draw the structure for the intermediate B. Draw the molecule on the canvas by choosing buttons from the Tools (for bonds and charges), Atoms, and Templates toolbars, including charges where needed.

POOO2^- --O--CH2--CH=O

Which of the following produces the largest number of reducing equivalents when oxidized?

Palmitic acid

The Foundation Figure introduces classes of chemical messengers and their modes of actions as extracellular messengers. There are several mechanisms that have evolved in which a signal can be transmitted across the cell membrane, with the simplest being the signaling molecule being able to freely diffuse across the membrane. This process is known as passive diffusion. Predict if the following chemical messengers can passively diffuse into the cell. Drag the appropriate chemical messengers to their respective bins.

Passive diffusion possible: - active vitamin D3 - progesterone - RU486 Additional transport mechanism required: - All-trans-retinoic acid - acetylcholine - cyclic AMP (cAMP) - primary structure of bovine insulin

In the disorder diabetes mellitus, the body is unable to maintain homeostasis of blood glucose. There are two major types of diabetes, type 1 and type 2. The graphs below show blood levels of glucose and insulin in three patients after each has consumed a sugary soft drink. Use the graphs to identify which patient is healthy, which has type 1 diabetes, and which has type 2 diabetes. Drag the labels to their appropriate locations above the graphs.

Patient A - type 1 diabetes Patient B - healthy Patient C - type 2 diabetes

Which of the following enzymes is NOT primarily used in salvage of nucleic acids because the pyrophosphate product is rapidly hydrolyzed?

Phosphoribosyltransferase

The action of glucagon on liver cells leads to inhibition of pyruvate kinase. What is the most probable mechanism for this effect?

Phosphorylation of pyruvate kinase by cyclic AMP-dependent protein kinase.

Which term describes ATP production resulting from the capture of light energy by chlorophyll?

Photophosphorylation

Which process produces oxygen?

Photosynthesis

Which of the following represents the CORRECT order of events in the two-photosystem light reactions of photosynthesis?

Photosystem I receives electrons from plastocyanin at the end of the electron transfer chain from the excited reaction center of photosystem II.

The light reactions require the cooperation of two photosystems to power linear electron flow from water to NADP+. Drag each item into the appropriate bin depending on whether the process is associated with Photosystem II (PS II) only, Photosystem I (PS I) only, or both PS II and PS I. Note that "electron transport chain" here refers to the electron transport chain between the two photosystems, not the one that functions after PS I.

Photosystem II (PS II) only: -oxidation of water -reduction of electron transport chain between the two photosystems Photosystem I (PS I) only: -reduction of NADP+ -oxidation of electron transport chain between the two photosystems both PS II and PS I: -reduction of primary electron acceptor -light absorption

Calculate the ATP yield from oxidation of palmitic acid, taking into account the energy needed to activate the fatty acid and transport it into mitochondria. Express your answer using one decimal place.

Yield - 106.0 ATPs

Calculate the ATP yield from oxidation of linoleic acid. Express your answer using one decimal place.

Yield = 116.0 ATPs

Calculate the ATP yield from oxidation of oleic acid. Express your answer using one decimal place.

Yield = 118.5 ATPs

Calculate the ATP yield from oxidation of stearic acid. Express your answer using one decimal place.

Yield = 120.0 ATPs

Calculate the energy yield from glucose. Express your answer using one decimal place.

Yield = 5.3 ATPs per carbon atom

Calculate the ATP yield per carbon atom oxidized. Express your answer using one decimal place.

Yield = 6.6 ATPs per carbon atom

Intramitochondrial ATP concentrations are about 5 mM, and phosphate concentration is about 10 mM. If ADP is five times more abundant than AMP, calculate the molar concentrations of ADP at an energy charge of 0.85. Express your answer to three significant figures and include the appropriate units.

[ADP] = 1.44*10^-3 M

Intramitochondrial ATP concentrations are about 5 mM, and phosphate concentration is about 10 mM. If ADP is five times more abundant than AMP, calculate the molar concentrations of AMP at an energy charge of 0.85. Express your answer to two significant figures and include the appropriate units.

[AMP] = 2.9*10^-4 M

If the intramitochondrial concentration of succinate was 10-fold higher than that of fumarate, what minimum [NAD+]/[NADH] ratio in mitochondria would be needed to make this reaction exergonic at 37 degrees C? Express your answer using two significant figures.

[NAD+]/[NADH] = 2.5*10^10

During acetyl CoA formation and the citric acid cycle, all of the carbon atoms that enter cellular respiration in the glucose molecule are released in the form of CO2. Use this diagram to track the carbon-containing compounds that play a role in these two stages. Drag the labels from the left (which represent numbers of carbon atoms) onto the diagram to identify the number of carbon atoms in each intermediate in acetyl CoA formation and the citric acid cycle. Labels may be used more than once.

a. 2 C b. 6 C c. 6 C d. 5 C e. 4 C f. 4 C g. 4 C h. 4 C i. 4 C

Which of the following generalized reaction products is NOT a result of a nucleophilic attack on a carbonyl group? 1) β-hydroxy products 2) α, β-unsaturated carbonyl products 3) β-keto products 4) carbinolamine products

Product 2 is NOT a result of a nucleophilic attack on a carbonyl group.

The following diagram shows the biosynthesis of B12 coenzymes, starting with the vitamin. DMB is dimethylbenzimidazole. Genetic deficiency in animals of enzyme B will result in excessive urinary excretion of what amino acid? Spell out the full name of the amino acid.

homocysteine

It is believed that the ratio of cyclic photophosphorylation to noncyclic photophosphorylation changes in response to metabolic demands. In each of the following situations, would you expect the ratio to increase, decrease, or remain unchanged? Chloroplasts carrying out both the Calvin cycle and the reduction of nitrite (NO−2) to ammonia (This process does not require ATP)

decrease

Once of the processes linked to receptor-mediated endocytosis is transcytosis. When might this mechanism be useful?

in transporting antibodies from mother's milk from an infant's stomach into the bloodstream

It is believed that the ratio of cyclic photophosphorylation to noncyclic photophosphorylation changes in response to metabolic demands. In each of the following situations, would you expect the ratio to increase, decrease, or remain unchanged? Chloroplasts carrying out not only the Calvin cycle but also extensive active transport

increase

It is believed that the ratio of cyclic photophosphorylation to noncyclic photophosphorylation changes in response to metabolic demands. In each of the following situations, would you expect the ratio to increase, decrease, or remain unchanged? Chloroplasts using both the Calvin cycle and the C4 pathway

increase

Xylulose-5-phosphate is an intermediate in the pentose phosphate pathway. As xylulose-5-phosphate levels rise in response to excess glucose shunting through the pentose phosphate pathway, does flux through glycolysis increase or decrease?

increase

The proximate (immediate) source of energy for oxidative phosphorylation is _______.

kinetic energy that is released as hydrogen ions diffuse down their concentration gradient

Avidin is a protein that binds extremely tightly to biotin, so avidin is a potent inhibitor of biotin-requiring enzyme reactions. Consider glucose biosynthesis from each of the following substrates and predict which of these pathways would be inhibited by avidin. Check all that apply.

lactate

N2 gas can be converted to NH3 by:

legumes.

Choose a balanced equation for the complete metabolic oxidation of each of the following. Include O2, ADP, and Pi as reactants and ATP, CO2, and H2O as products. Linoleic acid

linoleate + 25.5O2 + 118ADP + 116Pi -> 18CO2 + 117ATP + AMP + 135.5H2O

The main organ that monitors and stabilizes blood glucose is the ________.

liver

The primary gluconeogenic organ in animals is:

liver.

Shortly after a typical meal, your blood glucose will rise from its fasting level of 4.4 mM to 6.6 mM. How many grams of glucose does this increase represent? (An average adult male weighing 70 kg has a blood volume of about 5 L.) Express your answer to two significant figures and include the appropriate units.

m = 2.0 g

Which of the following statements about mTOR activity is NOT true?

mTOR is activated by AMPK under low-energy conditions.

Which folate structure (from the list below) What amino acid is synthesized as the end result of reaction mentioned in the part D? Spell out the full name of amino acid.

methionine

What coenzyme is involved?

methylcobalamin

What would you expect to happen to levels of mevalonate in human plasma if an individual were to go from a meat-containing diet to a vegetarian diet?

mevalonate levels increase

In what organelle of the cell does β oxidation of fatty acids take place?

mitochondria

Intracellular concentrations in resting muscle are as follows: fructose-6-phosphate, 1.0 mM; fructose-1,6-bisphosphate, 10 mM; AMP, 0.1 mM; ADP, 0.5 mM; ATP, 5 mM; and Pi, 10 mM. Is the phosphofructokinase reaction in muscle more or less exergonic than under standard conditions? By how much?

neither more nor less exergonic

Denitrifying bacteria convert ________ to ________.

nitrates ... nitrogen gas

Which of the following is NOT a secondary messenger? cAMP nitric oxide sn-1,2-diacylglycerol inositol 1,4,5-trisphosphate

nitric oxide

14CO2 was bubbled through a suspension of liver cells that was undergoing gluconeogenesis from lactate to glucose. Which carbons in the glucose molecule would become radioactive? Check all that apply.

none

If mevalonate labeled with 14C in the carboxyl carbon were administered to rats, which carbons of cholesterol would become labeled? Check all that apply. - C-1 - C-2 - C-3 - C-4 - C-5 - C-6 - C-7 - none

none

What is the trigger for the invagination of the vesicle?

occupied receptors accumulating in the coated pits

Choose a balanced equation for the complete metabolic oxidation of each of the following. Include O2, ADP, and Pi as reactants and ATP, CO2, and H2O as products. Oleic acid

oleate + 25.5O2 + 120.5ADP + 118.5Pi -> 18CO2 + 119.5ATP + AMP + 135.5H2O

In cellular respiration, most ATP molecules are produced by _______.

oxidative phosphorylation

The final electron acceptor of cellular respiration is _______.

oxygen

What happens to your blood pH if you go into ketosis?

pH decreases

Choose a balanced equation for the complete metabolic oxidation of each of the following. Include O2, ADP, and Pi as reactants and ATP, CO2, and H2O as products. Palmitic acid

palmitate + 23O2 + 108ADP + 106Pi -> 16CO2 + 107ATP + AMP + 122H2O

What enzyme catalyzes the conversion of fructose-6-phosphate to fructose-1,6-bisphosphate?

phosphofructokinase

Which of the following enzymes is NOT part of gluconeogenesis?

phosphofructokinase

In C3 plants the conservation of water promotes BLANK.

photorespiration

Electrons are transferred through the respiratory chain from reduced NADH or FADH2 to oxygen in small steps with each step in the pathway associated with a slightly more BLANK reduction potential.

positive

Keto-enol tautomerization is an important mechanism in glycolysis. What the mechanism for keto-enol conversion involve?

protonation of a keto group first, followed by movement of electrons from a carbon-oxygen double bond to the oxygen atom to form the hydroxyl group

ATP is the main energy-carrying molecule in living organisms. It is created and destroyed very rapidly in the cell. The vast majority of all ATP is made by a transmembrane protein complex known as ATP synthase. What is the source of energy that directly drives ATP synthase in its production of ATP?

protons diffusing through the F0 section of ATP synthase

Inosine 5'-monophosphate, which contains the base hypoxanthine, is a common intermediate for ________ nucleotide biosynthesis.

purine

The mechanism for the conversion of α-ketoglutarate to succinyl-CoA resembles the mechanism of which of the following enzymes?

pyruvate dehydrogenase complex

Among the products of glycolysis, which compounds contain energy that can be used by other biological reactions?

pyruvate, ATP, and NADH

Choose a balanced equation for the complete metabolic oxidation of each of the following. Include O2, ADP, and Pi as reactants and ATP, CO2, and H2O as products. Stearic acid

stearate + 26O2 + 122ADP + 120Pi -> 18CO2 + 121ATP + AMP + 138H2O

To determine whether isolated mitochondria exhibit respiratory control, one determines the ratio of rates of oxygen uptake in two different states. Which states?

step 3 divided by step 4

The primary factor that controls metabolism is:

substrate availability.

The glyoxylate cycle in plants and bacteria can be used for net carbohydrate synthesis from fat because isocitrate lyase yields glyoxylate and ________ without the loss of two carbons as occurs in the citric acid cycle.

succinate

To carefully prepared mitochondria were added succinate, oxidized cytochrome c, ADP, orthophosphate, and sodium cyanide. Referring to the following figure, answer the following. Write a balanced equation for the overall reaction occurring in this system, showing oxidation of the initial electron donor, reduction of the final acceptor, and synthesis of ATP.

succinate+2cytcox+ADP+Pi⟶fumarate+2cytcred+ATP+H2O

One substrate level phosphorylation occurs in the citric acid cycle in the reaction catalyzed by:

succinyl CoA synthetase.

Propionyl-CoA, the end product of β-oxidation of odd-chain fatty acids can enter the citric acid cycle after being converted to ________.

succinyl-CoA

Identify a pathway for utilization of the four carbons of acetoacetate in cholesterol biosynthesis. Carry your pathway as far as the rate-determining reaction in cholesterol biosynthesis. Drag the appropriate labels to their respective targets.

succinyl-CoA succinate acetoacetyl-CoA acetyl-CoA CoA-SH HMG-CoA 2NADPH + 2H+ 2NADP+

Formyltransferases, which are ________ dependent, are involved in several steps of purine nucleotide biosynthesis.

tetrahydrofolate

In the conversion of glyceraldehyde-3-phosphate to 1,3-bisphosphoglycerate (Gly-6), NADH is made and phosphate is attached. What drives this reaction?

that 1,3-bisphosphoglycerate is much lower in energy (G°′) than glyceraldehyde-3-phosphate

What protein does the term coated vesicle refer to?

the clathrin that forces the vesicle to invaginate

What reaction does glucose-6-phosphatase catalyze?

the conversion of glucose-6-phosphate to glucose

Which of the following is NOT a fate for pyruvate?

the conversion to malate

All amino acids can be synthesized from intermediates of the:

the glycolytic pathway, the pentose phosphate pathway and citric acid cycle.

In bacteria much of the putrescine is synthesized, not from ornithine but from arginine, which decarboxylates to yield agmatine. Formulate a plausible pathway from arginine to putrescine, using this intermediate.

the one with PLP

Which of the following is NOT stimulated by epinephrine?

the production of insulin

The same E1-E2-E3 multienzyme structure found in the pyruvate dehydrogenase and the αα-ketoglutarate dehydrogenase complexes is also used in the branched-chain αα-ketoacid dehydrogenase complex, which participates in the catabolism of branched chain amino acids. Draw the reaction product when the following substrate is acted on by the branched chain αα-keto acid dehydrogenase complex.

the structure with S-CoA

Which of the following is NOT one of the three stages of respiration?

the synthesis of pyruvate

Glycolysis is regulated primarily by:

three strongly exergonic, nonequilibrium reactions.

In mitochondrial electron transport, what is the direct role of O2?

to function as the final electron acceptor in the electron transport chain

C4 plants differ from C3 and CAM plants in that C4 plants BLANK.

transfer fixed carbon dioxide to cells in which the Calvin cycle occurs

In the molecular model of the E. coli F0 rotor, the c-ring rotates as a direct result of BLANK.

twisting of the C-terminal helix in a "c" subunit upon deprotonation of Asp61

In the pyrimidine degradative pathway, all pyrimidines undergo conversion to uracil, which undergoes an NADPH-dependent reduction. Choose a plausible structure for the product of uracil reduction (dihydrouracil).

two double bonded O (not parallel)

A futile cycle is:

two reactions or pathways that share substrates and products, and result in no net gain of ATP.

Assume a pH gradient of 4.0 units across a thylakoid membrane, with the lumen more acidic than the stroma. What is the standard free energy change per mol of protons associated with this gradient at 25 degrees C?

uH = 22.8 kJ/molH+

The proteasome is a large multisubunit ATP-dependent protease that degrades proteins that have been modified by the attachment of ________.

ubiquitin

The rotation of the F0 rotor is driven by the proton gradient. Examine the F0 rotor of E. coli. The protons enter through the entry half-channel and are passed on to a(n) BLANK.

unprotonated Asp residue on one "c" subunit's C-terminal helix

The end product of purine catabolism in humans is:

uric acid.

The light reactions of photosynthesis use BLANK and produce BLANK.

water ... NADPH

Intramitochondrial ATP concentrations are about 5 mM, and phosphate concentration is about 10 mM. Calculate ΔG for ATP hydrolysis at 37 ∘C under these conditions. The energy charge is the concentration of ATP plus half the concentration of ADP divided by the total adenine nucleotide concentration: [ATP]+1/2[ADP] / [ATP]+[ADP]+[AMP] Express your answer to three significant figures and include the appropriate units.

ΔG = -47.3 kJ/mol

Free energy changes under intracellular conditions differ markedly from those determined under standard conditions. ΔG∘′ = −30.5 kJ/mol for ATP hydrolysis to ADP and Pi. Calculate ΔG for ATP hydrolysis in a cell at 37 degrees C that contains [ATP] = 3 mM, [ADP] = 1 mM, and [Pi] = 1 mM. Express your answer with the appropriate units.

ΔG = -51.1 kJ/mol

Given what you know about the metabolic roles and/or intracellular concentration ratios of NAD+/NADH and NADP+/NADPH, would you expect ΔG (not ΔG∘′) for this enzyme to be higher, lower, or the same as ΔG for the real enzyme under intracellular conditions?

ΔG would be higher

Briefly explain your answer.

ΔG would probably be positive in vivo for an NADH-linked enzyme, because the high NAD+/NADH concentration ratio would promote the oxidation of GSH to GSSG.

According to the figure below, upon excitation the P700 reaction center is raised in potential from about +0.4 to -1.3 volts. To what value of ΔG∘′ does this correspond? Express your answer using three significant figures.

ΔG∘' = 164 kJ/mol

To carefully prepared mitochondria were added succinate, oxidized cytochrome c, ADP, orthophosphate, and sodium cyanide. Referring to the following figure, answer the following. Calculate ΔG∘′ for the overall reaction. ΔG∘′ for ATP hydrolysis is -32.2 kJ/mol, using E∘′ values from Table 14.1 in the textbook. Express your answer with the appropriate units.

ΔG∘′ = -10.3 kJ/mol

Calculate the standard free energy change as a pair of electrons is transferred from succinate to molecular oxygen in the mitochondrial respiratory chain. Express your answer to three significant figures and include the appropriate units.

ΔG∘′ = -152 kJ/mol

Calculate ΔG∘′ for the glutathione reductase reaction in the direction shown, using E∘′ values from Table 14.1 in the textbook. Express your answer with the appropriate units.

ΔG∘′ = -17.4 kJ/mol

Freshly prepared mitochondria were incubated with β-hydroxybutyrate, oxidized cytochrome c, ADP, Pi, and cyanide. β-hydroxybutyrate is oxidized by an NAD+-dependent dehydrogenase. The experimenter measured the rate of oxidation of β-hydroxybutyrate and the rate of formation of ATP. Calculate the net standard free energy change (ΔG∘′) in this system, using E′0 values from Table 14.1 in the textbook and a ΔG∘′ value for ATP hydrolysis of -32.2 kJ/mol. Express your answer with the appropriate units.

ΔG∘′ = -51.4 kJ/mol

When pure reduced cytochrome c is added to carefully prepared mitochondria along with ADP, Pi, antimycin A, and oxygen, the cytochrome c becomes oxidized, and ATP is formed, with a P/O ratio approaching 1.0. Calculate ΔG∘′ for the above reaction, using E′0 values from Table 14.1 in the textbook and a ΔG∘′ value for ATP hydrolysis of -32.2 kJ/mol. Express your answer to two significant figures and include the appropriate units.

ΔG∘′ = -78 kJ/mol

Referring to Table 14.1 in the textbook for E′0 values, calculate ΔG∘′ for oxidation of malate by malate dehydrogenase. Express your answer to two significant figures and include the appropriate units.

ΔG∘′ = 29 kJ/mol

What would ΔG∘′ be for an enzyme that oxidizes succinate with NAD+ instead of FAD? Express your answer to three significant figures and include the appropriate units.

ΔG∘′ = 67.5 kJ/mol

Suppose that a cell contained an isoform of glutathione reductase that used NADH instead of NADPH as the reductive coenzyme. Would you expect ΔG∘′ for this enzyme to be higher, lower, or the same as the corresponding value for the real glutathione reductase?

ΔG∘′ would be the same

Which of the following compounds is NOT derived from cholesterol?

α-tocopherol

Freshly prepared mitochondria were incubated with β-hydroxybutyrate, oxidized cytochrome c, ADP, Pi, and cyanide. β-hydroxybutyrate is oxidized by an NAD+-dependent dehydrogenase. The experimenter measured the rate of oxidation of β-hydroxybutyrate and the rate of formation of ATP. Write a balanced equation for the overall reaction occurring in this system (electron transport and ATP synthesis).

β−hydroxybutyrate + 2cyt cox + 2ADP + 2Pi + 4H+ ⟶ acetoacetate + 2cyt cred + 2ATP + 2H2O

Freshly prepared mitochondria were incubated with β-hydroxybutyrate, oxidized cytochrome c, ADP, Pi, and cyanide. β-hydroxybutyrate is oxidized by an NAD+-dependent dehydrogenase. The experimenter measured the rate of oxidation of β-hydroxybutyrate and the rate of formation of ATP. Indicate the probable flow of electrons in this system.

β−hydroxybutyrate ⟶ NADH ⟶ complex I ⟶ CoQ ⟶ complex III ⟶ cytochrome c

What is the longest wavelength of light that could provide enough energy per photon to pump one proton against this gradient, assuming a 20% efficiency in photosynthesis and T = 25 C?

λ =1050 nm

Each diagram below represents percent initial nucleotide level as a function of time after administration of one of the four agents (hydroxyurea, trimethoprim, thymidine or fluorodeoxyuridine), for each of the four nucleotides (semiquantitatively). Match the compounds to their effects on intracellular nucleoside triphosphate levels. Consider not only the primary effect of the compound but also any indirect effects on allosteric enzymes caused by nucleotide accumulation or depletion. Drag the appropriate labels to their respective targets.

a. Effect of thymidine on dNTP levels b. Effect of trimethoprim on bacterial rNTP levels c. Effect of fluorodeoxyuridine on dNTP levels d. Effect of hydroxyurea on dNTP levels

The four stages of cellular respiration do not function independently. Instead, they are coupled together because one or more outputs from one stage functions as an input to another stage. The coupling works in both directions, as indicated by the arrows in the diagram below. In this activity, you will identify the compounds that couple the stages of cellular respiration. Drag the labels on the left onto the diagram to identify the compounds that couple each stage. Labels may be used once, more than once, or not at all.

a. pyruvate b. NADH c. NAD+ d. NADH e. NAD+

In eukaryotes, all the reactions of photosynthesis occur in various membranes and compartments of the chloroplast. Identify the membranes or compartments of the chloroplast by dragging the blue labels to the blue targets. Then, identify where the light reactions and Calvin cycle occur by dragging the pink labels to the pink targets. Not that only blue labels should be placed in blue targets, and only pink labels should be placed in pink targets.

a. stroma b. thylakoid membrane c. cytosol d. location of Calvin cycle e. thylakoid space f. location of light reactions g. envelope membranes

Plants are best known for their ability to perform photosynthesis, the process by which light energy is converted to chemical energy in the form of sugars. But plants don't just make sugars; they use them, too. Like animals, plants must break sugars down to fuel cellular work. In this activity, you will complete a concept map showing the interrelatedness of sugar production and sugar breakdown in a plant cell. Drag the labels from the left to their correct locations in the concept map on the right. Not all labels will be used.

a. sunlight b. photosynthesis c. chloroplasts d. sugar e. chlorophyll f. carbon dioxide g. cellular respiration h. mitochondria

In the electron transport chain, protons are pumped from where to where in the mitochondria of eukaryotic cells?

from the mitochondrial matrix to the intermembrane space

Choose a balanced equation for the synthesis of sn-1-stearoyl-2-oleoylglycerophosphorylserine, starting with glycerol, the fatty acids involved, and serine.

glycerol + stearic acid + oleic acid + 3ATP + CTP + serine -> sn-1-stearoyl-2-oleylglycerophosphorylserine + ADP + 2AMP + 3PPi + CMP

Oxaloacetate is replenished via the ________ in plants and bacteria.

glyoxylate pathway

Choose a sketch of curves for reaction velocity versus [fructose-6-phosphate] for the phosphorylated and nonphosphorylated forms of PFK−2 in liver.

graph looks like a V with the nonphosphorylated line on top of the phosphorylated line

Choose the correct sketch of a curve that would describe the expected behavior of phosphofructokinase activity as a function of the adenylate energy charge.

graph looks like a right triangle

The following data, presented by G. Bowes and W.L. Ogre in J. Biol. Chem. (1972) 247: 2171-2176, describe the relative rates of incorporation of CO2 by Rubisco under N2 and under pure O2. Decide whether O2 is a competitive or uncompetitive inhibitor.

competitive

The electron-transport chain is involved in creating a proton gradient in cells. There are several complexes in the electron-transport chain. The proton gradient can be used by the ATP synthase to produce ATP. Coenzyme Q carries electrons between which stages of the electron-transport chain? Check all that apply.

complex I and complex III complex II and complex III

Which complexes of the electron transport system carry Fe-S clusters?

complexes I, II, and III

Which of the four complexes pump protons out of the mitochondrial matrix?

complexes I, III, and IV

At which site(s) are these protons pumped?

complexes III and IV

Chlorophylls, carotenes and xanthophylls absorb light in the visible part of the spectrum because they contain large BLANK double-bond systems.

conjugated

Name the amino acid residue that is used to complex zinc ions in the zinc finger motif of steroid receptors.

cysteine

In C4 and CAM plants carbon dioxide is fixed in the BLANK of mesophyll cells.

cytoplasm

Which statement about nucleotides is FALSE?

dTTP can only be synthesized from dCDP.

Match each compound with the complex it inhibits. 1) rotenone 2) CO 3) antimycin 4) cyanide 5) amytal A) complex IV B) complex III C) complex I

1:C; 2:A; 3:B; 4:A; 5:C

The purinosome contains enzymes that convert the serine hydroxymethyl group to the formyl group of 10-formyltetrahydrofolate. Choose the correct sequence of balanced equations for the reactions in this conversion.

1) Serine + THF ⟶ Glycine + 5,10−methylene−THF + H2O 2) 5,10−methylene−THF + NAD(P) + ⟶ 5,10−methenyl−THF + NAD(P)H + H+ 3) 5,10−methenyl−THF + H2O ⟶ 10−formyl−THF

Which of the following statements about hormones are correct? 1) Hormones can be peptides, steroids, or amino acid derivatives. 2) Hormones can stimulate the synthesis of target proteins through the activation of specific genes. 3) Hormones can directly activate or inhibit enzymes through the action of signal molecules. 4) Hormones can increase the cellular uptake of metabolites.

1, 2, and 4 are correct.

Which of the following molecules make up naturally occurring folates? 1) p-aminobenzoic acid 2) a tail of aspartate residues ranging from 3 to 8 residues 3) a bicyclic, heterocyclic pteridine ring 4) a tail of glutamate residues ranging from 3 to 8 residues

1, 3, and 4

With a better understanding of these three types of hormone signaling, the next question you should be asking yourself is how can a transmembrane protein either allow a molecule or ion to transverse the membrane, or how can they initiate the production of a secondary messenger or a metabolic pathway. Complete the paragraph to describe how hormone interaction with a transmembrane protein can signal the cell. Match the words in the left column to the appropriate blanks in the sentences on the right.

1. Hormone interaction with a transmembrane protein to signal the cell involves ALLOSTERY, which is the idea that the binding of a ligand to a protein at one site on the protein can cause a(n) 3-DIMENSIONAL CHANGE in the protein. 2. In the case of ion channels, the hormone binding causes a 3-DIMENSIONAL CHANGE that results in a(n) pore forming which allows an ion to transverse the membrane. 3. In the two other types, the hormone molecule that binds on the extracellular side of the transmembrane protein causes a conformational change located on the INTRACELLULAR portion of that same protein. The change result in the activation or opening of an active site. This new, active site then catalyzes the synthesis of a second messenger, or initiates the first step of a specific metabolic pathway.

In photosynthesis, a redox compound that is produced in the light reactions is required to drive other redox reactions in the Calvin cycle, as shown in this figure along with other components of photosynthesis. Drag the terms to the appropriate blanks to complete the following sentences summarizing the redox reactions of photosynthesis. Terms may be used once, more than once, or not at all.

1. In the light reactions, light energy is used to oxidize H2O to O2. 2. The electrons derived from this oxidation reaction in the light reactions are used to reduce NADP+ to NADPH. 3. The Calvin cycle oxidized the light-reactions product NADPH to NADP+. 4. The electrons derived from this oxidation reaction in the Calvin cycle are used to reduce CO2 to G3P.

The mitochondrial form of carbamoyl phosphate synthetase is allosterically activated by N-acetylglutamate. Briefly describe a rationale for this effect. Drag the terms on the left to the appropriate blanks on the right to complete the sentences.

1. N-Acetylglutamate is AN INTERMEDIATE in ornithine biosynthesis. Activity of the urea cycle requires both ornithine and carbamoyl phosphate. 2. If insufficient carbomoyl phosphate is available, ornithine WILL accumulate, and this could cause ACCUMULATION of the precursor, N-acetylglutamate. 3. This ACCUMULATION acts as a signal to stimulate carbamoyl phosphate synthesis to INCREASE urea cycle flux.

The Foundation Figure introduces classes of plasma membrane-bound receptors that play key roles in signal transduction. Signal transduction refers to the reception of an extracellular stimulus that leads to a metabolic change within a cell. Fill in the blanks in the paragraph below to given an overview regarding how extracellular stimulus can transmit signals. Match the words in the left column to the appropriate blanks in the sentences on the right.

1. There are three main methods of transmitting an extracellular signal into a cell where metabolic change occurs. The simplest might be to create a CHANNEL from one side of the membrane to the other. This pore could allow a specific extracellular molecules (often ions) into the cytosol initiating metabolic changes. 2. Of the three methods, two involve transmitting an extracellular stimulus through membrane bound proteins or receptors. An extracellular stimulus could interact with a membrane-bound protein and directly initiate a metabolic change through a SIGNAL CASCADE within the cell. 3. The second method involving transmitting an extracellular stimulus through membrane bound proteins or receptors would be that the signal induces a SECOND MESSENGER and that molecule initiates the metabolic change.

An animal's body maintains a relatively constant internal environment. How is this accomplished? It is surprisingly similar to the way a thermostat and heating system maintain a relatively constant temperature inside a room. The diagram below shows how a thermostat responds when the temperature becomes too hot or too cold. Drag the terms on the left to the appropriate blanks on the right to complete the sentences. Not all terms will be used.

1. This heating system maintains room temperature at or near a particular value, known as the SET POINT. 2. You open the window, and a blast of icy air enters the room. The temperature drops to 17 degrees Celsius, which acts as a STIMULUS to the heating system. 3. The thermostat is a SENSOR that detects the stimulus and triggers a response. 4. The heater turns on, and the temperature in the room INCREASES until it returns to the original setting. 5. The response of the heating system reduces the stimulus. This is an example of NEGATIVE feedback. 6. The way this heating system maintains a stable room temperature is similar to the way an animal's body controls many aspects of its internal environment. The maintenance of a relatively constant internal environment is known as HOMEOSTASIS.

This diagram shows the basic pattern of electron transport through the four major protein complexes in the thylakoid membrane of a chloroplast. For each step of photosynthetic electron flow from water to NADP+, drag the appropriate label to indicate whether or not that step requires an input of energy.

1. Water -> P680+ no energy input required 2. P680 -> Pq (plastoquinone) energy input required 3. Pq -> P700+ no energy input required 4. P700 -> Fd (ferredoxin) energy input required 5. Fd -> NADP+ no energy input required

Match the correct hormone class (on the left) with the correct protein (on the right) whose synthesis is increased by hormonal stimulation. 1) progesterone 2) 1,25-dihydroxy-vitamin D3 3) glucocorticoids 4) androgens A) calcium binding protein B) phosphoenolpyruvate carboxykinase C) uteroglobin D) aldolase

1:C; 2:A; 3:B; 4:D

For each glucose that enters glycolysis, _______ acetyl CoA enter the citric acid cycle.

2

In glycolysis, as in all the stages of cellular respiration, the transfer of electrons from electron donors to electron acceptors plays a critical role in the overall conversion of the energy in foods to energy in ATP. These reactions involving electron transfers are known as oxidation-reduction, or redox, reactions. Drag the words on the left to the appropriate blanks on the right to complete the sentences.

1. When a compound donates (loses) electrons, that compound becomes OXIDIZED. Such a compound is often referred to as an electron donor. 2. When a compound accepts (gains) electrons, that compound becomes REDUCED. Such a compound is often referred to as an electron acceptor. 3. In glycolysis, the carbon-containing compound that functions as the electron donor is GLUCOSE. 4. Once the electron donor in glycolysis gives up its electrons, it is oxidized to a compound called PYRUVATE. 5. NAD+ is the compound that functions as the electron acceptor in glycolysis. 6. The reduced form of the electron acceptor in glycolysis is NADH.

Indicate the chemistry involved in first step of the reaction. Include any cofactors that you think might be required for specific steps. Drag the appropriate labels to their respective targets. Note: not all labels will be used. When placing cofactors note they should be on one side of the arrow.

1. acetyl-CoA HS-CoA 2. H2O 3. H2O 4. CO2 NADH + H+ NAD+

Put the intermediates of the β oxidation of fatty acids in order from first to last. Rank from first to last in the β oxidation of fatty acids. To rank items as equivalent, overlap them.

1. fatty acyl CoA 2. trans-enoyl CoA 3. B-hydroxyacyl CoA 4. B-keotoacyl CoA 5. acetyl CoA

Arrange the following steps or substances by where they appear in the process of glucose metabolism. Rank the items from first to last. To rank items as equivalent, overlap them.

1. glycolysis 2. the citric acid cycle 3. electron transport chain 4. ATP synthase

Ammonia is a toxic byproduct of protein metabolism. To get rid of it, the body converts it to water-soluble urea through a process called the urea cycle. Put the enzymes in the urea cycle in the correct order, from first to last. Rank from first to last in the urea cycle.

1. ornithine transcarbamolyase 2. argininosuccinate synthetase 3. argininosuccinase 4. arginase

Rank the following molecules by the number of ATP molecules they produce. Rank the items from highest to lowest. To rank items as equivalent, overlap them.

1. pyruvate 2. acetyl CoA 3. NADH 4. FADH2 5. GTP

The Michaelis-Menten equation is non-linear and instead gives a rectangular hyperbola: (insert graph) The shape of this curve typically leads to underestimation of Vmax and, therefore, KM (which is dependent on Vmax. To get around these problems, the Michaelis-Menten equation can be transformed into a linear equation: 1/v=(KM/Vmax)(1/[S])+(1/Vmax) This equation can now be used to yield a linear graph called a Lineweaver-Burk plot. Below is a Lineweaver-Burk plot. Label each feature with the appropriate variable or parameter. Drag the appropriate labels to their respective targets.

1/v -1/KM 1/[S] 1/Vmax KM/Vmax

What proportion in a human running in a marathon? Express your answer using two significant figures.

10%

If oxidation of acetyl-CoA yields 10 ATPs per mole through the citric acid cycle, how many ATPs will be derived from the complete metabolic oxidation of 1 mole of alanine in a mammal?

12.5 ATPs

In cyclic photophosphorylation, it is estimated that two electrons must be passed through the cycle to pump enough protons to generate one ATP. Assuming that the ΔG for hydrolysis of ATP under conditions existing in the chloroplast is about -50 kJ/mol, what is the corresponding percent efficiency of cyclic photophosphorylation, using light of 700 nm?

14.6%

Match the tissue with its preferred fuel source: 1) brain 2) resting skeletal muscle 3) heart 4) adipose tissue 5) liver A) fatty acids B) glucose C) glucose, FA, AA

1:B; 2:A; 3:A; 4:A; 5:C

Match the biochemical action with the appropriate hormone: 1) increase in glucose uptake 2) increase in gluconeogenesis 3) increase in cAMP levels (muscle) 4) decrease in lipolysis 5) decrease in glycolysis 6) increase in ketogenesis A) glucagon B) insulin C) epinephrine

1:B; 2:A; 3:C; 4:B; 5:A; 6:A

Mutations that inactivate p53 have a recessive phenotype, whereas mutations affecting Ras are dominant. Complete the explanation of this difference. Drag the terms on the left to the appropriate blanks on the right to complete the sentences. Terms can be used once, more than once, or not at all.

A cell heterozygous for p53 could still make sufficient p53 from its one functional gene to carry out its SIGNALING FUNCTION, giving the heterozygote a wild-type function. Both genes must be inactive to block a p53 damage response. With respect to RAS, the presence of one functional gene is sufficient to initiate the abnormal GROWTH RESPONSE. Similar phenomena are seen generally in genetic analysis, where a LOSS-OF-FUNCTION mutation is typically recessive, whereas a GAIN-OF-FUNCTION mutation is dominant.

Which of the following statements about oxidative phosphorylation by ATP synthase is FALSE?

A proton gradient alone without a corresponding energy input is not sufficient to drive the synthesis of ATP.

Briefly summarize the regulatory advantage(s) of a substrate cycle in a pathway.

A substrate cycle allows amplification of a small change in an enzyme's activity; for example, a 10% change in activity gives a 50% change in flux.

Free fatty acids are released from the adipocyte after mobilization of fat stores by: - adipose triglyceride lipase - hormone sensitive lipase - monoacylglycerol lipase - lipoprotein lipase - A, B, and C

A, B, and C

A classic way to isolate thymidylate synthase-negative mutants of bacteria is to treat a growing culture with thymidine and trimethoprim. Most of the cells are killed, and the survivors are greatly enriched in thymidylate synthase-negative mutants. A. What phenotype would allow you to identify these mutants? Select all that apply. B.What is the biochemical rationale for the selection? (That is, why are the mutants not killed under these conditions?) Select all that apply. C. How would the procedure need to be modified to select mammalian cell mutants defective in thymidylate synthase?

A. - A deficiency of thymidylate synthase activity, measure in a cell-free extract. - A growth requirement of cells for thymidine. B. - The added thymidine satisfies the growth requirement for the mutants. - The mutants have negligible flux through the thymidylate synthase reaction because of the enzyme deficiency, they do not deplete their intracellular tetrahydrofolate coenzyme pools, which remain available for functions other than dTMP synthesis. C. Use methotrexate instead of trimethoprim for the selection, because trimethoprim is an ineffective inhibitor of mammalian dihydrofolate reductases.

A. Choose correct statements describing features of insulin signaling that affect glucose utilization. Select all that apply. B. A β-adrenergic response can be modulated through the actions of a receptor kinase and arrestin, because phosphorylation by the kinase desensitizes the receptor. How might signaling by a tyrosine receptor kinase, such as the insulin receptor, be modulated?

A. - The action of phosphoinositide 3-kinase stimulates an enzyme phosphorylation cascade that results in enhanced glycogen synthesis. - Over the long term insulin receptor activation stimulates protein synthesis and cell division. - Glucose uptake into cells is stimulated through mobilization of glucose transporters to the membrane. B. Modulation occurs through action of protein tyrosine phosphatases on phosphorylated tyrosine residues.

6-Diazo-5-oxonorleucine (DON) irreversibly inhibits glutamine-dependent amidotransferases. A. Which intermediate or intermediates in purine nucleotide biosynthesis would you expect to accumulate in DON-treated cells? Select all that apply. B. Which participants in pyrimidine synthesis? Select all that apply. C. Choose the most plausible mechanism by which DON inhibits these enzymes.

A. - XMP - PRPP - formylglycinamide ribonucleotide B. UTP C. The diazomethyl group on carbon no. 5 in this glutamine analog is reactive and is thought to alkylate the position in the enzyme that becomes amidated by glutamine amidotransferase reactions. This irreversibly blocks access to the site by glutamine.

A. Put the enzymes of the first half of the citric acid cycle in order from left to right. Rank the items from first to last. To rank items as equivalent, overlap them. B. Put the enzymes of the second half of the citric acid cycle in order from left to right. Rank the items from first to last. To rank items as equivalent, overlap them. C. Which of the following cannot be metabolized to make molecules that can enter the citric acid cycle?

A. - citrate synthase - aconitase - socitrate dehydrogenase - α-ketoglutarate dehydrogenase B. - succinyl CoA synthetase - succinate dehydrogenase - fumarase - malate dehydrogenase C. metal ions

Signaling molecules interact with cells through specific macromolecular receptors. For each of the four receptors identified below, check all characteristics, which accurately describe that receptor. A. An adrenergic receptor __________ Check all that apply. B. A steroid receptor __________ Check all that apply. C. The LDL receptor __________ Check all that apply. D. The insulin receptor __________ Check all that apply.

A. - is located at the cell surface - is a transmembrane protein - activation can inhibit the synthesis of glycogen - is a receptor whose biological activity involves interaction with guanine nucleotide-binding proteins B. - is a DNA-binding protein - is located in the cell interior - is a receptor whose receptor-ligand complex becomes concentrated in the nucleus - is a receptor whose the hormone-receptor complex activates specific gene transcription - is not known to act through a second messenger C. - is located at the cell surface - is associated with the protein clathrin - is a receptor whose receptor-ligand complex moves to the lysosome - action diminishes the synthesis and activity of β-hydroxy-β-methylglutaryl-CoA reductase (HMG-CoA reductase) - is not known to act through a second messenger D. - is located at the cell surface - is a transmembrane protein - is a receptor whose the hormone-receptor complex activates specific gene transcription - has a protein kinase activity

As stated in the text, bacteriophages have been discovered with the following base substitutions in their DNA. A. dUMP completely substitutes for dTMP. Choose a set of virus-coded enzyme activities that could lead to the observed substitution. Select all that apply. B. 5-hydroxymethyl-dUMP completely substitutes for dTMP. Choose a set of virus-coded enzyme activities that could lead to the observed substitution. Select all that apply. C. Complete the balanced equation for this reaction. Drag the appropriate labels to their respective targets. Labels can be used once, more than once, or not at all. D. 5-methyl-dCMP completely substitutes for dCMP E. Complete the balanced equation for this reaction. Drag the appropriate labels to their respective targets. Labels can be used once, more than once, or not at all.

A. - virus-specified inhibitor of thymidylate synthase - virus-specified dUTPase B. - virus-specified dUMP hydroxymethyltransferase - virus-specified hydroxymethyl-dUMP kinase C. dUMP -> 5,10-CH2-TFH -> THF -> hm-dUMP -> ATP -> ADP hm-dUDP -> ATP -> ADP -> hm-dUTP D. virus-specified dCMP methylase E. dCMP -> 5,10-CH2-TFH -> DHF -> CH3-dCMP -> ATP -> ADP CH3-dCDP -> ATP -> ATP -> CH3-dCTP

What is the metabolic significance of the following observations? A. Only the liver form of pyruvate kinase is inhibited by alanine. Drag the terms on the left to the appropriate blanks on the right to complete the sentences. B. Only gluconeogenic tissues contain appreciable levels of glucose-6-phosphatase.

A. 1. The liver is the MOST active gluconeogenic tissue. Since alanine is an important gluconeogenic PRECURSOR, its accumulation in liver is a signal that gluconeogenesis should be ACTIVATED. 2. In addition, alanine ACCUMULATES in muscle under conditions of high metabolic demand, where glycolysis must still function to provide ATP. B. The glucose-6-phosphatase plays a role only in primarily liver because its function is to manufacture glucose for export.

How many ATP equivalents are consumed in the conversion of each of the following to a glucosyl residue in glycogen? A. Dihydroxyacetone phosphate Express your answer as an integer. B. Fructose-1,6-biphosphate Express your answer as an integer. C. Pyruvate Express your answer as an integer. D. Glucose-6-phosphate Express your answer as an integer.

A. 1 ATP equivalent(s) B. 1 ATP equivalent(s) C. 7 ATP equivalent(s) D. 1 ATP equivalent(s)

The text states that a side effect of 5-fluorouracil therapy is its incorporation into RNA. A. Choose the correct sequence of pathways by which 5-fluorouracil could be converted to FdUMP. B. Choose the correct sequence of pathways by which 5-fluorouracil could be converted to an immediate RNA precursor.

A. 1) FU + deoxyribose−1−phosphate ⟶ Fluorodeoxyuridine (FUdR) + Pi (thymidine phosphorylase) 2) Fluorodeoxyuridine + ATP ⟶ F−dUMP + ADP B. 1) FU + ribose−1−phosphate ⟶ Fluorouridine + Pi (uridine phosphorylase) 2) Fluorouridine + ATP ⟶ F−UMP + ADP 3) F−UMP + ATP ⟶ F−UDP + ADP 4) F−UDP + ATP ⟶ F−UTP + ADP

Suppose that aconitase did not bind its substrate asymmetrically. A. What fraction of the carbon atoms introduced in one cycle as acetyl-CoA would be released in the first turn of the cycle? Express your answer as a fraction (for example, 1/2 for 0.5). B. What fraction of the carbon atoms that entered in the first cycle would be released in the second turn? Express your answer as a fraction (for example, 1/2 for 0.5).

A. 1/4 B. 3/8

A. How many NADH are produced by glycolysis? B. In glycolysis, ATP molecules are produced by ________. C. Which of these is NOT a product of glycolysis? D. In glycolysis, what starts the process of glucose oxidation? E. In glycolysis there is a net gain of ________ ATP.

A. 2 B. substrate-level phosphorylation C. FADH2 D. ATP E. 2

A. Which of these is a receptor molecule? B. A signal transduction pathway is initiated when a ________ binds to a receptor. C. Which of these is a signal molecule? D. A signal molecule is also known as a(n) ________. E. Which of these is the second of the three stages of cell signaling?

A. B B. signal molecule C. A D. ligand E. transduction

Most bacterial mutants that require isoleucine for growth also require valine. A. Why? B. Which enzyme or reaction would be defective in a mutant requiring only isoleucine (not valine) for growth?

A. Because the same enzymes are involved in comparable steps of both isoleucine and valine biosynthesis. B. Threonine dehydratase

Suppose that you made some wine whose alcohol content was 10%w/v (i.e., 10 g of ethanol per 100 mL of wine). A. The initial fermentation mixture would have had to contain what molar concentration of glucose or its equivalent to generate this much ethanol? Express your answer to two significant figures and include the appropriate units. B. Is it likely that an initial fermentation mixture would contain that much glucose? C. In what other forms might the fermentable carbon appear? Check all that apply.

A. Concentration = 1.1 M B. no C. starch, sucrose

A. Which of these acts as a second messenger? B. Which of these is responsible for initiating a signal transduction pathway? C. What role does a transcription factor play in a signal transduction pathway? D. Which of these is a membrane receptor? E. A signal transduction pathway is initiated when a ________ binds to a receptor. F. Which of these acts as a second messenger? G. Calcium ions that act as second messengers are stored in ________. H. ________ catalyzes the production of ________, which then opens an ion channel that releases ________ into the cell's cytoplasm. I. A protein kinase activating many other protein kinases is an example of ________.

A. D B. A C. By binding to DNA it triggers the transcription of a specific gene. D. B E. signal molecule F. cyclic AMP G. endoplasmic reticula H. Phospholipase C ... IP3 ... Ca2+ I. amplification

Outline the mechanism of the conversion of α-ketoglutarate to succinyl-CoA catalyzed by α-ketoglutarate dehydrogenase complex. A. Label the sequence of mechanism steps. Drag the appropriate labels to their respective targets. B. Choose correctly all products and coenzymes. Drag the appropriate labels to their respective targets. Note: not all labels will be used.

A. Decarboxylation -> Oxidation of 4-carbon group, reduction of lipoamide disulfide -> Transacylation -> Dihydrolipoyl dehydrogenase activity -> Enzymatical FADH2 reoxydation by NAD+ B. Step 1: Coenzyme - thiamine pyrophosphate Product - 4-carbon TPP derivative α-ketoglutarate Step 2: Coenzyme - lipoamide disulfide Product - dihydrolipoamide succinate Step 3: Coenzyme - coenzyme A Product - succinyl CoA Step 4: Coenzyme - FAD Product - FADH2 Step 5: Coenzyme - NAD+ Product - NADH

Wheeler and Mathews reported the concentrations of adenine nucleotides in rat liver mitochondria as follows: ATP, 5.5 mM; ADP, 5.1 mM; AMP, 1.8 mM. A. Calculate the adenylate energy charge within the mitochondrion. Express your answer using two significant figures. B. Most measurements of adenylate energy charge in whole cells or cytosol give values close to 0.9. Speculate on reasons why it might be advantageous for mitochondria to have an ADP concentration almost as high as that of ATP. C. succinyl-CoA + ADP + Pi -> succinate + ATP + CoA-SH ΔG∘′ = −2.9 kJ/mol If [Pi] within the mitochondrion is 0.05 M and succinate and succinyl-CoA are present at equimolar concentrations, what is the maximum mitochondrial concentration of CoA-SH at which the reaction can be exergonic? (Temperature is 25 degrees C.) Express your answer to two significant figures and include the appropriate units.

A. Energy change = 0.65 B. With ADP being the substrate for oxidative phosphorylation, its concentration must be relatively high in order to maintain the ATP synthase reaction at Vmax or close to it. C. [CoA-SH] = 0.15 M

The muscle isozyme of lactate dehydrogenase is inhibited by lactate. Steady-state kinetic analysis yielded the following, with lactate either absent or present at a fixed concentration. A. Pyruvate is the substrate whose concentration is varied in one plot, NADH in the other. Identify each. Label the following: A - Reciprocal of Vmax for the uninhibited enzyme. B - The line representing obtained in the presence of lactate acting as a competitive inhibitor with respect to the variable substrate. C - The line representing obtained in the presence of lactate acting as a noncompetitive inhibitor with respect to the variable substrate. D - Reciprocal of KM in the presence of lactate acting as a competitive inhibitor. Drag the appropriate labels to their respective targets. B. If KM for NADH is 2×10−5 M, then which of the following is the most appropriate NADH concentration to use when determining KM for pyruvate: 10−7 M, 10−6 M, 10−5 M, 10−4 M, or 10−3 M?

A. First graph: C A 1/NADH Second Graph: B A 1/pyruvate D B. 10^-3 M

Caffeine is an inhibitor of cyclic AMP phosphodiesterase. A. How would drinking several cups of coffee affect muscle function? B. How might it affect lipid metabolism?

A. Muscle function would be enhanced. B. Lypolysis would be promoted.

ATP has a high phosphoryl group transfer potential because:

cleavage of either of its two phosphoanhydride bonds proceeds with a large negative ΔGo' of hydrolysis.

Mammalian cells growing in culture were labeled with [3H]-thymidine to estimate the rate of DNA synthesis. The thymidine administered had a specific activity of 3000 cpm/pmol. At intervals, samples of culture were taken and acidified to precipitate nucleic acids. The rate of incorporation of isotope into DNA was 1500 cpm/10^6 cells/min. A portion of culture was taken to determine the specific activity of the intracellular dTTP pool, which was found to be 600 cpm/pmol. A. What fraction of the intracellular dTTP is synthesized from the exogenous precursor? Express your answer using two significant figures. B. What is the rate of DNA synthesis, in molecules per minute per cell of thymine nucleotides incorporated into DNA? Express your answer using two significant figures. C. How could you determine the specific activity of the dTTP pool?

A. Fraction = 20% B. Rate = 1.5*10^6 molecules/min*cell C. Prepare an acid extract of the cells (e.g., 5% trichloroacetic acid), separate the nucleotides by ion-exchange HPLC, and determine in the dTTP fraction its radioactivity and its mass, the latter from UV absorbance.

Refer to figure, which indicates ΔG for each glycolytic reaction under intracellular conditions. A. Assume that glyceraldehyde-3-phosphate dehydrogenase was inhibited with iodoacetate, which reacts with its active site cysteine sulfhydryl group. Which glycolytic intermediate would you expect to accumulate most rapidly? B. Why?

A. Fructose-1,6-biphosphate (FBP) B. As triose phosphate began to accumulate, the reverse reaction may drive both DHAP and GAP back to FBP.

Choose a balanced chemical equations for the pentose phosphate pathways. A. Ribose-5-phosphate synthesis is maximized. B. NADPH production is maximized, by conversion of the sugar phosphate products to glucose-6-phosphate for repeated operations of the pathway.

A. G6P + 2NADP+ + H2O -> R5P + CO2 + 2NADPH + 2H+ B. G6P + 12NADP+ + 7H2O -> 6CO2 + 12NADPH + 12H+ + Pi

A. Choose a pathway leading from glucose to lactose in mammary gland. B. Choose a balanced equation for the overall pathway.

A. Glc → G6P → G1P → UDP−Glc → UDP−Gal + Glc → lactose B. 2 glucose + ATP + UTP → lactose + ADP + UDP + PPi

Because of the position of arsenic in the periodic table, arsenate (AsO43−) is chemically similar to inorganic phosphate and is used by phosphate-requiring enzymes as an alternative substrate. Organic arsenates are quite unstable, however, and spontaneously hydrolyze. Arsenate is known to inhibit ATP production in glycolysis. A. Identify the target enzyme. B. Explain the mechanism of inhibition.

A. Glyceraldehyde-3-phosphate dehydrogenase B. The acyl arsenate analog of 1,3-bisphosphoglycerate spontaneously hydrolyzes.

One can identify phenylketonurics and PKU carriers (heterozygotes) by means of a phenylalanine tolerance test. One injects a large dose of phenylalanine into the bloodstream and measures its clearance from the blood by measuring serum phenylalanine levels at regular intervals. Choose corresponding curve showing relative blood phenylalanine concentration versus time that you would expect to be displayed by A. a normal individual B. PKU patient C. a heterozygote D. What kind of tolerance test could you devise to distinguish between PKU resulting from either phenylalanine hydroxylase deficiency or dihydropteridine reductase deficiency?

A. III B. I C. II D. A tryptophan tolerance test

How many high-energy phosphates are generated or consumed in converting 1 mole of glucose to lactate and 2 moles of lactate to glucose? A. Complete the sentence to indicate whether ATP equivalents are generated or consumed in converting 1 mole of glucose to lactate, and the total number. Match the words in the left column to the appropriate blanks in the sentences on the right. B. Complete the sentence to indicate whether ATP equivalents are generated or consumed in converting 2 mole of lactate to glucose, and the total number. Match the words in the left column to the appropriate blanks in the sentences on the right.

A. In converting 1 mole of glucose to lactate a total of 2 ATP equivalents are GENERATED. B. In converting 2 moles of lactate to glucose a total of 6 ATP equivalents are CONSUMED.

The glucose/glucose-6-phosphate substrate cycle involves distinct reactions of glycolysis and gluconeogenesis that interconvert these two metabolites. Assume that under physiological conditions, [ATP]=[ADP] and [Pi]=1 mM. Consider the following glycolytic reaction catalyzed by hexokinase: ATP + glucose ⇌ ADP + glucose−6−phosphate ΔG∘′ = −16.7 kJ/mol A. Calculate the equilibrium constant (K) for this reaction at 298 K. B. From the equilibrium constant, calculate the maximum [glucose-6-phosphate] / [glucose] ratio that would exist under conditions where the reaction is still thermodynamically favorable. C. The reverse of this interconversion in gluconeogenesis is catalyzed by glucose-6-phosphatase: glucose−6−phosphate+H2O⇌glucose+Pi ΔG∘′ = −13.8 kJ/mol K= 262 for this reaction. Calculate the maximum ratio of [glucose]/[glucose-6-phosphate] that would exist under conditions where the reaction is still thermodynamically favorable. D. Under what cellular conditions would both directions in substrate cycle be strongly favored? E. What ultimately controls the direction of net conversion of a substrate cycle such as this in the cell?

A. K = 845 B. [glucose-6-phosphate] / [glucose] = 845 C. [glucose] / [glucose-6-phosphate] = 2.62*10^5 D. Any condition where the [glucose]/[glucose-6-phosphate] ratio is between 262000 and 1/845. E. The relative concentrations of allosteric regulators that exert kinetic control over the enzymes in the opposing pathways.

Given what you know about the involvement of nicotinamide nucleotides in oxidative and reductive metabolic reactions, predict whether the following intracellular concentration ratios should be 1, >1, or <1. A. [NAD+]/[NADH] B. [NADP+]/[NADPH] C. Since NAD+ and NADP+ are essentially equivalent in their tendency to attract electrons, discuss how the two concentration ratios might be maintained inside cells at greatly differing values. Check all that apply.

A. Ratio is greater than 1. B. Ratio is less than 1. C. - [NADP+]/[NADPH] ratio less than unity provide concentrations that tend to drive these reactions in the direction of substrate reduction. - Because NAD+-dependent enzymes usually act to dehydrogenate (oxidize) substrates, an [NAD+]/[NADH] ratio greater than unity tends to drive reactions in that direction.

A. Predict which one of the five steps of the α-ketoglutarate dehydrogenase complex reaction is metabolically irreversible under physiological conditions. B. Explain why.

A. Step 1. Decarboxylation B. The CO2 product diffuses away from the enzyme, and does not rebind to any significant extent.

A. Which of the following best describes the mechanism by which steroid hormones control gene expression? B. Which of the following best describes the role of chaperone proteins in the regulation of gene expression by steroid hormones? C. The reason some cells respond to the presence of a steroid hormone which others do not is that ________.

A. Steroid hormones that enter the cell activate receptors. These hormone-receptor complexes then bind HREs and influence gene expression. B. Chaperone proteins maintain functionality of the receptor. C. the receptors necessary for regulation differ among cells of various types

GTPγS is a nonhydrolyzable analog of GTP. Suppose this compound were added to a cell-free system containing active components of an adrenergic signaling system. A. What consequences would you expect? B. What would be the effects on cAMP levels?

A. Stimulation of adenylate cyclase is lost much slower than in case of GTP. B. cAMP levels should increase

Suppose that a G protein undergoes a mutation that allows the exchange of bound GTP for GDP to occur in the absence of G protein binding to a receptor. A. How might this mutation affect signaling involving a GPCR? B. Which subunit of the G protein is most likely affected by the mutation?

A. Such a mutation would cause hyperactivation of downstream signaling. B. α-subunit is most likely to be affected by mutation

Choose balanced chemical equation for each of the following. A. Anaerobic glycolysis of 1 mole of sucrose, cleaved initially by sucrose phosphorylase. B. Aerobic glycolysis of 1 mole of maltose. C. Fermentation of one glucose residue in starch to ethanol, with the initial cleavage involving α-amylase.

A. Sucrose + 5Pi + 5ADP -> 4 lactate + 5ATP + 4H2O + 4H+ B. Maltose + 4Pi + 4ADP -> 4 lactate + 4ATP + 3H2O C. (Glucose)residue + 2Pi + 2ADP -> 2 ethanol + 2ATP +H2O +2CO2 + 2H+

Assume you have a solution containing the pyruvate dehydrogenase complex and all the enzymes of the TCA cycle, but none of the metabolic intermediates. When you supplement this solution with 5μ5μ moles each of pyruvate, oxaloacetate, coenzyme A, NAD+NAD+, FAD, GDP, and Pi, you find that 5μ5μ moles of CO2 are evolved and then the reaction stops. When you add alcohol dehydrogenase and its substrate acetaldehyde, additional CO2 is produced. A. How do you explain this result? B. How many μμ moles of acetaldehyde are required to allow complete oxidation of the pyruvate to 15μ moles of CO2? (Do not take into account the concentration that is already in the system.) Express your answer as an integer and include the appropriate units.

A. The addition of alcohol dehydrogenase plus its substrate allows regeneration of NAD+. Now oxidized NAD+ is available for the additional TCA reactions in which additional CO2 can be produced. B. 10 μmol

Write a one-sentence explanation for each of the following statements. A. In liver, glucagon stimulates glycogen breakdown via cAMP. Although you might expect glucagon to stimulate catabolism of the glucose formed as well, glucagon inhibits glycolysis and stimulates gluconeogenesis in liver. B. An individual with a glucose-6-phosphatase deficiency suffers from chronic hypoglycemia. C. The action of phosphorylase kinase simultaneously activates glycogen breakdown and inhibits glycogen synthesis. Drag the terms on the left to the appropriate blanks on the right to complete the sentences. D. The presence in liver of glucose-6-phosphatase is essential to the function of the liver in synthesizing glucose for use by other tissues.

A. The function of glucagon is to increase blood glucose concentration. All the mentioned processes are consistent with this function. B. The glucose-6-phosphatase deficiency would interfere with release of glucose from the liver for export to other tissues. C. Order of blanks: b a activates a b less D. Glucose-6-phosphate must be hydrolyzed in order for glucose to exit the liver cell and be exported to other tissues.

A. Given the roles of NAD+/NADH in dehydrogenation reactions and NADPH/NADP+ in reductions, would you expect the intracellular ratio of NAD+/NADH to be high or low? B. Explain your answer. C. What about the ratio of NADP+/NADPH? D. Explain your answer.

A. The intracellular ratio of NAD+/NADH should be high. B. NAD+ is oxidizing reagent, which accepts two hydride ions in dehydrogenation reaction, so high ratio of NAD+/NADH promotes this reaction. C. The intracellular ratio of NADP+/NADPH should be low. D. NADPH provides the reducing equivalents, so low ratio of NADP+/NADPH promotes the reduction reaction.

Mammals cannot undergo net synthesis of carbohydrate from acetyl-CoA, but the carbons of acetyl-CoA can be incorporated into glucose and amino acids. Present pathways by which this could come about.

Acetyl-CoA (citric acid cycle) -> oxaloacetate (PEP carboxykinase) -> PEP (gluconeogenesis) -> glucose

Which of the following is NOT a mechanism for controlling the pyruvate dehydrogenase complex oxidation of pyruvate?

Acetyl-CoA competitively inhibits E1 of the complex.

Briefly describe the biological rationale for each of the following allosteric phenomena. A. activation of pyruvate carboxylase by acetyl-CoA B. activation of pyruvate dehydrogenase kinase by NADH C. inhibition of isocitrate dehydrogenase by NADH D. activation of isocitrate dehydrogenase by ADP E. inhibition of αα-ketoglutarate dehydrogenase by succinyl-CoA F. activation of pyruvate dehydrogenase phosphatase by Ca2+

A. This is a signal that pyruvate can be shunted into gluconeogenesis instead of being oxidized in the citric acid cycle. In addition, it is a signal of unbalanced fat and carbohydrate metabolism. B. This tends to inactivate pyruvate dehydrogenase when level of NADH is sufficient for ATP production via the respiratory chain and, hence, to make pyruvate available for other purposes. C. This is a signal to reduce flux through the citric acid cycle when levels of reduced electron carriers are adequate for energy generation. D. When the energy charge is low, the accumulation of ADP provides a signal to activate the citric acid cycle and thereby increase the oxidation of nutrients for ATP production. E. This serves as a general indicator that when an energy-rich substrate (succinyl-CoA) is abundant, flux through the citric acid cycle can be reduced. F. Ca2+ is a critical signaling molecule for contraction in vertebrate muscle, which places a huge demand on ATP production.

Indicate whether each of the following statements is true or false, and briefly explain your answer. A. In general, the metabolic oxidation of protein in mammals is less efficient, in terms of energy conserved, than the metabolic oxidation of carbohydrate or fat. B. Given that the nitrogen of glutamate can be redistributed by transamination, glutamate should be a good supplement for nutritionally poor proteins. C. Arginine is a nonessential amino acid for mammals because the enzymes of arginine synthesis are abundant in liver. D. Alanine is an essential amino acid because it is a constituent of every protein.

A. True.The complete catabolism of amino acids yields CO2, H2O, and ammonia. However, the ammonia must be converted to urea for detoxification and excretion, and this requires ATP, which decreases the net ATP yield. B. False. Glutamate can be used to synthesize essential amino acids only if the carbon skeletons are available as keto acids. C. False. Arginine is used catalytically in the urea cycle in liver, and thus most of the arginine that is formed is cleaved to urea and ornithine. Little arginine is left over to meet the needs of other tissues. D. False. Although it is present in all proteins, alanine can be synthesized by mammalian cells and is not required in the diet.

CTP synthetase catalyzes the glutamine-dependent conversion of UTP to CTP. The enzyme is allosterically inhibited by the product, CTP. Mammalian cells defective in this allosteric inhibition are found to have a complex phenotype: They require thymidine in the growth medium, they have unbalanced nucleotide pools, and they have an elevated spontaneous mutation rate. A. Why do mutant cells have unbalanced nucleotide pools and require thymidine in growth medium? B. Why do mutant cells have an elevated spontaneous mutation rate?

A. Uncontrolled conversion of UTP to CTP elevates pools of cytidine and deoxycytidine nucleotides, while pools of uridine and thymidine nucleotide pools are diminished. The depletion of endogenous thymidine nucleotides explains the growth requirement for exogenous thymidine. B. The perturbed dNTP pool imbalance (dCTP/dTTP pool ratio is elevated) causes DNA replication errors, principally C incorporated opposite A, that lead to mutations.

A. What is the biochemical basis for the action of Viagra? B. What is the biochemical basis for the action of Prozac? C. Oral administration of S-adenosylmethionine has been reported to be effective in treating depression. Suggest a possible explanation. Select all that apply.

A. Viagra inhibits cyclic GMP phosphodiesterase, so cGMP levels remain high, thereby prolonging the effects of nitric oxide on regional blood flow. B. Prozac selectively inhibits reuptake of serotonin by presynaptic neurons, thereby increasing the amount of the neurotransmitter that reaches the postsynaptic cell, stimulating synaptic transmission in areas related to regulation of mood. C. - AdoMet could affect membrane fluidity through effects on phospholipid metabolism. - If AdoMet could cross the blood-brain barrier, it might conceivably increase methylation of norepinephrine to give epinephrine. - AdoMet could act through folate metabolism, increasing the pool of labile methyl groups.

In different organisms sucrose can be cleaved either by hydrolysis or by phosphorolysis. A. Calculate the ATP yield per mole of sucrose metabolized by anaerobic glycolysis starting with hydrolytic cleavage. Express your answer as an integer and include the appropriate units. B. Calculate the ATP yield per mole of sucrose metabolized by anaerobic glycolysis starting with phosphorolytic cleavage. Express your answer as an integer and include the appropriate units.

A. Yield = 4 mol B. Yield = 5 mol

Sort the following molecules based on whether or not they are biologically occurring ketone bodies. Drag each item to the appropriate bin.

Biological ketone bodies: - β-hydroxybutyrate - acetone - acetoacetate Nonbiological ketones: - 2-butanone - ethyl methyl ketone - 2-pentanone

To understand the steps of glycolysis. Glycolysis is a nearly universal pathway used by living organisms to derive energy from glucose. It involves several enzymes, and in the end produces two ATP molecules from each glucose molecule. A. Put the enzymes of glycolysis in the order in which they appear in the glycolytic pathway, from left to right. Rank from first to last. To rank items as equivalent, overlap them. B. Put the enzymes of the second half of the glycolytic pathway in the order they appear, from left to right. Rank from first to last. To rank items as equivalent, overlap them. C. One or more of the following molecules are substrates or products in the glycolytic pathway. Identify them. Check all that apply.

A. hexokinase, phosphoglucoisomerase, phosphofructokinase, fructose-1,6-biphosphate aldolase, triosephosphate isomerase B. glyceraldehyde-3-phosphate dehydrogenase, phosphoglycerate kinase, phosphoglycerate mutase, enolase, pyruvate kinase C. glucose, pyruvate, ATP, NADH

A. Choose a sketch of curves relating glycogen synthase reaction velocity to [UDP-glucose], for the a form of the enzyme, in the presence and absence of glucose-6-phosphate. B. Choose a sketch of curves relating glycogen synthase reaction velocity to [UDP-glucose], for the b form of the enzyme, in the presence and absence of glucose-6-phosphate.

A. lines super close together with -G6P on top B. lines V shaped with -G6P on top

A. What protein complex or complexes provide initial recognition of the vesicle with the target membrane? B. What is the role of Rab GTPase in the process? C. Botulinum toxin blocks neuromuscular transmission, causing paralysis. How does it do this? D. Consider the mechanism of Botulinum toxin. Is this toxin reversible? E. What is another likely role of the SNARE mechanism of vesicle fusion, in addition to secretion of material?

A. multisubunit tethering proteins and coiled-coil tethering proteins B. Rab GTPase locks the t-SNARE and v-SNARE together, facilitating fusion of the vesicle. C. Botulinum toxin cleaves the t-SNARE, blocking fusion of the vesicle. D. Yes, however the cell must regenerate new t-SNAREs. E. insertion of membrane proteins into the plasma membrane

[1−14C]Ribose-5-phosphate is incubated with a mixture of purified transketolase, transaldolase, phosphopentose isomerase, phosphopentose epimerase, and glyceraldehyde-3-phosphate. A. Predict the distribution of radioactivity in the erythrose-4-phosphate that is formed in this mixture. Check all that apply. B. Predict the distribution of radioactivity in the fructose-6-phosphate that is formed in this mixture. Check all that apply.

A. none B. C-1 C-3

Hormone action is one element of signal transduction mechanisms. The processes by which signals are transmitted from cell to cell include: A. Hormones may passively diffuse and interact with intracellular receptors; these act at the gene level, with the hormone-receptor complex affecting transcription of specific genes in the target tissue. B. Hormones that interact with plasma membrane receptors that are ion channels, with hormone-binding directly affecting membrane permeability to an ion. C. Hormones that interact with plasma membrane receptors that have a hormone-binding site on the exterior and an enzyme activity on the cytosolic side, with ligand binding stimulating that activity. D. Hormones that interact with plasma membrane receptors that act through G proteins to affect the levels of second messengers. Three types of signal transmission involve chemical messengers (hormones) that interact with plasma membrane receptors, while one diffuses passively. Predict which of the four processes of hormone action is represented by the figures below. Drag the appropriate processes to their respective bins.

A: picture of double helix B: Ion pore/Transmembrane subunits C: two other pictures D: Transmembrane domains

On the graph, identify the state that might predominate in each stage of the trace indicated with a letter.

A: state 1 B: state 2 C: state 3 D: state 4 E: state 5

The main energy-coupling compound in biochemical reactions that allows thermodynamically unfavorable processes to become favorable is ________.

ATP

In addition to the pathway form phosphatidic acid, eukaryotes can also synthesize phosphatidylcholine and phosphatidylethanolamine, starting with free choline or ethanolamine, respectively. In this salvage pathway, the phosphodiester linkage in the glycerophospholipid is formed by activating the −OH−OH of the head group instead of the diacylglycerol moiety. Propose a pathway for the synthesis of phosphatidylcholine from free choline using this salvage route. Drag the appropriate labels to their respective targets. Note: not all labels will be used.

ATP ADP phosphocholine CTP PPi CDP-choline DAG CMP

Which of the following statements about regulation of phosphofructokinase is FALSE?

ATP decreases the apparent Km for fructose-6-phosphate.

The rate of cellular respiration is regulated by its major product, ATP, via feedback inhibition. As the diagram shows, high levels of ATP inhibit phosphofructokinase (PFK), an early enzyme in glycolysis. As a result, the rate of cellular respiration, and thus ATP production, decreases. Feedback inhibition enables cells to adjust their rate of cellular respiration to match their demand for ATP. Suppose that a cell's demand for ATP suddenly exceeds its supply of ATP from cellular respiration. Which statement correctly describes how this increased demand would lead to an increased rate of ATP production?

ATP levels would fall at first, decreasing the inhibition of PFK and increasing the rate of ATP production.

Structure A is _______.

ATP synthase

Chloroplast membrane vesicles are equilibrated in a simple solution of pH 5. The solution is then adjusted to pH 8. Which of the following conclusions can be drawn from these experimental conditions?

ATP will not be produced because there is no ADP and inorganic phosphate in the solution.

Which state probably predominates in vivo in skeletal muscle fatigued from a long and strenuous workout?

state 5

Early studies of the activities of enzymes were not standardized: a given enzyme had a given activity depending on the source and amount of an enzyme. The development of Michaelis-Menten kinetics, however, allowed researchers to standardize the activities of enzymes. With Michaelis-Menten kinetics, researchers could now compare the catalytic efficiency of different enzymes on the same substrate and /or the activity of one enzyme on multiple substrates. Imagine you are looking for bacteria that can degrade petroleum in the event of an environmental oil spill. You find an enzyme in the bacteria that can break down the hydrocarbons of oil. You have identified three particularly effective bacterial strains, from each of which you isolate the enzyme and test for their activity for degrading oil. The results are below: (insert table) Which bacterial strain has the most catalytically efficient enzyme?

B

Nitrification is indicated by the letter(s) ________.

B

Which folate structure (from the list below) Is the substrate for the enzyme that is inhibited by methotrexate and trimethoprim?

B

Now consider a substrate cycle operating with enzymes X and Y in the same hypothetical metabolic pathway: A <-> B --X--> <--Y-- C <-> D Under intracellular conditions, the activity of enzyme X is 100 pmol/10^6 cells/s, and that of enzyme Y is 80 pmol/10^6 cells/s. What is the direction of metabolic flux between B and C?

B ---> C

Which of the following organs both uses and exports fatty acids as a fuel source? - Heart muscle - Liver - Adipose tissue - Resting skeletal muscle - B and C

B and C

You read a paper that describes a new inhibitor for the HIV protease. The experiments show that running the enzymatic assay in the presence of 20 nM inhibitor results in a decrease in KM from 1 uM to 0.1 uM. Vmax also decreases from 40 umol/min to 8 umol/min. Below is the graph that the authors provide for the data: (insert graph) Use this information to complete the sentence, keeping in mind that depending on the type of inhibitor, this KI may be KI or K'I.) Drag the appropriate labels to their respective targets.

Based on this information, you correctly deduce that the inhibitor must be a(n) uncompetitive inhibitor with a KI equal to 5 nM.

When pure reduced cytochrome c is added to carefully prepared mitochondria along with ADP, Pi, antimycin A, and oxygen, the cytochrome c becomes oxidized, and ATP is formed, with a P/O ratio approaching 1.0. What does this experiment tell you about the location of coupling sites for oxidative phosphorylation?

Because ATP is formed, with a P/O ratio of ~1.0, complex IV must be a coupling site (i.e., a site of proton pumping).

Freshly prepared mitochondria were incubated with β-hydroxybutyrate, oxidized cytochrome c, ADP, Pi, and cyanide. β-hydroxybutyrate is oxidized by an NAD+-dependent dehydrogenase. The experimenter measured the rate of oxidation of β-hydroxybutyrate and the rate of formation of ATP. Why is β-hydroxybutyrate added rather than NADH?

Because NADH cannot freely enter the mitochondrion.

Why is phenylketonuria resulting from dihydropteridine reductase deficiency a more serious disorder than PKU resulting from phenylalanine hydroxylase deficiency?

Because a pteridine reductase deficiency would impair all tetrahydrobiopterin-dependent reactions.

If a photosynthetic organism is illuminated in a closed, sealed environment, it is observed that the CO2 and O2 levels in the surrounding atmosphere reach a constant ratio. Suggest an explanation.

Because both O2 and CO2 are being consumed and produced by the opposing processes of photorespiration and photosynthesis, a steady-state ratio will be attained.

Lithium ion inhibits the synthesis of inositol trisphosphate by inhibiting a reaction in the breakdown of inositol trisphosphate. Choose the correct explanation of this apparent paradox.

Because inositol trisphosphate is derived from phosphatidylinositol, which in turn is derived from diacylglycerol and inositol, inositol phosphates must be completely dephosphorylated in order to be reincorporated into phosphatidylinositol for another round of synthesis.

Why does it make good metabolic sense for phosphoenolpyruvate carboxykinase, rather than pyruvate carboxylase, to be the primary target for the regulation of gluconeogenesis at the level of control of enzyme synthesis?

Because pyruvate carboxylase has two metabolic roles: replenishment of citric acid cycle intermediates and initiation of gluconeogenesis. PEPCK catalyzes the first reaction committed to gluconeogenesis.

Which antibiotic, nigericin or valinomycin, do you predict to have the greatest effect on oxidative phosphorylation when administered to respiring mitochondria? Assume the antibiotics are added to a suspension of mitochondria in equimolar amounts. Briefly explain your reasoning.

Because the electrical component (Δψ) makes a greater contribution to the proton motive force than the pH gradient (ΔpH), valinomycin would be expected to be a more effective uncoupler than nigericin.

Years ago there was interest in using uncouplers such as dinitrophenol as weight control agents. Presumably, fat could be oxidized without concomitant ATP synthesis for re-formation of fat or carbohydrate. Why was this a bad (i.e., fatal) idea?

Because the energy not used for ATP was dissipated as heat, and the subjects developed uncontrollable fevers.

You identify a new pathway in yeast that converts glucose to pyruvate. The pathway consists of the following four steps: ΔG0′(kJ/mol) / +1 +22 −37 −3A ⇌ B ⇌ C ⇌ D ⇌ EΔ Thinking back to your lessons in metabolism, you know that metabolic pathways typically run in both directions. Some of these steps use the same enzyme for both directions. Other steps must use different enzymes and, therefore, serve as a key step for regulation to control flow either up or down the pathway. Which two of the above steps would you most expect to serve as key steps for kinetic control? Check two that apply.

Between B and C Between C and D

In the experiment shown below, fibroblasts obtained from a normal subject (closed symbols) or from a patient homozygous for familial hypercholesterolemia (FH Homozygote) (open symbols) were grown in monolayer cultures. At time zero, the medium was replaced with fresh medium depleted of lipoproteins, and HMG−CoA reductase activity was measured in extracts prepared at the indicated times (panel a). Twenty-four hours after addition of the lipoprotein-deficient medium, human LDL was added to the cells at the indicated levels, and HMG−CoA reductase activity was measured at the indicated time.Based on your understanding of HMG−CoA reductase regulation, explain the following results. Cells from FH individuals exhibited high levels of reductase activity, regardless of the presence or absence of LDL in the culture medium.

Cells from FH individuals were deficient in their ability to take up cholesterol from the culture medium.

The cell has four primary ways of controlling the activity of enzymes in order to keep a tight control on the levels of metabolites. Match the following examples to the appropriate mode of enzyme regulation. Drag the appropriate items to their respective bins.

Changing enzyme concentrations: - When lactose levels rise in bacteria, genes are turned on to increase the levels of those enzymes that metabolize lactose. Reversible inhibition: - Hexokinase, a key enzyme of glycolysis, is strongly inhibited by its own product, glucose-6-phosphate. - The small molecule 2,3-BPG inhibits the binding of oxygen to hemoglobin by binding at a site distinct from that for oxygen (i.e. allosteric inhibition). Covalent modification: - Phosphorylation of the kinase domain of protein kinase C regulates its activity. - The activation of p53, a tumor suppressor, is directly regulated by acetylation of its lysine residues. Substrate cycling: - Fructose-6-phosphate (F6P) is converted into fructose-1,6-biphosphate (FBP) by phosphofructokinase; the reverse reaction is catalyzed by fructose-1,6-biphosphatase. If left unchecked, the production of FBP would lead right back to the formation of FBP again. To prevent such inefficiency, the two enzymes are regulated reciprocally.

________ is taken up by cells using receptor-mediated endocytosis, which involves the recognition of the B-100 apolipoprotein component of LDL.

Cholesterol

The following diagram shows the biosynthesis of B12 coenzymes, starting with the vitamin. DMB is dimethylbenzimidazole. Some forms of the condition described in Part B can be successfully treated by injection of rather massive doses of vitamin B12. What kind of genetic alteration in the enzyme would be consistent with this result?

Decreased affinity of enzyme C for B12 coenzyme (KM mutant)

In the experiment shown below, fibroblasts obtained from a normal subject (closed symbols) or from a patient homozygous for familial hypercholesterolemia (FH Homozygote) (open symbols) were grown in monolayer cultures. At time zero, the medium was replaced with fresh medium depleted of lipoproteins, and HMG−CoA reductase activity was measured in extracts prepared at the indicated times (panel a). Twenty-four hours after addition of the lipoprotein-deficient medium, human LDL was added to the cells at the indicated levels, and HMG−CoA reductase activity was measured at the indicated time.Based on your understanding of HMG−CoA reductase regulation, explain the following results. When cultured in the presence of LDL, normal cells showed low activity of HMG-CoA reductase. After removal of lipoproteins, including LDL, HMG-CoA reductase activities increased some 50- to 100-fold in normal cells (panel a). This high level of enzyme activity was rapidly suppressed upon addition of LDL back to normal cells (panel b).

Cholesterol is normally transported into the cell, where it regulates its own synthesis by suppressing the activity of HMG-CoA reductase, the rate-limiting enzyme.

Which of the following is NOT an electron acceptor in the mitochondrial respiratory chain?

CoQH2

Match the following schemes and graphs with the type of inhibition each describes. Keep in mind the mechanism of inhibition as you work through this exercise. Drag the appropriate items to their respective bins.

Competitive inhibition: E + S <-> ES -> E + P + I ^ | v EI Uncompetitive inhibition: E + S <-> ES -> E + P + I ^ | v EIS (No reaction) Mixed inhibition: E + S <-> ES -> E + P + + I I ^ ^ | | v v EI +S <-> EIS (No reaction)

In the previous exercise, you identified how each inhibitor interacts with the free enzyme and/or ES complex. In this exercise, match the following graphs, where the enzyme activity in the presence of an inhibitor is red, and modified Michaelis-Menten equations with the type of inhibition each describes. Keep in mind the mechanism of inhibition as you work through this exercise. Drag the appropriate items to their respective bins.

Competitive inhibition: *blue and red lines cross at 0* v = (Vmax[S]/αKM+[S]) = (Vmax[S]/KappM+[S]) Uncompetitive inhibition: *lines don't intersect" v = ((Vmax/α')[S]) / ((KM/α')[S]) = Vappmax[S]/KappM+[S] Mixed inhibition: *lines cross before the 0* v = ((Vmax/α')[S]) / ((αKM/α')+[S]) = Vappmax[S] / KappM+[S]

Inhibitors of electron transport chain are substances that bind to some of the components and block the passage of electrons at different points in the chain. This inhibition results in the accumulation of reduced forms before the inhibition point (where the inhibitor blocks the flow of electron), and oxidized forms of the components of the electrons-transport chain behind the inhibition point. Recall that reduction is gain of electrons, whereas oxidation is loss of electrons. When a component of the electron-transport chain accepts an electron, it gets reduced, and when it transfers its electrons to an acceptor, it gets oxidized and the acceptor becomes reduced. For example, in the following reaction coenzyme Q is in the oxidized form and accepts an electron to be in the reduced form. If we were to add a drug that blocks electron transfer at this step, coenzyme Q would remain oxidized, and FADH2 would remain reduced. FADH2 (reduced form) + CoQ (oxidized form) -> FAD (oxidized form) + CoQH2 (reduced form) Identify the complexes and mobile electron carriers that remain reduced and oxidized due to the following blocker/inhibitors. You can use the electron-transport chain labeled in part A to help answer this question. Drag the appropriate labels to their respective targets.

Complex I & Coenzyme Q Cytochrome c & Complex IV Complex III & Cytochrome c Complex I & Coenzyme Q

Which of the following does NOT regulate flux through the citric acid cycle?

Concentration of intermediates

The ATP that is generated in glycolysis is produced by substrate-level phosphorylation, a very different mechanism than the one used to produce ATP during oxidative phosphorylation. Phosphorylation reactions involve the addition of a phosphate group to another molecule. Sort the statements into the appropriate bin depending on whether or not they correctly describe some aspect of substrate-level phosphorylation in glycolysis.

Correct Statements: - One of the substrates is a molecule derived from the breakdown of glucose. - An enzyme is required in order for the reaction to occur. - A bond must be broken between an organic molecule and phosphate before ATP can form. Incorrect Statements: - The phosphate group added to ADP to make ATP comes from free inorganic phosphate ions. - The enzymes involved in ATP synthesis must be attached to a membrane to produce ATP.

Which of the following compounds can act as second messengers in signal transduction pathways?

Cyclic AMP

When pure reduced cytochrome c is added to carefully prepared mitochondria along with ADP, Pi, antimycin A, and oxygen, the cytochrome c becomes oxidized, and ATP is formed, with a P/O ratio approaching 1.0. Indicate the probable flow of electrons in this system.

Cyt c -> CuA -> heme a -> heme a3 -> CuB -> O2

Which of the following statements is FALSE? -All of the protein complexes of the mitochondrial respiratory chain are bound in the inner mitochondrial membrane. -During electron transport protons are pumped across the inner mitochondrial membrane from the matrix to the intermembrane space. -The citric acid cycle is linked directly to the mitochondrial respiratory chain at the site of complex II. -Oxygen is reduced to water at complex IV of the mitochondrial respiratory chain. -Cytochrome c and coenzyme Q are both soluble electron carriers that are loosely attached to the outside of the inner mitochondrial membrane.

Cytochrome c and coenzyme Q are both soluble electron carriers that are loosely attached to the outside of the inner mitochondrial membrane.

The cell uses organelles to sequester reactions and concentrate reactants. Match each stage of cellular respiration with its location in the cell. Drag the appropriate pathways to their respective bins.

Cytosol - glycolysis Mitochondrial matrix - citric acid cycle Inner mitochondrial membrane - oxidative phosphorylation

Which folate structure (from the list below) Is used in the conversion of serine to glycine?

D

Nitrogen-fixing bacteria is(are) indicated by the letter(s) _____.

D and E

Which of the following is NOT true of respiratory control?

Oxidative phosphorylation is regulated by allosteric mechanisms.

The diagram shows the mechanism for hormone signaling in which the hormone molecule binds on the outside of the membrane causing an intracellular conformational change of the transmembrane protein that result in the activation, often depicted as an opening, of an active site. This new, active site then catalyzes the synthesis of a secondary messenger molecule or the first substrate molecule of a specific metabolic pathway. where the substrate and product are as shown Complete the description of the type of signal transduction event depicted. Match the words in the left column to the appropriate blanks in the sentences on the right.

If the intracellular active site then catalyzes the synthesis of an EFFECTOR molecule, which for the system shown would be the conversion of ATP to cAMP, then this hormone signaling system might be a G PROTEIN coupled system.

Explain why glucose consumption must increase in hypoxic tissues to provide the same amount of ATP that could be produced from glucose in normoxic (normal O2 levels) tissues.

In hypoxic tissues, ATP is produced only by glycolysis, whereas in normoxic tissues, the citric acid cycle + oxidative phosphorylation can be used.

Shortly after a typical meal, your blood glucose will rise from its fasting level of 4.4 mM to 6.6 mM. Estimate the change in velocity for the liver hexokinase isozyme (IV) and the muscle hexokinase isozyme (I) that the fasting versus fed glucose concentrations would produce.

In liver, hexokinase IV velocity would increase from ~50% Vmax to ~60% Vmax. In muscle, hexokinase I is already operating at Vmax, so the velocity would not increase in this tissue.

Which of the following statements about photosynthesis is FALSE?

In plants much of the carbohydrate formed is stored as sucrose.

Under anaerobic conditions (a lack of oxygen), the conversion of pyruvate to acetyl CoA stops. Which of these statements is the correct explanation for this observation?

In the absence of oxygen, electron transport stops. NADH is no longer converted to NAD+, which is needed for the first three stages of cellular respiration.

Which of the following statements about the cyclic electron flow system is NOT true?

In this process, electrons excited in P700 are transferred to NADP+.

What would be the effect on fatty acid synthesis of an increase in intramitochondrial oxaloacetate level? Briefly explain your answer. Match the words in the left column to the appropriate blanks in the sentence on the right. Make certain the sentence is complete before submitting your answer.

Increase in citrate levels would increase generation of acetyl-CoA in cytosol, hence stimulating fatty acid synthesis.

Describe the probable effect in adipocytes of insulin-stimulated uptake of glucose into these cells. Match the words in the left column to the appropriate blanks in the sentences on the right. Make certain each sentence is complete before submitting your answer.

Increased glucose levels in the cytosol stimulate glycolysis, which provides pyruvate for oxidation in the mitochondrion. The resultant acetyl-CoA can return to the cytosol (as citrate) and generate precursors for increased fatty acid synthesis. The increased intracellular glucose also stimulates glycogen synthesis.

Which of the following is NOT an effect of glucagon?

Inhibition of glycogenolysis

Consider the following hypothetical metabolic pathway: A <-> B --X--> C <-> D Under intracellular conditions, the activity of enzyme X is 100 pmol/10^6 cells/s. Calculate the effect on metabolic flux rate (pmol/10^6 cells/s) of B -> C of the following treatments. Calculate as % change (increase or decrease). Drag the appropriate labels to their respective targets. Note: not all labels will be used.

Inhibitor that reduces activity of X by 10% (10% decrease) Activator that increases activity of X by 10% (10% increase)

Calculate the effect on direction and metabolic flux rate of the following treatments. Calculate as % change (increase or decrease). Drag the appropriate labels to their respective targets. Note: not all labels will be used.

Inhibitor that reduces activity of X by 10%: B ---> C 50% decrease Activator that increases activity of X by 10% B ---> C 50% increase Doubling the activity of enzyme Y C ---> B 400% decrease

As stated in the text, adenosine deaminase deficiency can be treated by injection of the stabilized enzyme. Why might this treatment be effective, while injection of the missing enzyme is ineffective in other conditions, such as HGPRT deficiency or a deficiency of one of the mitochondrial deoxyribonucleoside kinases?

Injected adenosine deaminase need not be taken up into cells in order to be effective because the substrates, adenosine and deoxyadenosine, are uncharged and, when they accumulate, can efflux from cells and undergo deamination in the extracellular space and then be taken up by cells. The other enzymes mentioned can only function within the cell or organelle.

From the following choices, identify those that are the inputs and outputs of the light reactions. (Recall that inputs to chemical reactions are modified over the course of the reaction as they are converted into products. In other words, if something is required for a reaction to occur, and it does not remain in its original form when the reaction is complete, it is an input.) Drag each item to the appropriate bin. If the item is not an input to or an output from the light reactions, drag it to the "not input or output" bin.

Input: NADP+, ADP, water, light Output: ATP, NADPH, O2 Not input or output: G3P, glucose, CO2

From the following choices, identify those that are the inputs and outputs of the Calvin cycle. Drag each item to the appropriate bin. If the item is not an input to or an output from the Calvin cycle, drag it to the "not input or output" bin.

Input: NADPH, ATP, CO2 Output: ADP, NADP+, G3P Not input or output: light, glucose, O2

The number of reactions, intermediates, and pathways of intermediary metabolism can at first seem overwhelming. However, these can all be simplified by viewing metabolism as a roadmap. This allows one to focus on the overall strategy, or one can focus on just a portion of the overall map. In this exercise, fill in the blanks to construct an overview of metabolism. Match the words in the left column to the appropriate blanks in the sentences on the right.

Metabolism can be bisected into two subcategories: catabolism and anabolism. Catabolism takes complex organic molecules and breaks them down into simpler molecules; this is often accompanied by the release of energy. Anabolism builds up biomolecules from simpler substances; this is often accompanied by the consumption of energy. Both pathways use simple two-carbon organic molecules (such as acetyl-CoA) as common intermediates. These intermediates can then enter the citric acid cycle, where they are further oxidized to generate carbon dioxide and reduced electron carriers, along with small amounts of ATP. The electrons on these carriers are finally sent through the electron transport chain to create a proton (H+) gradient. Release of this proton gradient results in the generation of ATP.

The following diagram shows the biosynthesis of B12 coenzymes, starting with the vitamin. DMB is dimethylbenzimidazole. Genetic deficiency in animals of enzyme C would result in excessive urinary excretion of what compound?

Methylmalonate

If algae are exposed to 14CO2 for a brief period while illuminated, the labeled carbon is initially found almost entirely in the carboxyl group of 3-phosphoglycerate. However, if illumination is continued after the label pulse, other carbon atoms become labeled. Explain.

Most of the 3PG that is initially labeled is reused in replacing RuBP, which then reenters the cycle.

Since all organs degrade amino acids, which leads to the production of ammonia, which of the following statements regarding the fate of ammonia is NOT correct?

Most tissues convert ammonia directly to urea.

NAD+ kinase catalyzes the ATP-dependent conversion of NAD+ to NADP+. How many reducing equivalents are involved in this reaction? Express your answer as an integer.

N = 0 reducing equivalent(s)

How many reducing equivalents are involved in the conversion of ferric ion to ferrous ion? Express your answer as an integer.

N = 1 reducing equivalent(s)

How many reducing equivalents are involved in reducing one molecule of oxygen gas to water? Express your answer as an integer.

N = 4 reducing equivalent(s)

Based on your answer in part A, calculate the maximum number of protons that could be pumped out of the matrix into the intermembrane space as these electrons are passed to oxygen. Assume 25 ∘C, ΔpH=1.4; Δψ= 0.175 V (matrix negative). Express your answer as an integer.

N = 6 proton(s)

Sirtuins are deacetylases that are ________-dependent and therefore can respond to the redox state of the cell.

NAD+

During electron transport, energy from _______ is used to pump hydrogen ions into the _______.

NADH and FADH2 ... intermembrane space

Which of the following molecules is the primary product of photosystem I?

NADPH

Consider the following questions about glutamate dehydrogenase. The reaction above has NH3 as a reactant, instead of NH4+, which is far more abundant at physiological pH. Why is NH3 preferred?

NH3 has an unshared electron pair that can initiate nucleophilic attack on the electron-poor carbonyl carbon atom.

Which one of these is a nitrite?

NO2-

Which one of these is a nitrate?

NO3-

From the following compounds involved in cellular respiration, choose those that are the net inputs and net outputs of glycolysis. Drag each compound to the appropriate bin. If the compound is not involved in glycolysis, drag it to the "not input or output" bin.

Net input: NAD+, ADP, glucose Net output: NADH, ATP, pyruvate Not input or output: O2, CO2, coenzyme A, acetyl CoA

Now that you have developed three signaling methods involving hormones, lets analyze them in more detail using the foundation figure as a guide. Match the words in the left column to the appropriate blanks in the sentences on the right.

One type involves hormones that interact with a plasma membrane receptor that act through a(n) G PROTEIN to affect the levels of a(n) SECOND MESSENGER, a nonprotein signaling molecule, that modulate a variety of intracellular metabolic processes. One type involves hormones that interact with plasma membrane receptors that have an extracellular hormone-BINDING SITE on a membrane bound protein, which directly initiates ENZYME ACTIVITY on the cytosolic side, with ligand binding stimulating that activity. One type involves hormones that interact with a plasma membrane receptor that is a(n) ION CHANNEL with binding directly affecting membrane permeability to an ion.

Which of the following are products of glycolysis? 1) ATP 2) pyruvate 3) NAD+ 4) NADP+

Only 1 and 2 are products of glycolysis.

Which of the following statements about metabolism are true? 1) Catabolism is the process in which complex substances are broken down, yielding a net output of energy. 2) Metabolism is an integrated system in which many of the same reactions participate in degradative (catabolic) and biosynthetic (anabolic) pathways. 3) Anabolism involves the synthesis of polymeric biomolecules and complex lipids from monomers such as fatty acids and nucleotides. 4) All organisms derive raw materials and energy from organic molecules, such as glucose.

Only 1, 2, and 3 are correct.

Under certain circumstances, glycolysis takes place in the absence of oxygen. When this happens, which of the following products may be produced? 1) lactate 2) ethanol 3) carbon dioxide 4) acetyl CoA

Only 1, 2, and 3 may be produced.

Which of the following are nucleophiles? 1) protonated imine 2) carbonyl group 3) carboxylate 4) imidazole

Only 3 and 4 are nucleophiles

Which of the following four statements is/are accurate with respect to glycolysis? 1) Glycolysis involves the conversion of monosaccharides into glucose. 2) Glycolysis involves the breakdown of glucose into glyceraldehyde-3-phosphate. 3) Glycolysis involves the conversion of pyruvate into glucose to glyceraldehyde-3-phosphate. 4) Glycolysis involves the conversion of pyruvate into acetyl CoA.

Only statement 2 is correct.

In the conversion of ethanol to acetaldehyde with concomitant conversion of NAD+ to NADH, which of the following statements is true? 1) Ethanol is the oxidant. 2) NAD+ is the reductant. 3) Acetaldehyde accepts electrons. 4) Ethanol gives up electrons.

Only statement 4 is correct.

G proteins __________. 1) are proteins that bind guanine nucleotides 2) can either stimulate or inhibit downstream enzymes such as adenylyl cyclase 3) are activated by hydrolyzing GTP to GDP 4) are in an inactive state when GTP is bound

Only statements 1 and 2 are correct.

How do FAs of different carbon lengths cross into the mitochondrial matrix for oxidation? 1) Small FAs of 10 carbons or fewer can freely diffuse through the membrane. 2) Large FAs are cleaved into two smaller units that are actively transported across the membrane. 3) A membrane-bound transferase catalyzes the conversion of long-chain FAs to FA-carnitine esters, which are transported across the membrane via a specific carrier. 4) The breakdown of large FAs starts outside the matrix, allowing for smaller particles to pass easily through the membrane.

Only statements 1 and 3 are correct.

Which of the following statements about the conversion of acetyl CoA to citrate are true? 1) The reaction is spontaneous. 2) The reaction involves the nucleophilic attack of the enol of acetyl-CoA onto the carbonyl carbon of oxaloacetate. 3) A serine residue hydrolyzes the thioester bond of citroyl-CoA to yield a protein ester intermediate that is subsequently hydrolyzed to citrate. 4) The final product, citrate, has two prochiral substituents.

Only statements 1, 2, and 4 are correct.

Which of the following statements about the ubiquitination of proteins are true? 1) Ubiquitin is a relatively small protein that is widely expressed in eukaryotic cells. 2) It is covalently attached to target proteins in an ATP-dependent reaction. 3) A carboxyl group of ubiquitin forms amide bonds with free amines found on the target protein. 4) Target selection is determined by ubiquitin-protein ligases.

Only statements 1, 2, and 4 are correct.

Which of the following statements about ATP are true? 1) ATP has reactive phosphoester bonds. 2) ATP has a high Eact for hydrolysis with water. 3) ATP is kinetically stable but thermodynamically unstable. 4) ATP spontaneously phosphorylates other molecules because it has a high group transfer potential.

Only statements 2 and 3 are true.

Glycogen synthesis and breakdown are regulated primarily at the hormonal level. However, important nonhormonal mechanisms also control the rates of synthesis and mobilization. Describe these nonhormonal regulatory processes. Match the words in the left column to the appropriate blanks in the sentences on the right. Make certain each sentence is complete before submitting your answer.

Order of blanks: AMP AMP AMP glucose-6-phosphate glucose-6-phosphate nonhormonal hormonal

To carefully prepared mitochondria were added succinate, oxidized cytochrome c, ADP, orthophosphate, and sodium cyanide. Referring to the following figure, answer the following. List the sequence of electron carriers in this system.

Succinate⟶FAD⟶Q⟶cytb⟶cytc1⟶cytc

Which of the following enzymes catalyzes a reaction of the citric acid cycle that does NOT produce reduced electron carriers?

Succinyl CoA synthetase

The precise mechanism of ammonia toxicity to the brain is not known. Speculate on a possible mechanism, based on possible effects of ammonia on levels of key intermediates in energy generation. Drag the appropriate labels to their respective targets. Note: not all labels will be used.

Succinyl-CoA -> Oxaloacetate -> Citrate -> α-Ketoglutarate -> NH3 -> Glu -> NH3 -> Gln

Which of the following techniques can be used to quantify protein-protein interactions?

Surface plasmon resonance

Describe how the action of hexokinase IV helps the liver to buffer the level of blood glucose. Drag the terms on the left to the appropriate blanks on the right to complete the sentences.

The HIGH K0.5 of hexokinase IV allows the liver to control the rate of glucose PHOSPHORYLATION over a wide range of glucose concentrations. Accumulation of glucose-6-phosphate activates the b form of GLYCOGEN SYNTHASE and promotes GLYCOGEN DEPOSITION. By several mechanisms, the liver also senses when blood glucose levels are low and promotes GLYCOGEN CONSUMPTION.

How would increasing LDH expression increase the rate of glycolysis?

The LDH reaction ensures a continuous supply of oxidized NAD+NAD+ to allow glycolysis to operate at an increased rate.

[methyl-14C]Pyruvate was administered to isolated liver cells in the presence of sufficient malonate to block succinate dehydrogenase completely. After a time, isocitrate was isolated and found to contain label in both carbon 2 and carbon 4:

The action of pyruvate carboxylase on the labeled pyruvate yields oxaloacetate labeled such that, when these carbons proceed through the citric acid cycle, so CC-4 of isocitrate is labeled in addition.

When the identical subunits of chicken liver fatty acid synthase are dissociated in vitro, all of the activities can be detected in the separated subunits except for the β-ketoacyl synthase reaction and the overall synthesis of palmitate. Explain these observations.

The acyl-ACP produced by one subunit undergoes the next round of reductive two-carbon addition on the other subunit.

Which of the following statements regarding different fuel sources and the tissues that make use of them is NOT true?

The brain relies almost exclusively on its glycogen stores to provide its only fuel source glucose.

Which of the following statements about the reaction catalyzed by phosphoglycerate kinase is NOT true?

The phosphate group from C-3 of 1,3-BPG is transferred to ADP.

Will all molecules of fructose-1,6-biphosphate carry two 14C atoms? Explain.

They may carry 0, 1, or 2 labeled carbons, depending on what combination of labeled and unlabeled trioses has been used in their formation.

Daniel Arnon and coworkers carried out experiments with intact, salt-washed chloroplasts to study photophosphorylation. When the chloroplasts were illuminated in the presence of ADP+Pi, ATP was produced, but oxygen was not produced or nor consumed. ATP formation was not accompanied by a measurable electron transport involving any external electron donor or acceptor. The overall reaction for this result is: ADP+Pi⟶(hν) ATP When NADP+ was included in addition to the ADP + Pi, illumination of the intact chloroplasts again resulted in the photophosphorylation of ADP to ATP. In addition, the NADP+ was reduced to NADPH + H+ and O2 was produced. Moreover, the light-induced reduction of NADP+ was greatly decreased if ADP + Pi were omitted. The equation for this reaction is: NADP+ + H2O + ADP + Pi ->(hv) NADPH + H+ + ATP + 1/2O2 Briefly describe the mechanism(s) of these two types of photophosphorylation that explain all of these results. Drag the appropriate items to their respective bins.

The first reaction: -cyclic photophosphorylation -only PS I is involved -the electrons are transferred back into the electron transport chain -O2 is not produced -reduced NADPH is not produced -the internal electron transport stimulates ATP synthesis The second reaction: -the electrons is PS I are used to reduce NADP+ -the omission of ADP + Pi decreases the reduction of NADP+ -NADPH is produced -both PS I and PS II are involved -noncyclic photophosphorylation -O2 is produced

If only chlorophyll a were involved in the light reactions, would blue light (wavelength about 490 nm) be effective in driving photosynthesis?

The graph indicates that chlorophyll a absorbs very little blue light, so we can predict that blue light would not be effective.

Hexokinase IV has a Km value that is substantially higher than the Km for other hexokinases. What is the significance of this fact?

The higher Km reflects lower affinity for its substrate, and therefore this kinase is only operational when glucose levels are high.

Which set of reactions uses H2O and produces O2?

The light-dependent reactions

Which of the following statements best describes the relationship between the light-dependent and light-independent reactions of photosynthesis?

The light-dependent reactions produce ATP and NADPH, which are then used by the light-independent reactions.

What is the importance of the light-independent reactions in terms of carbon flow in the biosphere?

The light-independent reactions turn CO2, a gas, into usable carbon in the form of sugars.

Proteolysis increases during the early phases of fasting, but later it decreases as the body adapts to using alternative energy sources. Given that feedback control mechanisms have not been described for intracellular proteases, how might you explain these apparent changes in protease activity? Drag the terms on the left to the appropriate blanks on the right to complete the sentences.

The measured proteolysis represents the SUM OF RATES of protein synthesis and breakdown. The actual rate of protein breakdown DOES NOT RISE, but in the early stage of a fast, the utilization of amino acids in catabolic pathways REDUCES the concentrations needed to support protein synthesis at rates that counterbalance breakdown. Later, as FATTY ACIDS AND KETONES are used more for energy, AMINO ACIDS are spared for this purpose and are more readily available to be used for protein synthesis.

Which statement about oxidative phosphorylation is NOT true?

The mechanisms of phosphorylation and oxidation are directly coupled.

Two-dimensional gel electrophoresis of proteins in a cell extract provides a qualitative way to compare proteins with respect to intracellular abundance. Describe a quantitative approach to determine the number of molecules of an enzyme per cell. Match the words in the left column to the appropriate blanks in the sentences on the right. Make certain each sentence is complete before submitting your answer.

The most direct way is to PURIFY the enzyme to homogeneity and determine both ITS MOLECULAR WEIGHT AND TURNOVER NUMBER. Then, from the activity observed in a crude extract, one can calculate the number of active enzyme molecules needed to achieve that activity (assuming that the extract DOES NOT CONTAIN inhibitors of the activity). Another approach is to treat the extract with a specific antibody to the enzyme of interest and quantitate the PROTEIN immunoprecipitated. One still needs to know THE MOLECULAR WEIGHT to convert this value to number of molecules of enzyme.

Engelmann counted the number of bacteria that were attracted to the algal filament associated with each color of light. As shown in the image below, most of the bacteria were attracted to the regions of the alga illuminated by red or violet-blue light. This distribution of bacteria shows that red and violet-blue wavelengths are most effective in driving photosynthesis. By measuring oxygen production with aerotactic bacteria, Engelmann described an action spectrum for photosynthesis. The action spectrum (indicated by the black line plot in the image above) shows the relative effectiveness of each color of light in driving photosynthesis. What assumptions did Engelmann make in order to conclude that red and violet-blue light were more effective than green light in driving photosynthesis? Select the two that apply.

The number of bacteria clustered at each wavelength (color) was approximately proportional to the amount of oxygen being produced by that portion of the alga. The distribution of chloroplasts within each algal cell was approximately the same.

The rate of O2 production by the light reactions varies with the intensity of light because light is required as the energy source for O2 formation. Thus, lower light levels generally mean a lower rate of O2 production. In addition, lower light levels also affect the rate of CO2 uptake by the Calvin cycle. This is because the Calvin cycle needs the ATP and NADPH produced by the light reactions. In this way, the Calvin cycle depends on the light reactions. But is the inverse true as well? Do the light reactions depends on the Calvin cycle? Suppose that the concentration of CO2 available for the Calvin cycle decreased by 50% (because the stomata closed to conserve water). Which statement correctly describes how O2 production would be affected? (Assume that the light intensity does not change.)

The rate of O2 production would decrease because the rate of ADP and NADP+ production by the Calvin cycle would decrease.

Which statement about the interconversion of DHAP to GAP is NOT true?

The reaction is an oxidation catalyzed by triose phosphate isomerase.

If a photosynthetic organism is illuminated in a closed, sealed environment, it is observed that the CO2 and O2 levels in the surrounding atmosphere reach a constant ratio. What factor would you think primarily determines the value of this ratio?

The relative affinity of rubisco for CO2 and O2.

The text states that ATP is synthesized primarily by energy metabolism, whereas other nucleoside triphosphates are formed from the action of nucleoside diphosphate kinase. What additional pathway exists for GTP synthesis?

The succinyl-CoA synthetase reaction in the citric acid cycle synthesizes GTP from GDP.

In the citric acid cycle as well as in other processes, the human body takes advantage of the reactivity of thioesters. Which of the following statements about thioesters is NOT true?

The thioester has extensive π overlap between the S and carbonyl carbon.

Enolase has a strict requirement for two Mg2+ ions in its active site. Propose a role for these ions in the catalytic mechanism of the enzyme.

The two Mg2+ ions neutralize the negative charges on the substrate and lower the pKa of the C-2 proton, making it easier to abstract.

An action spectrum plots the rate of photosynthesis at various wavelengths of visible light, and it shows that blue light with a wavelength of about 490 nm is effective in driving photosynthesis. Based on this information and the absorption spectra shown at left, what role may chlorophyll b and carotenoids play in photosynthesis?

These pigments are able to absorb more wavelength of light (and thus more energy) than chlorophyll a alone can absorb. As part of light-harvesting complexes in photosystems, they broaden the range of light that can be used in the light reactions.

β-adrenergic receptors are subject to phosphorylation at several serine residues by β-adrenergic receptor kinase, and this is followed by binding to the phosphorylated sites by a protein called arrestin. Why do you think might be the purpose of these modifications?

These processes control receptor activity and modulate the adrenergic response.

Which statement about the Hill reactions is NOT true?

These reactions are thermodynamically favorable, even in the absence of light energy.

You obtain the pigments called carotenoids in your diet when you eat carrots. Why do carotenoids appear yellow and orange?

They absorb blue/green light and reflect yellow and red wavelengths of light.

Which statement about uncouplers is NOT true? - 2,4-DNP is an example of an uncoupler. - They allow ATP synthesis with no electron transport. - They allow electron transport without ATP synthesis. - Their mechanism of action is basically to transport protons back into the matrix, preventing the required buildup of charge on the other side.

They allow ATP synthesis with no electron transport.

What is the biological significance of the light-independent reactions of photosynthesis?

They convert carbon dioxide to sugar.

Fluoroacetate functions as a poison by what mechanism?f

This compound is first converted to 2-fluorocitrate, which subsequently inhibits aconitase.

Chemical synapses transmit information from the sending (presynaptic) cell to the receiving (postsynaptic) cell in the form of neurotransmitters. The release of neurotransmitter into the synaptic cleft and the resulting changes in the membrane potential of the postsynaptic cell (postsynaptic potentials) all depend on the presence of several different types of gated ion channels and the distribution of these channels in the pre- and postsynaptic cells. The image here illustrates a chemical synapse. Drag the labels onto the table to indicate which type(s) of gated ion channels are found in each membrane associated with a chemical synapse. (The letters in the table refer to the lettered structures in the image above.) Labels can be used once, more than once, or not at all.

a. voltage-gated Na+ voltage-gated K+ b. voltage-gated Na+ voltage-gated K+ voltage-gated Ca2+ c. ligand-gated d. none e. voltage-gated Na+ voltage-gated K+

In the degradation of amino acids in muscle NH4+ is carried to the liver for conversion into urea by:

alanine.

Tetrahydrofolate is a coenzyme involved in the mobilization and utilization of single carbon functional units in the: - metabolism of serine, glycine, methionine, and histidine. - biosynthesis of thymine. - biosynthesis of purine nucleotides. - biosynthesis of formylmethionyl-tRNA - all of the above

all of the above

The shuttle system for transfer of acetyl-CoA generated in the mitochondrial matrix to the cytosol includes: - the production of NADPH by malic enzyme - ATP hydrolysis - citrate from the citric acid cycle - citrate lyase in the cytosol - all of the above

all of the above

Hexokinase, phosphofructokinase and pyruvate kinase are each subject to ________ control.

allosteric

Nitrifying bacteria convert _____ to _____.

ammonium ... nitrites

Nuclear receptors:

are transcriptional regulators.

2-Bromopalmitoyl-CoA inhibits the oxidation of palmitoyl-CoA by isolated mitochondria but has no effect on the oxidation of palmitoylcarnitine. What is the most likely site of inhibition by 2-bromopalmitoyl-CoA?

carnitine palmitoyltransferase I

Photosynthesis relies predominantly on light in the visible and near-infrared region of the electromagnetic spectrum. This light is capable of __________.

causing transitions in the electronic states of organic molecules that can drive reactions

One of the assumptions that Engelmann made was that the sun (his light source) emits equal numbers of photons at each wavelength in the visible spectrum. In reality, the sun's emission peaks in the yellow region of the spectrum, with relatively fewer photons emitted in the red and violet-blue regions. Recall that the action spectrum from Engelmann's experiment plotted the rate of photosynthesis (as measured by oxygen production) versus wavelength. In each of the following graphs, the black line shows Engelmann's original action spectrum deduced from the distribution of aerotactic bacteria around the alga. Which red line shows the same action spectrum corrected for the unequal number of photons emitted across the visible spectrum?

red line on top lines only touch around the 550 nm mark

The energy of a photon absorbed by an antenna molecule of the photosystem finds it way to a chlorophyll molecule in the reaction center of a light-harvesting complex by a process of:

resonance transfer and electron transfer.

Draw the structure of the next intermediate formed in this reaction. Draw the molecule on the canvas by choosing buttons from the Tools (for bonds and charges), Atoms, and Templates toolbars, including charges where needed.

same as molecule at the top but without the COO^-

The membrane-bound receptor in G protein-coupled signal transduction is quite separate to the effector enzyme which catalyses the formation of an intracellular ________.

second messenger

Draw a plausible mechanism for the oxidation of dihydrolipoamide to lipoamide by the E3 subunit (dihydrolipoamide dehydrogenase) of pyruvate dehydrogenase complex. Drag the appropriate labels to their respective targets.

see picture

Draw a plausible mechanism for the oxidative decarboxylation of isocitrate by isocitrate dehydrogenase. Draw all missing reactants and/or products in the appropriate boxes by placing atoms on the canvas and connecting them with bonds. Add charges where needed. Electron flow arrows should start on an atom or a bond and should end on an atom, bond, or location where a new bond should be created. Use the molecular structure of nicotinamide instead of NAD+NAD+ structure.

see picture

Draw the first missing intermediate. Draw the molecule on the canvas by choosing buttons from the Tools (for bonds and charges), Atoms, and Templates toolbars, including charges where needed.

see picture

Draw the second missing intermediate. Draw the molecule on the canvas by choosing buttons from the Tools (for bonds and charges), Atoms, and Templates toolbars, including charges where needed.

see picture

Draw the third missing intermediate. Draw the molecule on the canvas by choosing buttons from the Tools (for bonds and charges), Atoms, and Templates toolbars, including charges where needed.

see picture

Predict the product(s) for the first reaction. Draw the molecule(s) on the canvas by choosing buttons from the Tools (for bonds and charges), Atoms, and Templates toolbars.

see picture

Predict the product(s) for the first reaction. Draw the molecule(s) on the canvas by choosing buttons from the Tools (for bonds and charges), Atoms, and Templates toolbars. To write a radical, right-click on an atom and choose Atom Properties. (Mac users: Use an equivalent for right-clicking.) Then, clear the check mark to enable the Alias field before entering a symbol "R".

see picture

Predict the product(s) for the second reaction. Draw the molecule(s) on the canvas by choosing buttons from the Tools (for bonds and charges), Atoms, and Templates toolbars.

see picture

Pyruvate carboxylase is thought to activate CO2 by ATP, through formation of carboxyphosphate as an intermediate. Propose a mechanism for the formation of this intermediate. Draw all missing reactants and/or products in the appropriate boxes by placing atoms on the canvas and connecting them with bonds. Add charges where needed. Electron flow arrows should start on an atom or a bond and should end on an atom, bond, or location where a new bond should be created.

see picture

Which state probably predominates in heart muscle most of the time?

state 3

Which state probably predominates in resting skeletal muscle of a well-nourished animal?

state 4


Set pelajaran terkait

minerals & water (quiz 3; test 3)

View Set

SET 1: PRACTICE QUESTIONS, ANSWERS & EXPLANATIONS

View Set

Unit 1 - My face (New English Adventure 1)

View Set

Chapter 7; Written Communications

View Set

Unit 1: The Global Tapestry - Dar al Islam -- 2021

View Set

AP Human Geography Models & Theories

View Set

World History 1: SOL Review - important people

View Set